You are on page 1of 192

Dr. K. N.

Modi University,
Newai Rajasthan

QUESTION BANK
(2nd Semester B.Tech) Volume - I
NAME BRANCH ROLL NO :_____________________________________ : ____________________________________ : ____________________________________

Campus: Plot No. INS-1, RIICO Industrial Area Ph-II, Newai, Distt. Tonk, Rajasthan 304021 (India) E-mail : contact@dknmu.org Website : www.dknmu.org Contact No. 08875010002/08 1|Page

CONTENTS
S. No. 1. 2. 3. 4. 5. 6. 7. 8. 9. 10. 11. Particulars CONTINUOUS ASSESSMENT GUIDELINE FOR SUBMISSION OF ASSIGNMENT CURRICULUM PLAN SYLLABUS : MATHEMATICS - II FUNDAMENTALS OF ELECTRICAL ENGINEERING BASIC ELECTRONICS QUESTION BANK MATHEMATICS FUNDAMENTALS OF ELECTRICAL ENGINEERING BASIC ELECTRONICS Page No. 3-5 6-7 8 9 10 11 12 13 80 81 135 136 - 192

2|Page

Continuous Assessment
All courses undertaken by students are evaluated during the semester using internal system of continuous assessment. The students are evaluated on class /tutorial participation, assignment work, lab work, class tests, mid-term tests, quizzes and end semester examinations, which contribute to the final grade awarded for the subject. Students will be notified at the commencement of each courses about the evaluation methods being used for the courses and weightages given to the different assignments and evaluated activities. In order to make the evaluation system as similar and transparent with any of the globally reputed educational institutions like N.I.Ts, I.I.Ts etc. the Dr. K. N. Modi University Academic Council has adopted the grading practices. Here marks obtained in the continuous assessment and end semester examination are added together and a 10-point grading system will be used to award the student with on overall letter grade for the course (subject). Distribution of Marks Courses without Practical Components Continuous Assessment - 25 Mid Term Examination - 15 End Term Examination - 60 __________________________________________ Total : 100 Courses with Practical Components only Continuous Assessment - 30 Mid Term Examination (Practical) - 20 End Term Examination (Practical) - 50 ___________________________________________ Total : 100

Letter Grading system


Final evaluation of course is carried out on a TEN POINT grading system. Performance Grade and Grade Points are as shown below.

Table 1
Marks 91 to 100 81 to 90 71 to 80 61 to 70 51 to 60 41 to 50 Less than 41 Absent in the University Final Examination Grade Value 10 9 8 7 6 5 0 0 Grade A+ A B C E E F I Description Out Standing Excellent Very Good Good Average Fair Fail Incomplete

3|Page

A student who earns a minimum of 5 grade Point (E grade) in a course (subject) is declared to have successfully completed the course, and is deemed to have earned the credits assigned to that course. A course successfully completed cannot be repeated. A student should have appeared for the end semester examination of the prescribed course of study (mere appearance in the continuous assessment test is not sufficient) to be eligible for the award of the degree in the course. If a student is eligible for but-fails to appeared in the end semester examination, he/she will be awarded an I grade (in complete) on the grade sheet. For all practical purposes an IGrade is treated as an F. If a student is not eligible to appear in the end semester examination owing to his/her not fulfilling the minimum attendance requirements, he may be permitted to re-register for those courses in which he/she had attendance shortage, at the next available opportunity. Grade Point Average (GPA) &Cumulative Grade Point Average (CGPA) Each course grade will be converted into a specific number of points associated with the grade as mentioned in Table I. Here points are weighted with the number of credits assigned to a course. The Grade Point Average (GPA) is the weighted average of grade points awarded to a student. The Grade Point Average for each semester will be calculated only for those students who have passed all the courses of that semester. The weighted average of GPAs of all semester that the student has completed at any point of time is the Cumulative Grade Point Average (CGPA) at that point of time. CGPA upto any semester will be calculated only for those students who have passed all the courses upto that semester. Calculation of GPA and CGPA : Example: Table 2 Courses Mathematics Chemistry Physics Language Lab TOTAL Credits 3 3 3 2 11 Letter Grade C B A B Grade Value 7 8 9 8 Credit Value 3x7 3x8 3x9 2x8 TOTAL Grade Points 21 24 27 16 88

In this case GPA =

Total Grade Points Credits

88 11

8.0

Suppose the GPAS in two successive semesters are 7.0 and 8.0 with 26 and 24 respective course credits, then the

4|Page

CGPA =

7x26+8x24 = 374 26+24 50

= 7.48

After the results are declared, grade cards will be issued to each student which will contain the list of courses for that semester and the grades obtained by the student, as well as GPA of that semester. However, a conversion factor of 9.1, will be included, enabling students and future employers for transforming CGPA into percentage of marks at par with the existing practices of I.I.Ts, N.I.Ts and A.I.C.T.E.

Minimum Eligibility Requirements in Dr. K. N. Modi University for proceeding to the next academic year of study.
A First year Student of Dr. K. N. Modi University satisfying the below mentioned requirements is eligible to study in the 3rd Semester of next academic year. Serial No. 1. Course B.Tech Minimum Requirement Pass with Minimum E Grade in Four Theory Papers & Pass in Four Laboratory Papers in the I & II Semester ( Combined)

A Second year Student of Dr. K. N. Modi University satisfying the below mentioned requirements is eligible to study in the Vth Semester of the next academic year. Serial No. 1. Minimum Requirement Pass with Minimum E Grade in Four Theory Papers & Pass in Four Laboratory Papers in the III rd & IV Semester (Combined) A Third year Student of Dr. K. N. Modi University satisfying the below mentioned requirements is eligible to study in the VIIth Semester of the next academic year. Serial No. 1. Course B.Tech Minimum Requirement Pass with Minimum E Grade in Four Theory Papers & Pass in Four Laboratory Papers in the Vth & VI Semester (Combined) Course B.Tech

Proficiencies:
Extra-curricular activities as listed below will be offered to students of all programmes. These activities will run in both semesters and evaluated. Activities will be graded as Out Standing/Excellent/ Very Good/Good/ Average/ Fair/Fail/Incomplete. The extracurricular activities are sports, cultural: 1. 4. 7. 10. 13. 16. 19. 22. 25. Tennis Badminton Squash Football Cricket Volleyball Basketball Kho - Kho Any other activity with prior approval of the President. 2. 5. 8. 11. 14. 17. 20. 23. Athletics Gymnastics Gardening Electronics N.S.S. Music and Dramatics Debate Throw Ball 3. 6. 9. 12. 15. 18. 21. 24. Table Tennis Chess Organization& Management Fine Arts & Paintings Rovering & Rangering Model and Sculptures Equestrian Race Yoga & Meditation

5|Page

Guideline for submission of assignment


A. Assignments (Theory)
Following are the guidelines of assignments, their evaluation. Assignment means a set of work, tasks and/or numerical problems given to the student, on the basis of topics recently covered in the class as homework to be solved and submitted, within a week. Each assignment should require 5 6 hours work to be done by the student. The Date of Submission (DoS) duly announced on the on the Date of Allotment (DoA) to the student and duly mentioned in the Student Record Register. 1. In a multiple-section course, the preparation, duplication and distribution is the responsibility of the Course Coordinator. a. Allotment of an assignment should be made in the academic calendar of the semester. b. The Date of Submission (Dos) of an assignment should be the tutorial in the prescribed week wherever applicable. Where tutorials are not scheduled, submission should be in the first lecture of the subsequent week. 2. Assignment should NOT have any descriptive questions (that can be directly copied from a book or from the internet). However, in those course(s) where only descriptive problems are feasible, prior approval for the same is to be sought from the President in writing mentioning the justification for the same. 3. The effective teaching for term is generally of 14 weeks. The minimum number of assignments to be given throughout the term is five. No assignment should be due in the last week of the term. 4. The assignment is to be submitted on or before the Date of Submission (DoS) as announced. 5. The evaluation of numerical assignment will be done through a test based on the assignment. The test would comprise of one of the questions from the assignment to be solved in the class. The following process may be adopted for the purpose: a) Ask students to bring the assignment sheets to the class (along with calculators, if required). b) Take 60 sheets of A4 sheets. On each sheet write the roll number of a student and the question number from the assignment that he/she has to solve. Different question for adjacent students. Make student sit roll-number-wise, so that no two adjacent students are given the same problem. c) Give student just sufficient time to solve the problem assuming that they have done the assignment at home. d) Make sure they have submitted the assignment before the start of the test and that they are not copying. 6. Marks to be awarded in these assignment-quizzes only if the assignment is submitted in time. 7. For non-numeric assignments the rest could have questions based on the assignment. Make sure that there are multiple shuffled sets for these tests to prevent copying. The comments on the assignments are mandatory. The marks are to be allotted to submission and test separately. 8. Minimal time to be given to the students to attempt the said tests because they should not require any thinking for solving these as they have already solved these problems earlier. 9. The evaluated assignments/tests are to be shown to the student (as done in scrutiny of the End Term Examination answer sheets) and are to be retained by the instructor. The evaluated assignments/test should be retained till the next assignment is evaluated. This is to permit checking by designated authority at any instance. 10. The assignment-based tests should be given on the Date of Assignment (DoS). Only the students who have submitted the assignment on time should be allowed to take the test, otherwise, the student should be awarded ZERO marks for the same. 11. This procedure is to be announced and explained to the students in the very first class. The importance of timely submission of assignments should be explained. 12. No deviation from this policy is permitted except with a written prior approval from the president.

6|Page

B. Laboratory Assessments
Following are the guidelines for the conduct and evaluation of practical in all courses with laboratory components: 1. A practical is where a student is taken to a laboratory and is asked to perform a set of task on the given computer, equipment or on a setup comprising of devices or components. This includes on-the spot conduct of an activity to derive desired results and to report the findings. 2. A student will have to maintain record of the performance done in the lab in the bound lab notebook. 3. The lab notebook should be maintained in the format of a lab journal, where (in general) the aim of the experiment, the observations, calculations, results ad discussions are reported. These should not have any description like method etc, unless the method itself is the aim of the experiment. 4. Error analysis forms an essential part of the lab journal. 5. A student will be evaluated on every experiment/lab performed. The components of practical assessment are to be re-defined, notified to the student and to be strictly adhered to. 6. The practical assessment components and distribution is to be as follows For each experiment Component Conduct/Performance/Execution Written Record a. Observations b. Analysis c. Error Analysis d. Results and Discussions Viva Voce % 20 50

30

7. This policy is to be duly announced and explained to the students in the very first class where the importance of timely submission of lab reports must be emphasized. The students are also to be informed about secure retention of assignments/tests/term papers after their scrutiny.

7|Page

CURRICULUM PLAN
B.Tech 2nd Semester (Common to all B.Tech Branches)
CONTACT HOURS NAME OF SUBJECT L
3 5 5 5 5 5 4

SUBJECT CODE
01BT201 01BT202 01BT203 01BT204 01BT205 01BT206 01BT207

T
2 0 0 0 0 0 0

P
0 0 0 0 0 0 0

MATHEMATICS II FUNDAMENTALS OF ELECTRICAL ENGINEERING BASIC ELECTRONICS COMPUTER CONCEPTS AND PROGRAMMING IN C APPLIED ENGINEERING PHYSICS MANUFACTURING PROCESS PROFESSIONAL ETHICS

PRACTICAL CODE 01BP202 01BP203 01BP204 01BP206 01BP1011

SUBJECT NAME ELECTRICAL ENGINEERING LAB ELECTRONICS LAB C PROGRAMMING LAB MECHANICS LAB CO-CURRICULAR ACTIVITIES

L 0 0 0 0 0

T 0 0 0 0 0

P 2 2 2 2 2

8|Page

Dr. K. N. Modi University,


Newai Rajasthan

SYLLABUS
For 2nd Semester

B.Tech Programme

9|Page

SYLLABUS OF MATHEMATICS - II
CODE: 01BT201
Credits - 3

Course Objective: To make students acquainted with various methods of curve tracing, solving differential equations, Laplace and Fourier Transforms. Laplace transform is used to solve differential equations. Fourier Transform converts non sinusoidal wave form into sinusoidal wave forms used in solving electrical and electronic problems. These studies will help the students for resolving students complete problems in deriving solution for various engineering applications. UNIT I : ORDINARY DIFFERENTIAL EQUATIONS AND APPLICATION: First order differential equations exact and reducible to exact form. Linear differential equations of higher order with constant coefficients. Solution of simultaneous differential equations. Variation of parameters, Solution of homogeneous differential equations. UNIT II - SERIES SOLUTION AND SPECIAL FUNCTIONS : Series solution of second order differential equation. Bessels functions, Legendre functions. UNIT III - LAPLACE TRANSFORMATION: Existence condition, Laplace transform of standard functions, Properties, Inverse Laplace transform of functions using partial fractions, Convolution and convolution theorem. Solving linear differential equations using Laplace transform. Unit step function, Impulse function and Periodic function and their transforms. UNIT IV - FOURIER SERIES: Periodic function, Trignometric series, Fourier series of period 2 Eulers formulae, function having arbitrary period ,change of interval ,even and odd function, half range, sine and cosine series. First order Lagranges linear partial diff. equation. UNIT V - APPLICATION OF DIFFERENTIAL EQUATIONS: Method of separation of variable, Wave equation up to two dimension, Laplace equation in two dimensions, Heat conduction equation up to two dimensions. Text books: 1 H. K . Dass : S. Chand Publication. 2. Jain, R. K.and Iyengar, S. R. K. Advanced Engineering Mathematics, Narosa, 2003 (2nd Ed). 3. Differential Equation; Shanti Narayan & Gorakh Prasad.

10 | P a g e

FUNDAMENTALS OF ELECTRICAL ENGINEERING CREDIT : 3 CODE: 01BT202 COURSE OBJECTIVE: To teach students and make them understand about electrical quantities, Measuring Equipments used in industries as well as in houses. UNIT I - Electrical Engineering Materials Materials for Resistor , Inductors and Capacitors, Types of Capacitors, Types of substances Viz; dielectric, piezo electric , diamagnetic paramagnetic and ferro magnetic UNIT II - Circuit Analysis Ohms Law, KCL, KVL Mesh and Nodal Analysis, Circuit parameters, energy storage aspects, Superposition, Thevenins, Nortons, Reciprocity, Maximum Power Transfer Theorem, Millmans Theorem, Star-Delta Transformation. Application of theorem to the Analysis of dc circuits. UNIT III - A. C. Circuits R-L, R-C, R-L-C circuits (series and parallel), Time Constant, Phasor representation, Response of R-L, R-C and R-L-C circuit to soinusoidal input Resonance-series and parallel R-L-C Circuits, Q-factor, Bandwidth. UNIT IV - Measuring Instruments Principles, Construction and application of moving coil, moving iron, dynamometer type, induction type instruments, extension of range of ammeter, voltmeter (shunt and multiplier), Two-wattmeter method, for the measurement of power, Cathol-ray Oscilloscope and Applications. UNIT V - Transformers Construction and Working principles and phaser diagrams of Single-phase Transformer, Emf equation, Equivalent circuit, Regulation and efficiency, and Auto transformer. Rotating Machines Construction and working principles of dc motor and generator and its characteristics Applications of DC machines Construction and working principles of 3--Induction motor, Torque-speed characteristics, and Industrial applications. Text Books: 1. P.C. Sen Principles of Electric Machines and Power Electronics, Wiley Eastern 2003. 2. Vincent DEL TORO Electrical Engineering Fundamentals Prentice Hall India, Ed 2002. 3. D,P . Kothari, I . J Nagrarh Mc Graw Hills

11 | P a g e

BASIC ELECTRONICS
CREDIT : 3

CODE: 01BT 203


Course Objective: Electronics Industry is called the backbone of IT industry also Electronic Equipments are used every where irrespective of the type of industry. Hence the course is designed to make students fundamentally clear about the basics. UNIT I Electronics Engineering Materials and Semiconductor Devices (EEMSD): Solids, Classification of solids, Energy levels and Energy levels diagrams, Semiconductor and its types, intrinsic and extrinsic Semiconductors, conduction in metals and Semiconductors, difference among metals, semiconductors and insulators . various methods of formation of PN Junction, drift and diffusion current ,difference between drift and diffusion currents , PN Junction diodes, diode under forward bias , diode under reverse bias, V- I characteristics of PN Junction diode, Break down and types Break down diodes ,diode as a switch, diode as a Rectifier, Half wave Rectifier, Full wave Rectifier . UNIT II Transistor Amplifiers Power Electronic Circuits (TAPEC): Bipolar Junction Transistors , Construction and Principles of Operation of npn and pnp transistors , BJT configurations e.g.: common base, common emitter, common collector, Junction field effect Transistors , physical structure , Construction and Principles of Operation of Junction field effect Transistors, V- I characteristics of Principles of Operation of Junction field effect Transistors, JFET Configurations as common source, common gate , common drain. MOSFET Construction and Principles of Operation. UNIT III Power Electronic Circuits: SCR,Thyristor, Principles of Operation and characteristics of SCR, Triggering of SCR, DIAC,TRIAC Thyristor characteristics , Phase controlled Half wave and Full wave Rectification. Choppers, Power Supply , UPS and SMPS. UNIT IV Digital Electronics : Digital Circuits systems and Devices (DCSD): Number system, Binary Decimal, Octal , hexa decimal systems conversion among them , Boolean Algebra, Basic Logic Gates AND ,OR, NOT, NAND, XOR, XNOR ,different types flip flops, Memory units and devices , Registers ,counter, multiplexer, encoders and decoders. A-D and D A Converters SOP and POS , Minimization Techniques , K- Map . Microprocessors(8085) UNIT V Telecommunication Systems and Modern Techniques(TSMT): Basic Principles of Communication Modulation, Need of modulation Analog Modulation(A.M), modulation index, Frequency modulation , Phase modulation Advantages and Disadvantages of modulation Examination Scheme:
Component Weightage Assignment 25 Mid Term Examination 15 University Examination 60 Total 100

12 | P a g e

QUESTION BANK FOR


MATHEMATICS - II
(01BT201)

13 | P a g e

BASIC FORMULA OF MATHEMATICS CREDIT: 3 CODE: 01BT201 e=2.7183 =3.1416 =1.4142 Quantity Length Mass Time Force 1/e=0.3679 1/=0.3183 = 1.732 FPS System Foot(ft) pound Second Lb.wt. loge2=0.6931 loge10=2.3026 1 rad. = 5701745 CGS System Centimeter(cm) gram Second Dyne loge3=1.0986 log10e=0.4343 10=0.0174 rad. MKS System Meter(m) Kilogram Second newton

1. Quadratic equation ax2+bx+c=0 has rots =


( )

,=

+=- = Roots are Equal if b2-4ac = 0 Roots are real and distinct if b2-4ac >0 Roots are imaginary if b2-4ac <0 2. Progressions 1. Numbers a, a+d, a+2d..are said to be in arithmetic progression(AP) Its nth term Tn= a+(n-1)d
2

and sum Sn= (2a+(n-1)d)


( )

2. Numbers a, ar, ar ..are said to be in Geometric Progression(GP) Its nth term Tn=arn-1 and sum Sn = , S= (r<1) 3. Numbers 1/a, 1/(a+d), 1/(a+2d)are said to be in Harmonic Progression(HP)(i.e. a sequence is said to be in HP if its reciprocals are in AP its nth term T n= 1/(a+(n-1)d). 4. If a and b be two numbers then their Arithmetic mean = (a+b), Geometric mean= Harmonic mean = 2ab/(a+b). 5. Natural numbers are 1 2 3 .n n=
( )

=
-1

)(

y= r sin , or r =

1. Co-ordinates of the point: cartician (x,y) and polar (r,) then x= r cos , , = tan

r y
14 | P a g e

Distance between two points (x1,y1) and (x2,y2) = (

) )

Points of a division of a line joining (x1,y1) and (x2,y2) in the ratio m1: m2 is ( In a triangle having vertices (x1, y1), (x2, y2),(x3, y3) I. II. ( III. Area = | |

Centroid (point of intersection of medians) is ) Incenter ( point of intersection of the internal bisectors of the angles) is ( )

Where a, b, c are the lengths of the sides of the triangle. IV. Circumcentre is the point of intersection of the perpendiculars V. Orthocenter is the point of intersection of the perpendiculars drawn from the vertices to the opposite sides of the triangle.

15 | P a g e

UNIT -1 DIFFERENTIAL EQUATIONS Any relation between known functions and an unknown function is called a differential equation if it involves the differential coefficient of the unknown function. It is usual to denote the unknown function by y finding the unknown function is called saving or integrating the differential equation. The solution or integral of the differential equation is also called its primitive because the differential equation can be regarded as relation derived from it (i)(x2-y2) =xy
{ ( ) }

(1)

(II)p= (iii)x3 (iv)x (v)

(2) - 3y=x2+ x

+2x2 d2y/dx2+3x + y =K +y2 ( ) +z =xyz

(vi)y=2x

Which involves differential coefficients are called the differential equations. Differential equations which involve only one independent variable are called ordinary differential equations. Equations (i),(ii),(iii) and (iv) are of this type. Differential equations which involve two or more independent variables are called partial differential equations. Equations (IV) and (v) are of this type. The order of a differential equation-The order of a differential equation is the order of the highest derivative involving in the equation. The degree of a differential equation-The degree of a differential equation is the degree of the highest order derivative involving in the equation when the equation is free from radicals and fractional powers For example-The differential equations+ xy= a + x ( =2

) - 6x2 ( ) +ex= sin xy

16 | P a g e

( ) ]

PART A (SOLVED QUESTAION) 1.Find the differential equation of the family of circles of radius r whose centers lies on the x axis. Solution-The equation of the circle with radius r and centre on x axis is (x-a)2+y2=r2 Differentiating (1)with respect to x, we get 2(x-a)+2y =0

Eliminating a between (1) and(2) we get ( ) + y2 =r2 ]= r 2

y2[( )

Which is the required differential equation? 2. Solve (1+e x/y) dx+ e x/y [1-(x/y)] dy =0 Solution The given equation may be written as e x/y ( ) + ( 1+ e x/y) =0 ) =0 =v+ y =0

ev (1-v) + (1+ ev) ( Putting x= v y or

ev-vev+v+vev+(1+ev) y (v+ev)+ (1+ev) y


( )

=0

=0

Integrating, log(v+ev)+logy=log c ,when C is an arbitrary constant Log{(v+ev)y}=log C (v+ev) y=C [x/y+ e x/y ]
17 | P a g e

y =C, Putting v=

( x+y e x+y) =C 3. Solve + 2y tan x=sin x,given that y=0 when x=/3

Solution here P=2tan x and Q =sin x Integrating factor =e pdx =e 2tan xdx =e2log sec x =elog(sec x)2=sec2x Multiplying the given equation by sec2x we get Sec2x (dy/dx +2y tan x) = sin x sec2x Or d/dx(y sec2 x)= sec x tan x Integrating both sides with respect to x,we get y sec2 x= C + sec x tan xdx, where C is an arbitrary constant y sec2 x =C + sec x It is given that when x=/3,y=0 From (1) 0 x sec2 = C + sec /3 0=C+ 2 Sec /3 =2 From(1)the required solution is y sec2 x = - 2 + sec x y= -2 cos2 x + cos x 4. Solve (1+y2)dx+dx+(x-e tan-1 y)dy=0 Solution:The given equation can be written as dx/dy+ x/1+y2= etan-1y/1+y2 Therefore the integrating factor=e 1/1+y2 dy =etan-1y Multiplying both sides of (1) by the integrating factor and integrating ,we have x.e tan-1 y=C + c tan-1y/1+y2 x e tan-1y dy

Where C is an arbitrary constant x e tan-1 y =C+ e2t dt, where t=tan-1 y


18 | P a g e

=C+1/2 e2

5. Solve Solution here P=cos x and Q= Integrating factor = Multiplying the given equation by the integrating factor get y. y. =C+t. =C+ y. 6. Solve Solution Dividing both sides of the given equation by ) Putting tan y=v or The above equation reduces to + 2xv = ) we get ( ) ( ) y2 and integrating with respect to x,we

sin x cos x,where C is an arbitrary constant where t=sin x

Which is a linear equation, whose integrating factor = Multiplying both sides of (1) by the integrating factor, we have v. v. =C+ =C+ ( )

19 | P a g e

Or ( 2tan y=2C 7. Solve Solution. (

) ) ) ( )

(v=tan y)

Dividing both sides of the given equation by z( ( Putting Or = = = = v or ( )

) ,we get ( )

,the above equation reduces to

Proceeding is the ususal way ,the solution of above equation is v

( 8. Solve Solution. ( )

Dividing both sides of the given equation by y,we get ( ( Putting v=log y = the above equation reduces to ) )

20 | P a g e

Which is a linear equation in v? Its integrating factor= = ,

Multiplying both sides (1) by the integrating factor x and integrating ,we have v.x=C+ where C is an arbitrary constant v.x=C+x (logy) x=C+x 9. Sovle (1+xy)ydx+(1-xy)xdy=0 Solution The given equation can be written as (ydx+xdy)+(x D(yx)+x Dividing both sides of this equation by
( )

+ where z=xy

Integrating ,+ log x- log y =C ,putting z=xy

Log(x/y)=C+ 10. Solve (xysinxy+cos xy)ydx+(xy sin xy cos xy)xdy=0 Solution integrating factor = -( = Multiplying both sides of the given equation by this integrating factor,we get ( )ydx+ ( )
) ( )

(tanxy)(ydx+xdy)+ (tanxy)d(xy)+ dy =0

21 | P a g e

Tan z dz+ Integrating term by term we get Log(sec z)+log x-log y=log C,where C is an arbitrary constant log{ }= log C

(sec z)=C Xsec(xy)=Cy 11. Solve (y2+2x2y)dx+(2x3-xy)dy=0 Or y(ydx-xdy)+2x2(ydx+xdy)=0 Solution The given equation can be written as Y(y+2x2)dx+x(2x2-y)dy=0 Which is of the form as given in method vi Let xhyk be an integrating factor Multiplying the given equation by xhyk,we get(xh ) Here M= =(k+2)xh And =2(h+3) ( ( ) ) (1) (2) ) (

If the equation (A) be exact we must have Or (k+2) From (1) and (2) Equating coefficients of On both sides we get k+2=-(h+1) and 2(k+1)=2(h+3) Hence solving we get H=- ,k=The integrating factor=xh yk= Multiplying the given by
22 | P a g e

,we get

(x-5/2 y 3/2+ 2x-1/2)dx+(2x1/2 y-1/2 x-3/2 y )dy =0 In this form we have Regarding y as constant 12. Solve (D2-2D+5)y= Solution here auxiliary equation is m2-2m+5=0,whose roots are M=-1 C.F.=e-x[ AND P.I.= Here a=-1 The required solution is y=C.F.+PI. i.e. y= ( ) ],where C1 nad C2 are arbitrary constants ( ) , the equation is exact

13. SOVLE (D2+D+1)y=sin 2x Solution Here the auxiliary equation is m2+m+1=0 Which gives m=C.F.= { ( ( ) ) ( )}

Where C1 and C2 are arbitrary constants And P.I.= =( = =( = =


)( ) )

, replacing

( ) )

) (D+3)sin 2x [( ) ]

( (

23 | P a g e

] since D means differentiation with respect to x

The complete solution is y= [ ( ) ( )] ( )

14. Solve (D3-D2-6D)y=x2+1 where D= Solution-Here the auxiliary equation is m3-m2-6m=0 m(m2-m-6)=0 m(m-3)(m+2)=0 m=0,3,-2 C1 F=C1 C.F.=C1+C2 Where C1,C2 and C3 are arbitrary constants P.I.= =
(

(
)

) ( ) ( ) ( ) ] ) ] ( ) D= ) ( )] ]( ) D(1)=0

= = = ==- [

[ [ [ (

The required solution is y=C.F.+P.I. Y=C1+C2 15. Solve ( Solution The auxiliary equation is m2+a2=0 or m = ai C.F.=C1 cos ax+ C2 sin ax where C1 and C2 are arbitrary constants And P.I.=
24 | P a g e

+ )

(1)

Now ( [ Similarly = = = [ ( ( ) ( ) ] ) ( ) ( ) ] )

From (1) we have P.I.= =[ ( =( ) The required solution is y=C.F. +P.I. y=C1 cos ax+C2 sin ax +( ) ( ) [ { ) ( ( } )( ) { )] }]

PART B (UNSOLVED QUESTION) 1. xy2. Solve Ans. y= ( - 2y = ) (3sin x+ cos x) Ans. = ( )

25 | P a g e

3. Solve Ans.y=C1+ 4. Solve (D3+D2-D-1)y=cos 2x Ans.y=(C1 ( ) ( [ ( ) ( ) ( )] )

5. Solve (D2-4D-5)y= Ans. Y=C1 6. Solve (D2+a2)y=sec ax Ans.C1 cos ax+ C2 sin ax +( )

7. (D2+16)y=sin 2x,given that y=0 and Ans.12y=2sin 4x+sin 2x 8. (D2+4D-12)y=(x-1) Ans. y=C1 9. Solve Ans. y= ( ) ( ( ( ) ) )

10. Apply the method of variation of parameters to solve

Ans. Y=C1cos x+ C2 sin x- cos x log(sec x +tan x)

26 | P a g e

UNIT-2
SERIES SOLUTION AND SPECIAL FUNCTION Introduction Consider the second order homogeneous linear differential equation a0 (x) d2y/dx2 + a1 (x) dy/dx + a2 (x) y = 0, a0 (x) 0

where, a0 (x), a1 (x) and a2 (x) are polynomials. The equation (i) is called a variable coefficient second order homogeneous equation. A number of problems of physics and engineering involve differential equations of the form (i) in which a0 (x), a1 (x) and a2 (x) are polynomials..Solution of such equations can be obtained in terms of infinite series. In this chapter, we shall present methods for determining the solution of (i) in terms of infinite series. The methods can be classified into two categories, power series method and generalized power series method (Frobenius method). In the following sections, we shall discuss the applications of these methods in solving special functions. The study of those solutions (and of other higher functions not discussed in calculus) is called the theory of special functions. The series solution of certain differential equations give rise to special functions such as Bessels function, Legendres Polynomial ,strum-Lioville problem based on the orthogonality of function is also included which shows that Bessels Legendres and other equations can be considered from a common point of view. These special functions, have many applications in mathematical physics and engineering. 1. Solve the following differential equation in series (1-x2) d2y/dx2 x dy/dx +4y =0 Solution. d2y/dx2 [x/ (1-x2) dy/dx] +4y/(1-x2) =0 Here, p(x) = - x/ (1-x2), q(x) = 4/ (1-x2) p(x) and q(x) both exist at x= 0,so x=0 is an ordinary point of the equation Let,

Putting the values of dy/dx and d2y/dx2 (1-x2) ( ) ( ) + 4 ( )

Equating to zero the coefficients of various power of x, Coefficient of x0 a 2. 2.1 + 4a0 =0, a2 = -2a0

Now equating the coefficients of xn, we get an+2(n+2)(n+1)-(n-2)(n-1)an=0


27 | P a g e

an+2= (n-2) an/n+1 putting n=1,2,3,..we get a3=-a1/2, a4=0, a5=-a1/8, a6=0

Substituting these values in equation (ii),we get y =a0+a1x 2a0x2 - a1x3/2- a4x5/8+. y= a0(1-2x2)+ a1x(1- x2/2-x4/8+..) 2. Solve the following equation in power series about x=0 2x2 d2 y/d2 x + x dy/dx (x+1)y=0 Solution. 2x2 d2 y/d2 x + x dy/dx (x+1)y=0 d2 y/d2 x + (1/2x) dy/dx (x+1)y/2x2=0 Hence, xP(x) and x2 Q(x) both exist at the point x=0.So, x=0 is a regular point, we assume the solution in the form y= dy/dx = d2 y/d x2 = ( ( ) )( )

substituting the values of y, dy/dx and d2 y/dx2 in given equation we get ( )( [ ( )( ) ) ( ) ( ] ) -(x+1) =1

Equating to zero the coefficients of lowest power of x,we get ao [2m(m-1)+m-1]=0 m=1,-1/2 Coefficient of next lowest power of x, a1[2(m+1) m+(m+1)-1]-a0=0 a1= a0/m (2m+3) Now, equating to zero the coefficient of ar [2(m+r)(m+r-1)+(m+r-1)]-ar-1=0 ar = ar-1/(m+r-1)(2m+2r+1) a1 = a0/(2m+3),
28 | P a g e

by putting r = r-1,

a2 = a1/(m+1)(2m+5), a3 = a2/(m+2)(2m+7), a4 = a3/(m+3)(2m+9), For m=1, a1 = a0/5 a2 = a1/14 a3 = a2/27 Here, y1 = a0x [1+x/5+x2/70+x3/1890+..] For, m =-1/2 a1=- a0 a2=- a0/2, a3=- a0/18,.

y2= a0x-1/2[1-x-x2/2-x3/18..] Hence the solution is y1=Aa0 x[1+x/5+x2/70+x3/1890+.]+ Ba0 x-1/2 [1-x-x2/2-x3/18-...] 3. Obtain the general solution of the equation x2 d2 y/d2 x + 5x dy/dx +x2 y=0 Solution. Let us assume the series solution of the given equation as y= Substituting this in the given equation we get [( )( ) ( )]

Equating to zero the coefficient of the lowest degree term xm, we get the identical equation as m(m+4)a0=0,which gives m=0,-4 as a0 Equating to zero the coefficient of xm+1, we get (m+1)(m+5)a1 =0, a1=0 as m .Equating to zero the coefficient of xm+r, we get, (m+r)(m+r+4)ar +ar-2 = 0 ar = ar-2 /(m+r)(m+r+4) putting r = 2,3,4.. a2 = - a0/(m+2)(m+6), a4=- a2/(m+4)(m+8)
29 | P a g e

and m

Now, equation (i) becomes, y = a0 xm [1- x2/(m+2)(m+6) +x4/(m+2)(m+4)(m+6)(m+8) -..] putting m=0, y1 = a0[1-x2/2.6+x4/2.4.6.8-] putting m=-4,the denominator of each term from third onwards vanishes. A0 =b0(m+4) and taking m y = b0/-2.2.4[1-x2/2.6+] which is nothing but a multiple of y1 hence not an independent solution. y2 = / m [b0 (m+4)ym]m=-4 = b0 xm [(m+4)- (m+4)x2/(m+2)(m+6) + 1x4/(m+2)(m+6)(m+8)-.] y/ m = y log x + b0 xm [1 +(m2+8m+20)/ (m 2 +8m+12)2-] Hence, y = A(y1) + B(y2) = A a0 [1-x2/12 x4/384-...] + B b0 x-4 log x[-x4/16-x6/16-....] + B b0 x-4 + [ 1- x2/4-x4/4+] 4. Find the series solution of the equation x d2 y/d x2 + dy/dx +x2 y=0 Solution: x d2 y/d x2 + dy/dx +x2 y=0 Here XP(X) and X2Q(X) exist at the point X=0. So X=0 is a regular singular point. Let ( ) -4, we get

)(

Substituting these values in equation (i) we get ( ( ) )( + ) ( ) + =0

Equating to the zero the coefficient of lowest power of X i.e Xm-1, We get the identical equation as a0m2=0 m= 0,0 as a0 0

Equating the coefficient of Xm, we get a1(m+1)2=0


30 | P a g e

a1=0 as m1 -1

Equating the coefficient of Xm+1, we get a2 (m+2)2=0 a2=0 as m1 -2

Equating the coefficient of Xm+2, we get a3 (m+3)2+ a0 =0 a3=(


)

Equating the coefficient of Xm+r+2,We get ar+3(m+r+3)2+ar =0 ( Putting r =1,2,3., We get


( )

=(
) (

=0,
)

=0

From equation (ii) we have y1=(y) m=0= a0 + a1x + a2x2 + y1=(y) m=0= a0 y1= a0[1 Again Y = a0xm (
( ) ) ( ) ( ) ( )

(iii)

Or Y = a0xm [1 ( = a0xm + a0xm [0 + ( Y2 = ( [1 (


)

) (

] ] ] ] + a0 [ (

) (

) (

) m=0 = a0 ]

[1 (

( ) ( )

( ) ( ) ( )

( ) ( )

Hence the complete solution is Y = Ay1 +B ( ) m=0 ) [1 ] + Ba0 [ ]

Or y = (Aa0 +Ba0
31 | P a g e

Or y = (c1 +c2

) [1

] +

Where c = Aa0, B = Ba0 5. XJn =-nJn+ xJn-1. Proof. We have =


( ) ( ( ) )

( )

( (

) )

( )

Therefore ( ( ) ( ( ) ) ( )

( ) ( (

) ( ( ) )

( ) ( ( ) )

) ( ( ( (

( ) ) ( )( ) ( )

( )

) )

( )
)

= XJn =-nJn+ xJn-1 6. 2Jn =Jn-1- Jn+1

( ) ( ) ( )

Proof. By Recurrence formula (i), we have XJn =nJn- xJn+1 (1)

And by Recurrence formula (ii),We have XJn =-nJn+ xJn-1 (2)

Adding first and second we get 2XJn =x (Jn-1+ Jn+1) Or 2Jn = (Jn-1- Jn+1)

32 | P a g e

7. 2nJn =x (Jn-1+ Jn+1) Proof. By formula (i) we have XJn =nJn- xJn+1 And by formula (ii) we have XJn =-nJn+ xJn-1 Sub (ii) - (i), we get 0=2nJn -x (Jn-1+ Jn+1) Or 2nJn =x (Jn-1+ Jn+1) 8. ( )

Proof. By formula (i) we have XJn =nJn- xJn+1 Multiplying both sides of the above by Jn =n or or ( Jn =n ) JnJn= Jn+1 Jn+1 , we get

9.

Proof. By formula (ii) we have XJn =-nJn+ xJn-1 Multiplying both sides of the above by Jn =-n Or Or ( Jn + n ) (x) = [ [ + ] ] Jn+ Jn-1 Jn Jn-1 , we get

10. Prove that ( ) Or ( )

( )

)(

( )

)(

Putting n =- , We get
33 | P a g e

(x) =

[
( )

Or

(x) = (x) = (x) =

11.

(x) = (x) = By recurrence formula 2nJn =x (Jn-1+ Jn+1) Or =x (Jn-1+ Jn+1) = Putting n = ( ) (3)

in eq. 3 we get (x) ( ) ) ) Jn+4

(x) + (x) = ( ) (x) =-( ) (x) = (

Or

Or

12. Jn+3+ Jn+5 = ( 2nJn =x (Jn-1+ Jn+1)

Now putting (n+4) for n in the given formula, we get ( ) Jn+4= Jn+3+ Jn+5 ) Jn+4

Or Jn+3+ Jn+5 = (

13. Prove that 4 = Jn+2+ Jn-2-2Jn 2Jn = (Jn-1- Jn+1)


34 | P a g e

4Jn = (2Jn-1- 2Jn+1) Applying (1) for 2Jn-1 and 2Jn+1, we have 4Jn = (Jn-2- Jn)- (-Jn-2+ Jn) 4 = Jn+2+ Jn-2-2Jn 14. Prove that ( ( ) ( )= 1 )

Replace n by 0,1,2,3successively in (1)we get ( ( ( ) ) ) ( ( ( ) ) )

Adding these by column wise and nothing that Jn0 and n [ ( )

Integrating above we get ( ) ( ) (2)

Replacing x by 0 in (2) and nothing that ( ) 1+2(0+0+0+.) =c C=1 Hence (2) becomes ( 15. Prove that following relation ( )
35 | P a g e

( )

)= 1

) ( )

( )

Solution The solution of x2 Is ( ) ( )

+x

(1)

(2)

By recurrence relation (i) we have XJn =nJn- xJn+1 Putting the value of XJn from (3) in (2), we have ( ) ) ( ) ( ( ) ) ( ) (

PART B (UNSOLVED QUESTION) 1. Prove that 2. Prove that 3. Prove that ( ( ( )) ) ( ) ( ( ) ( ))

4. Express ( ) in terms of ( ) and ( ) 5. Show that Bessels function ( ) is an even function When n is even function and is odd function when n is odd. Express ( ) in terms of ( ) and ( ). 6. Prove that 7. Express ( )= 8. Show that wrt x 9. Show that 10. Show that ( ) {
(

( )

( )

. in term of legendres.
)

} where dashes denote differentiation

(0)=0 for n is odd. ( ) =0 n

36 | P a g e

UNIT- 3 LAPLACE PRINCIPLE DEFINITION: Let F(t) be a function of t defined for all positive values of t. then the Laplace transformation of F(t) is defined by L{F(t)}= ( ) ( ) provided that the integral exists, s is a parameter

We also write L{F(t)} = f(s)= which may be real or complex number.

L is known as laplace transform operator. The given function F(t) known as determining function depends on t, while the new function to determined f(s), called as generating function depends only on s. f(s) is known as the laplace transform of F(t). TRANSFORMS OF ELEMENTORY FUNCTIONS The direct application of the definition gives the following formulae: 1.) L{1} = , s>0 2.) L{ }= 3.) L{
( )

if s>0 and n>-1

}=

if s>a if s>0 if s>0 if s>a if s>a Part A

4.) L{sin at} = 5.) L{cos at} = 6.) L{sin hat}= 7.) L{sin hat}=

(solved question) 1. Find L{ } Solution: We have L{ }= L{ = }=( = f(s) say Then by first shifting property
) ( )

2. If L{cos2t} = L{cos2t} =

, find L{cos2at}

Solution:- we have
( )

= f(s)

By change of scale property we have


37 | P a g e

L{cos2t} = = = [ =
( ( ) ( ( ) ) )

( )

3. Prove that

Solution L {sin t} = = f(s) say


( )

L{t3sint} = (-1)3
==2 = [ ]

) ) ) ) ) ) ( ( ) )

[(
( ( (

=>

Putting s=1, we have


( ( ) )=0

4. Express the following function in terms of unit step function F(t)= and find its Laplace transform. Solution: F(t) = F(t) = (t-1)[u(t-1)-u(t-2)]+(3-t)[u(t-2)-u(t-3)] = (t-1)u(t-1)- (t-1)u(t-2)+(3-t)u(t-2)- (3-t)u(t-3) = (t-1)u(t-1)- 2(t-2)u(t-2)+(t-3)u(t-3) L{F(t)}= 5. Find L-1 Solution: L-1 = L-1 { = } { } { } { }

38 | P a g e

= 6.) Evaluate {(
) ( ) ( )

SOLUTION { = = 7. { } { =3 { } { } { } } { } { } { } ( ) ( { ) } ( ) }

=3 = = =

{ {

} }

} } { }

8. Find the inverse laplace transform of Solution{ = = = {(


( ) )

( ( )

} }

{( {

} }

39 | P a g e

9.Find

( )} )

Letf(s)=log( log( =log( log( =f(s)= f(s)=2( ) ) ) )

( ) =2(cost-1) =t = { ( ) ( {
(

{ }]

( )}
)

) ( } )

10.FIND Solution L L Or Or Or Or Or { { } }

{ ( { {(
(

)}
)

Since we know { Then we have


40 | P a g e

( )

( )

{ =[ =[ =1( ( ) { =[ =t+ =t 11. Find the laplace transform of Solution. We have L By theorem L{ L{ =[ = = Now L{ = [ *= [ } ( ( )] ) ( )] = ( )
( )

] )]

)]

( (

) )

] ( )

} } ]

( )

Which does not exist,lim log(x2+a2) is infinite Hence L{ }

41 | P a g e

12. Using Laplace transform,evaluate Solution.we have { = ( ) ( = [ = [ = [ = [

] ] ]

= *

) (

Putting s=1,we get = 13. L =- ( =(


( ) )

( )

14. Draw the graph of the periodic function F(t)= <t<2


42 | P a g e

Also find its Laplace transform Solution.We know that L ( ) ( )

= = = = = = [ [ [( { [ (

( ) ( ( ) ) {( )

T=2 ] ( ) } ) ( )] )( ] ( ) }]

DIAGRAM f(t) ( ) 2 (2 )

15.Find Solution

{ = = = { } { } { } { } { }

43 | P a g e

PART B
(UNSOLVED QUESTION) 1. Find L-1{(
) ( ) ( )

2. Evaluate L-1 3. Find the inverse Laplace transform of 4. Find L-1 5. find L-1
(

(
)

) } ( )

6. obtain the inverse Laplace transform of 7. use convolution theorem to find L-1
( )
-1

8. find L { 9. using Laplace transform, find the solution of the initial value problem Where (0) =2, Y (0) = 0 10. solving by using the Laplace transform method Y (t) + 4Y (t) +4Y (t) = 6e-t Where Y(0) = -2, Y(0) = 8

44 | P a g e

UNIT -4 FOURIER SERIES


INTRODUCTION Most of the events in nature and many other systems, being periodic in nature. In many engineering problems, especially in the study of periodic phenomena in conduction of heat, electrodynamics and acoustic, it is necessary to express a function in a series of sines and cosines. Most of the single-valued functions which occur in applied mathematics can be expressed in the form
2

cosx +

cos2x +

cos3x + .+

sin x +

sin2x +

sin3x + ..

Where , , , , , , , , are real constants and the series is known as Fourier series. The constants , , , , , , , , are called Fouriers Coefficients of the periodic function. Eulers Formulae The Fourier series for the function f(x) in the interval ( ) + 2 is given by (i)

Determination of the Fouriers Constants: To find = =


2

: assume that (i) can be integrated from x = c to x = c+2 , term by term, so that we have ( ) ( )

( ) ( ) (

To find

( )

: multiplying each side of (i) by cosnx and integrate with respect to x between the limits

x = C to x = C + 2 , so that we have

45 | P a g e

( )

( )

To find : multiply both side of (i) by sinnx and integrate with respect to x between the limits x = c to x = C+2 , so that we have ( )

( )

. ( )

( )

( )

are called Eulers formulae Eulers Formulae for Different Intervals Case (i) : If C= 0, then the interval for the above series (i) become 0 formulae reduce to
46 | P a g e

and the Eulers

( )

( )

( )

Case (ii): If c = , then the above interval for the Fourier series become Eulers formulae, reduce to ( )

, then the

( ) ( ) Note: Periodic Functions Consider any function of x as f(x) = f (x +2T) = ., then the function f(x) is said to be periodic with its period T. This T is non-zero, smallest and positive real variable. For example consider f(x) = sin x, f(x) = sin x = sin (x+ 2 ) = sin (x + 4 = Hence, f(x) = sinx, is a periodic function with the period 2 . This is also called sinusoidal periodic function f(x) = sin x
1

x -2

O 2

-1 2

Even and odd Functions (a) Even Function: A function f(x) is said to be even if f(-x) = f(x) for all x. Properties of even functions: 1. The graph of f(x) is symmetrical about y axis.
47 | P a g e

2.

f(x) contains only even powers of x and may contain only cos x, secx and their higher powers.

( )

( )

( )

4. the sum of two even funtions is even i.e. h(x) = f(x) + g(x) is even when both f(x) and g(x) are even 5. Product of two even functions is even i.e. h(x) = f(x) g(x) is even when both f(x) and g(x) are even. 6. The product of two odd function is an even function For example: x2, 2x4, cos x, cos2x, 3x2 + 5, x4 + cos 2x +2 are all even functions. y

(Even function) cos nx is even (b) Odd Function: A function f(x) is said to be odd if f(-x) = -f(x) for all x. Properties of odd Functions: 1. The graph of f(x) is symmetric about the origin lies in opposite quadrant 1st and IIIrd. 2. f(x) contains only odd powers of x and may contain only sin x, cosec x and their higher powers. ( ) 4. 5. 6. ( )

The sum of two odd functions is odd i.e. h(x) = f(x) + g(x) is odd when both f(x) and g(x) are odd Product of an odd function and even function is odd i.e. h(x) = f(x). g(x) is odd, when f(x) is even and g(x) is odd or vice versa. Product of two odd function is even y

odd function (sin nx is odd)


48 | P a g e

PART A (SOLVED QUESTION) 1. Given that f(x) = x + x2 for find the Fourier expression of f(x).

Solution ( )

( )

[ [

] ]

( ) ) ) ( ) ( ) ( )( ) )]

( [(

[(

) (

( )

( )

[(

)(

)(

[ (

49 | P a g e

Substituting the values of ao, an, bn in (1) we get [ [ Putting x = (2) becomes [ putting x = (2) becomes [ Adding (3) and (4), we have [ ] ] ( ) ] ( ) ] ]

(2)

2. Expand f(x) = x sin x as a Fourier series for ( ) ( )

[ (

)]

50 | P a g e

( (

) )

( (

) ] ) ( ( ) ) ( ( ) )

[ {

} )

} ]

} ]

)]

[ (

( )

( (

) ] ) ( ( ) ] ) ( ( ) )

[ { ( ( )

} ]

[ = 0, 3. (n

) ) (

( )

) ( ) ( )

Find the Fourier series expansion for ( ) ( ) ( )

( )

51 | P a g e

( )

( ( )

( )

Substituting the values of ao, an and bn in (1) we get ( ) ( ) ( )

4. Obtain the Fourier series expansion for the function ( )


Hence, deduce that ( )

( ) ( ) Solution: We have f(x) = x2 f(x) is an even function, therefore, f(x) contains only cosine terms. Hence b n = 0 ( ) ( )

52 | P a g e

( )

) ]

( )

[ [ (

( ]

)]

Substituting in (1), we get [ [ ( ( ) ) ( ] ) ( ) ]

[ [ At x = and x = 0, the function f(x) is continuous is (2), we get [ [ [ ] ] ]

] ] ( )

Putting x

53 | P a g e

] ( )

putting x = 0, in (2), we get [ [ [ ] ( ) Adding (3) and (4) we get ] ]

5. Find Fourier series for the function defined by ( ) {

Hence proved that

Solution: By Fourier series, we have ( )

( )

[ (

54 | P a g e

[(

) [

( ) ] ]

( )

{ [ [ ] ] [

] }

( )

{ [ [ ] [ [ ] [

] } ] ( ) ]

[ If n is even

) ] n = 1, 2, 3, ..

Hence from (1) we have ( )

( ) The expansion (2) is required Fourier expansion


55 | P a g e

( )

( ) [ [ [ ( ) ( ) ] ( ) ] ]

6. Obtain Fourier series of the function ( ) {

and hence show that ( Solution. Here f(x) is an even function so )

( )

[ ]

( )

[ [ [ ( ) ] ) ]

56 | P a g e

Thus , the Fourier series expansion is ( ) [ ]

7. Find the Fourier series of the function ( ) and ( ) { ( )

Solution. Let the Fourier series expression be ( ) ( ) [( [( [ [ { Thus, the Fourier series is ( ) ( [( ) ( )( ) )] ( [( ) ) ( )( )] )( )( ] ( ) [ ] ) ) ( ( ( )( ) [ ] ( ) ] )( )] )]

57 | P a g e

[( {

( )

( )

( )

[(

)( [(

( ) (

)( )

)] ( )( )]

[ [ {

] ( )

[ ]

) [

] ( ) ]

Thus, the Fourier series is ( ) ( 8. Expand for f(x) = k for ) ( )

(i) Since series (ii) cosine series () ( ) ( )

(U.P.T.U. 2007)

58 | P a g e

Half range sine series is ( )

[ [ ( )

) ] ] ( )

( )

( )

Therefore, from ( ) ( ) ( ) 9. Find the Fourier half range cosine series of the function () { ( ) ( )

()

()

( )

()

()

()

59 | P a g e

[ ] [( ( () [ )] ) ]

()

[ [ ( ) (

( )] ] [

) [( )( ) ( )( ] )]

( an = { ( )

) )

cos

= (

When is even

Hence from (i) the half range Fourier cosine series is () () () 10. Obtain a Fourier cosine series expansion of the periodic function defined by () ( ) ( ) ( )

Solution: we have () ( )

60 | P a g e

() [ ] ( ( ) ) ( )

)]

) ( )

) ( ( [ ) ) ( ] ) )

[ [ [(

( ( )

) ) ] ( ]

] ]

( (

) )

( ) [(

( ( (

) ) ) {

[ (

( ) [( ( )

) (

) )

] )

Here, we see that the above relation is not applicable to find

( ( Thus, we have
61 | P a g e

) )

() [ ]

11. Obtain the half range series for the function f(x) = x2 in the interval ( Solution. We know that half range sine series is given by ( ) ( ) Here, we have half range ( ) ( ) )

( [{ [ [ (

) ) ( ] )

)( ( (

) ) } ) ] ]

)(

)(

Therefore, half range sine series is f(x) = b1 sin x + b2 sin 2x + b3 sin 3x +. [ ( ( 12. Expand for f(x) = k for ) )] ( )

(i) Sine series (ii) cosine series () ( ) ( )

62 | P a g e

( Half range sine series is ( )

[ ( )

[ ]

) ]

( )

( )

( )

Therefore, from ( ) ( ) ( ) ( )

13. Find the Fourier series for the periodic function ( ) ( ( ) { ) ( )

[ ]

63 | P a g e

[ [ ( ) ]

( )(

)]

0, when n is even

[ [

( ( ) ]

( )( ( )

)]

Substituting the values of ao, an and bn in (1) we get ( ) [ ] [ ]

14. Find the Fourier series for the function () {

( )

{ (

{( =0

[ ]

{ (

( )

( )

64 | P a g e

{[

}}

{ (

( )

( )

{[

Substituting the values of () (

( ) )

15. Obtain the Fourier series of the function ( ) {

Solution: The function f(x) is odd. Therefore, the Fourier series of f(x) contains only sine terms ( )

( [ [ { ]

[(

( ) ( ) [ ]

The graph of f(x) is given in figure as given below

65 | P a g e

PART-B
(UNSOLVED QUESTION)

1. Find a Fourier series to represent, ( ) Ans. [

2. Let f(x) = x for Ans. f(x) = x = 2sinx sin2x +

Write the Fourier series of f on [-, ]

3. Find the Fourier series expansion for the function f(x) = x cosx, -<x< (U.P.T.U. 2002) Ans. f(x) =

4. Expand the function f(x) = x sinx as a Fourier series in the interval -x. Hence deduce that ( Ans. [ ] )

5. Find the Fourier series representing f(x) = x, o<x<2 and sketch its graph from X = - 4 to x = 4 Ans. [ ]

f(t)

-4

-2

6. Find the Fourier series for f(x), if ( ) {

( )

66 | P a g e

7. Find the Fourier series of the function ( ) where ( [ ( ) { ( ) { ) ( ) ] (U.P.T.U. 2005)

Using half range cosine series, show that ( 9. An alternating current after passing through a rectifier has the form i = l sin, =0 for o<< for <<2 )

f() 1 o Find the Fourier series of the function ( ( ) ( ) { ) 2 3 1

( )

67 | P a g e

UNIT-5
APPLICATION OF DIFFRENTIAL EQUATION INTRODUCTION The problems related to fluid mechanics, solid state physics, heat transfer, electromagnetic theory, Wave equation and other areas of physics and engineering are governed by partial differential equations subject to certain given conditions, called boundary conditions. The process to find all solutions of a partial differential equation under given conditions is known as a boundary value problem. The method of solution of such equations differ from that used in the case of ordinary differential equations. Method of separation of variables is a powerful tool to solve such boundary value problem when partial differential equation is linear with homogenous boundary conditions. Most of the problems involving linear partial differential equations Can be solved by the method of separation of variables discussed below. METHOD OF SEPARATION OF VARIABLES It involves a solution which breaks up into a product of functions each of which contains only one of the variables. The following example explains this method.

Solution Assume the trial solution z = X(x) Y(y) (i) where X is a function of x alone and Y that of y alone, substituting this value of z in the given equation we have X". Y - 2X'.Y + XY = 0 where X' = separating the variables, we get

since x and y are independent variables, therefore, (ii) can only be true if each side is equal to the same constant, K(say), so we have =k

Therefore, and

i.e X"-2X' - kX =0 i.e. Y' + kY =0

(iii) (iv)

To solve the ordinary linear equation (iii) the auxiliary equation is m2 - 2m - k =0 m = 1( )

68 | P a g e

The solution of (iii) is X = Y = C2 e-ky

)}

and the solution of (iv) is

Substituting these values of X and Y in (i), we get z={ i.e. z = {


( ) { ( )}

} C3e-ky }e-ky

)}

where a = c1 C3 and b = c2 C3 which is the required complete solution.

2 Using the method of separation of variables, solve , where u(x, 0) = 6 e-3x Solution Assume the solution u(x, t) = X(x) T(t) Substituting in the given equation, we have X'T = 2XT' + XT or (X' - X) T = 2 XT' ( X' - X - 2kX =0 Or = l + 2k k solving (ii), log X = (l+2k)x + log c or X = ce(1+2k)x From (iii), log T = kt + log c' or T = c'ekt Thus u(x,t) = XT = cc'e (1+2k)x ekt Now 6e-3x = u(x, 0) = cc e(1+2k)x cc1 = 6 and 1+2k = -3 or k = -2 Substituting these values in (iv) we get u = 6e-3x e-2t i.e. u = 6e-(3x + 2t) which is the required solution
69 | P a g e

(i)

(ii) (iii)

(iv)

Q. 3. Find the solution of the wave equation ytt = a2 yxx, when initial displacement is y (x, 0) = f(x) = b sin ( ) where symbols have usual meaning. Solution. Consider an elastic string tightly stretched between two points O and A. Let O be the origin and OA as x-axis on giving a small transverse displacement i.e. the displacement perpendicular to its length.

let y be the displacement at the point P (x, y) at any time, the wave equation ytt = a2yxx or As the end points of the string are fixed, for all time, y(0,t)=0 y(L,t)=0 Since, the initial transverse velocity of any point of the string is zero, therefore ( ) Also y (x, 0) = b sin The general solution of (i) is y = (Ci cos px + C2 sin px) (C3 cos apt + C4 sin apt) Applying the boundary condition y =0 at x =0 0 = C1 (C3 cos apt + C4 sin apt) C1 = 0 Therefore, y = C2 sin px (C3 cos apt + C4 sin apt) Again applying = 0, at t = 0 on

(ii) (iii) (iV) (v) (vi)

(vii) (vii)

= C2 sin px ap. (- C3 sin apt + C4 cos apt) 0 = C2 sin px. ap. C4 => C4 =0 Then (vii) becomes y = C2 C3 sin px cos atp Applying y = 0 at x = L 0 = C2 C3 sin pL cos atp sin pL = 0 = sin n , n = 0,1,2,3.................. pL = n , or p= putting p= in (viii), we have

(viii)

70 | P a g e

y = C2 C3 sin At t = 0, v = b sin b sin = C2 C3 sin

(ix)

C2 C3 = b, n =1 putting C2 C3 = b, n =1 in (ix) we get y = bsin( )cos( )

which is our required solution. 4.A string is stretched and fastened to two points I apart. Motion is started by displacing the string into the form y =k (Ix -x2) from which it is released at time t =0. Find the displacement of any point on the string at a distance of x from one end at time t. Solution The vibration of the string is given by the equation

As the end points of the string are fixed for all time, y(0, t) =0 y(1,t)=0 Since the initial transverse velocity of any point of the string is zero, therefore ( ) and y(x, 0) = k (1 x - x2) solution of (i) is y = (ci cos px + C2 sinpx) (C3 cos cpt + C4 sin cpt) At x = 0, y = 0 gives c1=0 y = C2 sin px (C3 cos cpt + C4 sin cpt) At t = 0, =0

(ii) (iii)

(iv) (v)

(vi)

(vii)

0 = c2 sin px. cp. C4 c4 = 0 y = C2 C3 sin px cos cpt At x = l, y =0 0 = C2 C3 sin pl cos cpt .. sin pl = 0 = sin n ; n = 0,1,2,3............. p=
71 | P a g e

y = C2C3 sin( Let C2 C3 = bn

)cos(

As bn is arbitrary constants and a differential equation satisfy solution for all constants. Then we can write sin( )cos( )

At t = 0, y = k (Ix - x2) k(lx-x2) = sin

Applying half range Fourier sine series ( ) ) ) ]

( [( { 0, when n is even Or
(

sin
)

cos
(

when n is odd
) ( )

Which is required solution. Q.5 A string of length I is fastened of both ends A and C. At a distance 'a' from the end A, the string is transversely displaced to a distance 'd' and is released from rest when it is in this position, find the equation of the subsequent motion. OR Find the half period sine series for f(x) given in the range (I, 0) by the graph ABC as shown in figure.

72 | P a g e

Solution let y(x, t) is the displacement of the string Now, by the one dimensional wave equation we have

The solution of equation (i) is given by y(x, t) = (C1 cos px + C2 sin px) (C3 cos cpt + C4 sin cpt) Now using the boundary conditions as follows The boundary conditions are At x =0 (at A), y =0 and At x =1 (at C), y =0 y(0, t) =0 y(Z, t) =0

From (ii), we have 0 = C1 (C3 cos cpt + C4 sin cpt) using C1 = 0 in equation (ii), we get y(x,t) = C2 sin px (C3 cos cpt + C4 sin cpt) using second boundary condition, from (iii), we have 0 = C2 sin pi (C3 cos cpt + C4 sin cpt) sin pl = 0 sin pi = sin nn rot C1 =0

using the value of p in (iii) we have ( ) ( )

Next, the initial conditions are as follows: velocity = 0 at t =0

and displacement at t =0 is 6. A lightly stretched string of length l with fixed ends is initially in equilibrium position, it is set vibrating by giving each point a velocity V0 sin3 Find the displacement y(x, t). Solution The equation of the vibrating string is The boundary condition are y(0, t) = 0, y (I, t) = 0
73 | P a g e

Also the initial conditions are y(x, 0) = 0 ( )

Since the vibration of the string is periodic, therefore, the solution of (i) is of the form y(x, t) = (C1 cos px + C2 sin px) (C3 cos cpt + C4 sin cpt) by (ii) y(0, t) = C1(C3 cos cpt + C4 sin cpt) =0 For this to be true for all time Ci =0 .. y (x, t) = C2 sin px (C3 cos cpt + C4 sin cpt) Also y (Z, t) = C2 sin pI (C3 cos cpt + C4 sin cpt) =0 for all t. This gives pl= n or p = , n being an integer Thus y(x,t) = C2 ( )

y (x,t) = bn sin sin where bn = C2 C4 Adding all such solutions the general solution of (i) is ( Now ) ( ) )= =

By (iv), V0 sin3 Or (

7. Solve completely the equation

' representing the vibrations of a string of length I,

fixed at both ends, given that y(O, t) =0, y (I, t)= 0 y(x, 0) = f(x), and y (x,O) = 0, O<x<l Solution Here the given equation is

The solution of equation (i) is given by . Y = (C1 cos px + C2 sin px) (C3 cos cpt + C4 sin cpt) Now, applying the boundary conditions y =0 when x =0, we get 0= Cl(C3 cos cpt + C4 sin cpt)
74 | P a g e

C1=0 Therefore, equation (ii) becomes y = C2 sin px (C3 cos cpt + C4 sin cpt) Now putting x =1 and y =0 in equation (iii), we get 0= C2 sin pI (C3 cos cpt + C4 sin cpt) sin pi =0 = sin n or pI = n p=

Thus, equation (iii) becomes Y=C2 ( )

Differentiating equation (iv) with respect to t, we get ( Using given boundary condition 0= C4=0 Thus, equation (iv) becomes Y=C2C3 Now applying the last boundary condition given, we get f(x)=bn Where Bn= bn=C2C3 ( ) we get )

( )

Thus, the required solution is Y=bn Q. 8 Solve the boundary value problem given that y(O, t) =0
75 | P a g e

y(5, t) =0, Y (x, 0) =0 and ( )x=0 = 5sin nx Solution: Applying the method of separation of variables to the wave equation

The suitable solution is y = (C1 cos px + C2 sin px) (C3 cos 2pt + C4 sin 2pt) Applying the initial'condition y(x, 0) =0 we have 0 = C3 (C1cos px + C2 sin px) C3=0 :. y = C4(C1 cos px + C2 sin px) sin 2pt Now using y(0, t) = 0, we get 0 = C1C4 sin 2pt :. y = C2 sin 2pt sin px Further y(S, t) =0 we have C2 sin 2pt sin Sp =0 sin 5 p=0 = sin n p= Therefore, y-C2sin( ) ( ) C1 =0

Also the boundary condition ( ) C2 ( )

n =5 and 2 C2 =5 Therefore, we have Y= sin Q.9 A string of length I is fastened of both ends A and C. At a distance 'a' from the end A, the string is transversely displaced to a distance 'd' and is released from rest when it is in this position, find the equation of the subsequent motion. OR Find the half period sine series for f(x) given in the range (I, 0) by the graph ABC as shown in figure.

76 | P a g e

Solution let y(x, t) is the displacement of the string Now, by the one dimensional wave equation we have

The solution of equation (i) is given by y(x, t) = (C1 cos px + C2 sin px) (C3 cos cpt + C4 sin cpt) Now using the boundary conditions as follows The boundary conditions are At x =0 (at A), y =0 and At x =1 (at C), y =0 y(0, t) =0 y(Z, t) =0

From (ii), we have 0 = C1 (C3 cos cpt + C4 sin cpt) using C1 = 0 in equation (ii), we get y(x,t) = C2 sin px (C3 cos cpt + C4 sin cpt) using second boundary condition, from (iii), we have 0 = C2 sin pi (C3 cos cpt + C4 sin cpt) sin pl = 0 sin pi = sin nn rot C1 =0

using the value of p in (iii) we have ( ) ( )

Next, the initial conditions are as follows: velocity = 0 at t =0

and displacement at t =0 10. Solve the Laplace equation 0 and u (x, a ) = sin Solution. The three possible solution of = 0 subjected to the conditions u(o, y) = U (I, y) = U (x, 0) =

77 | P a g e

=0

are u = (Cl epx + C2 e-px) (C3 cos py + C4 sin py) u = (C5 cos px + C6 sin px) (C7 ePY + Cs e-PY) u = (C9 x +C10) (C11 y + C12) we have to solve (i) satisfying the following boundary conditions u(O, y) =0 u(I, y) =0 u(x, 0) =0 u(x, a) = sin n x/1 using (v) and (vi) in (ii), we get C1 + C2 =0, and C1 epl +C2 e-pl=0 Solving these equations, we get C1= C2 =0, which lead to trivial solution. Similarly we get a trivial solution by using (v) and (vi) in (iv). Hence the suitable for the present problem is solution (iii), using (v) in (iii), we have C5 (C7 ePY + C8 e-PY) =0 i.e. C5 =0 :. (iii) becomes u = C6 sin px (C7 epy + C8 e-PY) using (vi), we have C6 sin pI (C7 ePY + C8 e-PY) =0 :. either C6 =0 or sinpl =0 If we take C6 =0, we get trivial solution Thus sin pI =0 => pI = n p= , where n=0,1,2..
78 | P a g e

(ix) become u=C6 sin(

)(

) ) ( c7 + C8 ) i.e. C8 = -C7

Using (vii), we have 0 = c6 sin(

Thus the solution suitable for this problem is U(x,y)=bn sin ( ) where bn =C6 C7

Now using the condition (viii), we have U(x,a)= sin We get bn =( =bn sin (
)

Hence the required solution is U(x,y) = u(x,y) =


( ( ) )

sin

PART-B (UNSOLVED QUESTION) 1.3 +2 =0 given u(x, 0) =4ex Ans. u(x, y) = 4e1/2(3y-2x) 2.
2+4 2=0

Ans. z= (c1ekx+ c2e-kx) (c3sin1/2 ky +c4cos1/2ky) 3. 2 +u, u(x,0)=6e-3x

Ans. u(x,t) = 6e(-3x+2t) 4. 2 - 3 =0, u(x,0)=5e3x

Ans. u(x,y) = 5e(3x+2y) 5. 3 +2 =0 Ans. u(x,y) = ce1/6k(2x-3y) 6. The vibration of an elastic string is governed by the P.D.E. .
2= 2.

The length of the stribng

is and the ends are fixed.The initial velocity is zero and the initial deflection is u(x, 0)=2(sin x +sin 3x). Find the deflection u(x,t) of the vibrating string for t>0.
79 | P a g e

Ans. u(x,t) = 4cosx cos2tsin 2x 7. Find the displacement of a string ,stretched between the fixed point(0,0) and (1,0) and released from rest from rest from position a sin x+ cos 2 . Ans. u(x,t) = a sin x cos 8. solve the wave equation . (i) u(0,t) = u(l,t) =0 t (ii) u(x,0) ={ (iii) ( ) Ans. u(x,t)= + bsin 2 xcos 2
2 2=c 2 subjected

to the conditions.

A tant string of length 2l is fastended at both ends. The mid point of the string is taken to a height b and then released from the rest in that position. Find the displacement of the string. u(x,t)=
( )

Find the temperature u79x,t) in a homogeneous bar of heat conduction material of length l cm with its ends kept at zero temperature and initital temperature is dx(l-x)/l2 u(x,t)=
( ( ) ) ( )

80 | P a g e

QUESTION BANK FOR


ELECTRICAL
(01BT202)

81 | P a g e

UNIT-1 PART-A

1. Define resistance. Resistance is the property of a material by virtue of which it tends to oppose the flow of current through it. 2. Define inductance. Inductance is that property of a circuit element by which energy is stored in a magnetic flux field. Inductance comes into picture only when there is a time rate of change of current. 3. What is Faradays law of electromagnetic induction? According to this law, whenever the number of magnetic lines of force linked with a closed circuit changes, an induced emf is developed in the circuit and lasts so long as the change lasts. The magnitude of induced emf is equal to the rate of change of flux through the circuit. Thus e = d /dt 4. What is Lenzes law? The law states that the direction of induced emf is such that it opposes the change which is causing the induction. 5. Define capacitance. Capacitance of a conductor is numerically equal to the quantity of charge required to raise the potential of that conductor by unity. 6. What is Amperes Law? Amperes law states that the line integral of magnetic induction around any close path is numerically equal to the product of permeability and current enclosed by the path, i;e., =i

7. What is the maximum transmission voltage in India? The maximum transmission voltage in India is 765KV. 8. The instantaneous voltage and current for an ac circuit are v=155.6sin377tv,i=7.07sin(377t36.870)A. Represent these in phasor diagram. V=155.6 sin377tv I=7.07sin (377t-36.870) A Phasor diagram:
82 | P a g e

V 36.870

9. The normal secondary distribution voltage in our country is.V 220V. 10. If the frequency of the excitation mmf is f. The hysteresis losses and eddy-current losses would be proportional to..and .respectively. f and f2

PART-B 11. What is the formula which is used to express active power in a balanced three-phase circuit? 12. A moving coil instrument gives full scale deflection with 20mA.The resistance of coil is 4ohm .The value of series resistance needed for the instrument to read upto 30V is A moving coil instrument coil instrument gives full scale deflection with 20mA.The Resistance of coil is 4ohm.The value of series resistance needed for the instrument to read Up to 30V is -2.5 13. What is the type of resistors? There are two classes of resistors; fixed resistors and the variable resistors. They are also classified according to the material from which they are made. The typical resistor is made of either carbon film or metal film. There are other types as well, but these are the most common. The resistance value of the resistor is not the only thing to consider when selecting a resistor for use in a circuit. The "tolerance" and the electric power ratings of the resistor are also important. The tolerance of a resistor denotes how close it is to the actual rated resistance value. For example, a 5% tolerance would indicate a resistor that is within 5% of the specified resistance value. The power rating indicates how much power the resistor can safely tolerate. Just like you wouldn't use a 6 volt flashlight lamp to replace a burned out light in your house, you wouldn't use a 1/8 watt resistor when you should be using a 1/2 watt resistor. 14. Derive the method for calculating power from resistance. The maximum rated power of the resistor is specified in Watts. Power is calculated using the square of the current (I2 ) x the resistance value ( R) of the resistor. If the maximum rating of the resistor is exceeded, it will become extremely hot and even burn.
83 | P a g e

Resistors in electronic circuits are typically rated 1/8W, 1/4W, and 1/2W. 1/8W is almost always used in signal circuit applications. When powering a light emitting diode, a comparatively large current flow through the resistor, so you need to consider the power rating of the resistor you choose. 15. What Materials Are Capacitor Plates Made Out Of? By definition, capacitor plates are made of conducting materials. This usually means metals, though other materials are also used. In addition to being conducting, capacitor plates need mechanical strength and resistance to deterioration from electrolytic chemicals. On top of that, most capacitors need extremely thin plates to pack the most capacitance in a small package. Manufacturers use ductile metals to make thin plates from foils. The materials must also be inexpensive and have good availability to be suitable for mass production. Aluminum 1. Aluminum is a workhorse material for making the majority of capacitors. It's inexpensive, highly conductive and easily formed into plates or foils. Tantalum 2. Capacitors that use tantalum are more temperature and frequency stable than those using aluminum, though tantalum costs more. Silver 3. Silver appears in silver-mica capacitors. These also cost more than aluminum plate capacitors, and are used in high-accuracy audio applications. 16. What are the matrials used for electrolytic capacitor? Materials for electrochemical capacitors Electrochemical capacitors, also called super capacitors, store energy using either ion adsorption (electrochemical double layer capacitors) or fast surface redox reactions (pseudo-capacitors). They can complement or replace batteries in electrical energy storage and harvesting applications, when high power delivery or uptake is needed. A notable improvement in performance has been achieved through recent advances in understanding charge storage mechanisms and the development of advanced nanostructures materials. The discovery that ion desolvation occurs in pores smaller than the solvated ions has led to higher capacitance for electrochemical double layer capacitors using carbon electrodes with subnanometre pores, and opened the door to designing high-energy density devices using a variety of electrolytes. Combination of pseudo-capacitive nanomaterials, including oxides, nitrides and polymers, with the latest generation of nanostructure lithium electrodes has brought the energy density of electrochemical capacitors closer to that of batteries. The use of carbon nanotubes has further advanced micro-electrochemical capacitors, enabling flexible and adaptable devices to be made. Mathematical modelling and simulation will be the key to success in designing tomorrow's high-energy and high-power devices. 17. What are the types of capacitor? Types of Capacitor There are a very, very large variety of different types of capacitor available in the market place and each one has its own set of characteristics and applications from small delicate trimming capacitors up to large power metal-can type capacitors used in high voltage power correction and smoothing
84 | P a g e

circuits. Like resistors, there are also variable types of capacitors which allow us to vary their capacitance value for use in radio or "frequency tuning" type circuits. Commercial types of capacitor are made from metallic foil interlaced with thin sheets of either paraffin-impregnated paper or Mylar as the dielectric material. Some capacitors look like tubes, this is because the metal foil plates are rolled up into a cylinder to form a small package with the insulating dielectric material sandwiched in between them. Small capacitors are often constructed from ceramic materials and then dipped into an epoxy resin to seal them. Either way, capacitors play an important part in electronic circuits so here are a few of the more "common" types of capacitor available. Dielectric Capacitor Dielectric Capacitors are usually of the variable type were a continuous variation of capacitance is required for tuning transmitters, receivers and transistor radios. Variable dielectric capacitors are multi-plate air-spaced types that have a set of fixed plates (the stator vanes) and a set of movable plates (the rotor vanes) which move in between the fixed plates. The position of the moving plates with respect to the fixed plates determines the overall capacitance value. The capacitance is generally at maximum when the two sets of plates are fully meshed together. High voltage type tuning capacitors have relatively large spacings or air-gaps between the plates with breakdown voltages reaching many thousands of volts. Film Capacitors which use polystyrene, polycarbonate or Teflon as their dielectrics are sometimes called "Plastic capacitors". The construction of plastic film capacitors is similar to that for paper film capacitors but use a plastic film instead of paper. The main advantage of plastic film capacitors compared to impregnated-paper types is that they operate well under conditions of high temperature, have smaller tolerances, a very long service life and high reliability. Examples of film capacitors are the rectangular metallised film and cylindrical film & foil types as shown below. Ceramic Capacitors Ceramic Capacitors or Disc Capacitors as they are generally called are made by coating two sides of a small porcelain or ceramic disc with silver and are then stacked together to make a capacitor. For very low capacitance values a single ceramic disc of about 3-6mm is used. Ceramic capacitors have a high dielectric constant (High-K) and are available so that relatively high capacitances can be obtained in a small physical size. They exhibit large non-linear changes in capacitance against temperature and as a result are used as de-coupling or by-pass capacitors as they are also nonpolarized devices. Ceramic capacitors have values ranging from a few picofarads to one or two microfarads but their voltage ratings are generally quite low. 18. What are dielectric materials? A dielectric material is a substance that is a poor conductor of electricity, but an efficient supporter of electrostatic field s. If the flow of current between opposite electric charge poles is kept to a minimum while the electrostatic lines of flux are not impeded or interrupted, an electrostatic field can store energy. This property is useful in capacitor s, especially at radio frequencies. Dielectric materials are also used in the construction of radio-frequency transmission lines. In practice, most dielectric materials are solid. Examples include porcelain (ceramic), mica, glass, plastics, and the oxides of various metals. Some liquids and gases can serve as good dielectric materials. Dry air is an excellent dielectric, and is used in variable capacitors and some types of
85 | P a g e

transmission lines. Distilled water is a fair dielectric. A vacuum is an exceptionally efficient dielectric. 19. Write the composition of piezoelectric substance. A piezoelectric substance, which comprises a ternary solid solution having a composition represented by the general formula: xPbZrO3 --yA--zPbTiO3 and having a composition ratio represented by coordinate in a square formed by successively connecting point P (x=0.905; y=0.095; z=0.000), point Q (x=0.960; y=0.040; z=0.000), point R (x=0.932; y=0.005; z=0.063), point S (x=0.905; y=0.005; z=0.090) and said P by straight lines, including the respective lines except axis y in a ternary composition diagram showing the composition of said ternary solid solution, and said A being Pb(Mn 1/3 Nb2/3)O3, Pb(Mn1/2 Nb1/2)O3, Pb(Fe1/2 Sb1/2)O3, Pb(Mn1/2 Ta1/2)O3, Pb(Mn1/3 Sb2/3)O3, Pb(Mn1/2 Sb1/2)O3, Pb(Fe1/2 Nb1/2)O3, Pb(Fe1/3 Sb2/3)O3, Pb(Mn1/3 Ta2/3)O3, Pb(Fe1/3 Nb2/3)O3, Pb(Sb1/2 Ta1/2)O3, Pb(Fe1/3 Ta2/3)O3, Pb(Mn2/3 W1/3)O3, Pb(Sb1/2 Nb 1/2)O3, Pb(Fe1/2 Ta1/2)O3, or Pb(Fe2/3 Te1/3)O3. The piezoelectric substance has a high electromechanical anistoropy in electromechanical coupling factors, a high electromechanical coupling factor of thickness vibration mode, and a low dielectric constant and serves as a distinguished material for ultrasonic vibrator, etc.

20. What is diamagnetism?

Diamagnetism is a fundamental property of all matter, although it is usually very weak. It is due to the non-cooperative behavior of orbiting electrons when exposed to an applied magnetic field. Diamagnetic substances are composed of atoms which have no net magnetic moments (ie., all the orbital shells are filled and there are no unpaired electrons). However, when exposed to a field, a negative magnetization is produced and thus the susceptibility is negative. If we plot M vs H, we see:

Note that when the field is zero the magnetization is zero. The other characteristic behavior of diamagnetic materials is that the susceptibility is temperature independent. Some well known diamagnetic substances, in units of 10-8 m3/kg, include: quartz (SiO2) -0.62,Calcite (CaCO3) 0.48,water -0.90 PART-C 21. Define paramagnetism of substance. Paramagnetism

86 | P a g e

This class of materials, some of the atoms or ions in the material has a net magnetic moment due to unpaired electrons in partially filled orbitals. One of the most important atoms with unpaired electrons is iron. However, the individual magnetic moments do not interact magnetically, and like diamagnetism, the magnetization is zero when the field is removed. In the presence of a field, there is now a partial alignment of the atomic magnetic moments in the direction of the field, resulting in a net positive magnetization and positive susceptibility.

In addition, the efficiency of the field in aligning the moments is opposed by the randomizing effects of temperature. This results in a temperature dependent susceptibility, known as the Curie Law. At normal temperatures and in moderate fields, the paramagnetic susceptibility is small (but larger than the diamagnetic contribution). Unless the temperature is very low (<<100 K) or the field is very high paramagnetic susceptibility is independent of the applied field. Under these conditions, paramagnetic susceptibility is proportional to the total iron content. Many iron bearing minerals are paramagnetic at room temperature. Some examples, in units of 10-8 m3/kg, include: Montmorillonite (clay) 13, Nontronite (carbonate) 100, Pyrite (sulfide) 30 (Fe-rich clay) 65, Biotite (silicate) 79, Siderite

The paramagnetism of the matrix minerals in natural samples can be significant if the concentration of magnetite is very small. In this case, a paramagnetic correction may be needed. 22. What are ferromagnetic substances? Ferromagnetism: When you think of magnetic materials, you probably think of iron, nickel or magnetite. Unlike paramagnetic materials, the atomic moments in these materials exhibit very strong interactions. These interactions are produced by electronic exchange forces and result in a parallel or antiparallel alignment of atomic moments. Exchange forces are very large, equivalent to a field on the order of 1000 Tesla, or approximately a 100 million times the strength of the earth's field. The exchange force is a quantum mechanical phenomenon due to the relative orientation of the spins of two electrons. Ferromagnetic materials exhibit parallel alignment of moments resulting in large net magnetization even in the absence of a magnetic field. The elements Fe, Ni, and Co and many of their alloys are typical ferromagnetic materials. Two distinct characteristics of ferromagnetic materials are their (1) Spontaneous magnetization and the existence of (2) Magnetic ordering temperature
87 | P a g e

Spontaneous Magnetization The spontaneous magnetization is the net magnetization that exists inside a uniformly magnetized microscopic volume in the absence of a field. The magnitude of this magnetization, at 0 K, is dependent on the spin magnetic moments of electrons. A related term is the saturation magnetization which we can measure in the laboratory. The saturation magnetization is the maximum induced magnetic moment that can be obtained in a magnetic field (Hsat); beyond this field no further increase in magnetization occurs. 23. The difference between spontaneous magnetization and the saturation magnetization has to do with magnetic domains (more about domains later). Saturation magnetization is an intrinsic property, independent of particle size but dependent on temperature. 24. There is a big difference between paramagnetic and ferromagnetic susceptibility. As compared to paramagnetic materials, the magnetization in ferromagnetic materials is saturated in moderate magnetic fields and at high (room-temperature) temperatures: Hsat Tesla T range (K) c 10-8m3/kg paramagnetic >10 <<100 ~300 ~50 1000-10000 ferromagnetism ~1

23. What are anti ferromagnetism? ANTI FERROMAGNETISM

If the A and B sublattice moments are exactly equal but opposite, the net moment is zero. This type of magnetic ordering is called ant ferromagnetism.

The clue to antiferromagnetism is the behavior of susceptibility above a critical temperature, called the Nel temperature (TN). Above TN, the susceptibility obeys the Curie-Weiss law for paramagnets but with a negative intercept indicating negative exchange interactions.

88 | P a g e

25. Write the magnetic properties of minerals. Magnetic Properties of Minerals Mineral Oxides Magnetite Ulvospinel Hematite Ilmenite Maghemite Jacobsite Trevorite Sulfides Pyrrhotite Greigite Troilite Oxyhydroxides Goethite Lepidocrocite Feroxyhyte Metals & Alloys Iron Nickel Cobalt Awaruite Wairauite Fe Ni Co Ni3Fe CoFe FM FM FM FM FM 770 358 1131 620 986 55 161 120 235 aFeOOH gFeOOH dFeOOH AFM, weak FM ~120 AFM(?) ferrimagnetic -196 ~180 <10 <1 Fe7S8 Fe3S4 FeS ferrimagnetic ferrimagnetic AFM 320 ~333 305 ~20 ~25 Fe3O4 Fe2TiO2 aFe2O3 FeTiO2 gFe2O3 MNFe2O4 NiFe2O4 ferrimagnetic AFM canted AFM AFM ferrimagnetic ferrimagnetic ferrimagnetic ferrimagnetic 575-585 90-92 -153 675 -233 ~600 300 585 440 ~80 77 51 21 0.4 Composition Magnetic Order Tc(C) ss (Am2/kg)

Magnesioferrite MgFe2O4

FM = ferromagnetic order AFM = antiferromagnetic order Tc = Curie or Nel Temperature ss = saturation magnetization at room-temperature 26. What are diamagnetic, paramagnetic and diamagnetic substances? Ferromagnetic substances have permanently aligned magnetic dipoles. In the presence of the external field the sample moves toward the strong field, attaching itself to the pointed pole. Paramagnetic substances have permanent but unaligned magnetic dipoles (thermal random motion de-aligns them, that's why the effect is stronger at low temperatures). An external
89 | P a g e

magnetic field does align the dipoles however, and the substance will move toward the strong field and attach itself to the pointed pole. Diamagnetic substances have no intrinsic magnetic dipole. However, a strong external field can induce a dipole, but in the opposite direction to the applied field (Lenz's Law). A diamagnetic substance will therefore move away from the strong field. Diamagnetic effects are weak (all substances are diamagnetic, but it is swamped by any Para or ferromagnetic property), so its movement towards the flat pole is subtle. 27. What is inductance and what is its storage energy? Inductance (measured in henry) is an effect which results from the magnetic field that forms around a current carrying conductor. Current flowing through the inductor creates a magnetic field which has an associated electromotive field which opposes the applied voltage. This counter electromotive (emf) is generated which opposes the change in voltage applied to the inductor and current in the inductor resists the change but does rise. This is known as inductive reactance. It is opposite in phase to capacitive reactance. Inductance can be increased by looping the conductor into a coil which creates a larger magnetic field. Stored energy The energy (measured in joules, in SI) stored by an inductor is equal to the amount of work required to establish the current flowing through the inductor, and therefore the magnetic field. This is given by:

Where L is inductance and I is the current flowing through the inductor 28. Write inductor construction and its application. Inductor construction: An inductor is usually constructed as a coil of conducting material, typically copper wire, wrapped around a core either of air or of ferrous material. Core materials with a higher permeability than air confine the magnetic field closely to the inductor, thereby increasing the inductance. Inductors come in many shapes. Most are constructed as enamel coated wire wrapped around a ferrite bobbin with wire exposed on the outside, while some enclose the wire completely in ferrite and are called "shielded". Some inductors have an adjustable core, which enables changing of the inductance. Small inductors can be etched directly onto a printed circuit board by laying out the trace in a spiral pattern. Small value inductors can also be built on integrated circuits using the same processes that are used to make transistors. In these cases, aluminum interconnect is typically used as the conducting material. However, practical constraints make it far more common to use a circuit called a "gyrator" which uses a capacitor and active components to behave similarly to an inductor. Inductors used to block very high frequencies are sometimes made with a wire passing through a ferrite cylinder or bead. Applications:

90 | P a g e

Inductors are used extensively in analog circuits and signal processing. Inductors in conjunction with capacitors and other components form tuned circuits which can emphasize or filter out specific signal frequencies. This can range from the use of large inductors as chokes in power supplies, now obsolete, which in conjunction with filter capacitors remove residual hum or other fluctuations from the direct current output, to such small inductances as generated by a ferrite bead or toroid (torus) around a cable to prevent radio frequency interference from being transmitted down the wire. Smaller inductor/capacitor combinations provide tuned circuits used in radio reception and broadcasting, for instance. Two (or more) inductors which have coupled magnetic flux form a transformer, which is a fundamental component of every electric utility power grid. The efficiency of a transformer increases as the frequency increases; for this reason, aircraft used 400 hertz alternating current rather than the usual 50 or 60 hertz, allowing a great savings in weight from the use of smaller transformers. An inductor is used as the energy storage device in a switched-mode power supply. The inductor is energized for a specific fraction of the regulator's switching frequency, and de-energized for the remainder of the cycle. This energy transfer ratio determines the input-voltage to output-voltage ratio. This XL is used in complement with an active semiconductor device to maintain very accurate voltage control. 29. Write the basic formulae used for inductor.

1. Basic inductance formula: L = Inductance in henries 0 = permeability of free space = 4 10-7 H/m r = relative permeability of core material N = number of turns A = area of cross-section of the coil in square metres (m2) l = length of coil in metres (m)

2. Inductance of a straight wire conductor: L = inductance in nH l = length of conductor d = diameter of conductor in the same units as l (note: the following formulas were optimized to be used with imperial units) 3. Inductance of air core inductor in terms of geometric parameters: L = inductance in H r = outer radius of coil in inches l = length of coil in inches N = number of turns 4. For multilayered air core coil: L = inductance in H r = mean radius of coil in inches
91 | P a g e

l = length of coil in inches N = number of turns d = depth of coil in inches 5. Inductance of a spring coil: L = inductance in H r = mean radius of coil in inches N = number of turns d = depth of coil in inches 30. What is the equivalent caspacitance when two capacitors are connected in series and parallel? The series and parallel combination of capacitors is important in the electronic repair line. Not every Capacitor value you can find from the electronic suppliers thus you have to use the series and parallel capacitor formula to calculate your desire capacitor value and voltage rating for replacement. It is quite simple to find a replacement with the formula given below Capacitor In Series

If two or more capacitors are connected in series as shown above, the total capacitance is less than that of the smallest capacitor in the group. The value of the total capacitance C can be calculated by the formula below. The results are exactly the same as the resistances in parallel. If the capacitors of 1 microfarad each are connected in series, the net capacitance value offered by the combination of the two capacitors is: -

So the net value is just half of an individual capacitor. Capacitors are used in series to provide a higher voltage breakdown rating for the combination. In many circuits,high value resistors are connected in parallel with all capacitors to ensure that the voltage is distributed equally. These resistors should have enough wattage rating to dissipate heat generated by the flow of current through them. When electrolytic capacitors are to be connected in series, connect the positive side of the capacitor to negative exactly as when you are connecting batteries in series. (Fig. 1)

92 | P a g e

In series capacitors, the working voltage of each capacitor is sum up. This series connection decreased the capacitance but an increase in working voltage.

The total value of this combination is: -

but the working voltage becomes 700 volt. Capacitors in Parallel When a number of capacitors are connected in parallel as shown below, the total or effective capacitance of the group is equal to the sum of the individual capacitances The equation for calculating the total capacitance C obtained by capacitances C1, C2 C3 etc.The formula for parallel capacitor is same as the resistance in series.

If 5, 10 and 15 microfarad capacitors are connected in parallel as shown below, the combination would provide a total value capacitance of 30 microfarad. This type of connection is used to get higher capacitances. The voltage rating of each capacitor should be sufficient to withstand the voltage that will be applied to them in practical circuits.

When electrolytic capacitors are connected in parallel, make sure the capacitor polarity connect from positive to positive and negative to negative otherwise the capacitor may not work or blow.

93 | P a g e

The working voltage of parallel capacitors is equal to the lowest working voltage rating in the combination. For example, the total value capacitance of the combination is:CT = C1 + C2 + C3

CT = 1F + 2F + 1F = 4F and working voltage = 100 volt Series and parallel capacitors formula is a good way to find a capacitance value for replacement. Use a digital capacitance meter and test all the results you have connected the capacitors either in series or in parallel.

94 | P a g e

UNIT-2 PART-A 1. Define Ohms law? This law stating that the direct current flowing in a conductor is directly proportional to the potential difference between its ends. It is usually formulated as V = IR, where V is the potential difference, or voltage, I is the current, and R is the resistance of the conductor. 2. What is Kirchhoffs first law? At any node in an electrical circuit, the sum of currents flowing into that node is equal to the sum of currents flowing out of that node. 3. What is Kirchhoffs second law? Almost as famous and useful as Ohm's Law, Kirchhoff's Laws address current into and out of a node, and voltage drops and rises around a closed loop: The sum of the currents flowing into a node must equal the sum of the currents flowing out. The sum of the voltage drops around a closed loop must equal the sum of the voltage rises. Kirchhoffs voltage law: - it states that sum of all emfs and voltage drops is equal to zero Kirchhoffs current law: - at a node, incoming currents is equal to outgoing currents. 4. Compare Kirchhoffs voltage law to Ohms law. Kirchhoffs voltage law states that the sum of the signed voltage rises in a series circuit adds up to zero. A consequence of this is that the voltages across elements in a parallel circuit are equal to each other. Ohm's law states that voltage is equal to resistance times current. The two laws cannot be compared, because they talk about different things. They are, however, tools that can be used together, along with other laws such as Kirchhoffs current law, and Norton and Thevanin equivalents, to perform circuit analysis. 5. What conservation laws do Kirchhoff's laws uphold? Kirchhoffs laws conserve current and voltage: the sum of the current into a node is the same out of a node; and the sum of the voltages around a loop is zero. 6. What is Mesh analysis in network theorem? Mesh analysis works by arbitrarily assigning mesh currents in the essential meshes. An essential mesh is a loop in the circuit that does not contain any other loop. When looking at a circuit schematic, the essential meshes look like a window pane. Figure 1 labels the essential meshes with one, two, and three. Once the essential meshes are found, the mesh currents need to be labeled. A mesh current is a current that loops around the essential mesh. The mesh current might not have a physical meaning but it is used to set up the mesh analysis equations. When assigning the mesh currents it is important to have all the mesh currents loop in the same direction. This will help prevent errors when writing out the equations. The convention is to have all the mesh currents looping in a clockwise direction.[2] Figure 2 shows the same circuit shown before but with the mesh currents labeled.
95 | P a g e

The reason to use mesh currents instead of just using KCL and KVL to solve a problem is that the mesh currents can account for any unnecessary currents that may be drawn in when using KCL and KVL. Mesh analysis ensures that the least possible number of equations regarding currents is used, greatly simplifying The problem.

7. What are the assumptions made in the Mesh analysis? A. Always assume that the mesh in which KVL is applied has meshed current higher than the other. B. Always assume the direction of mesh currents in clockwise direction. C. Always take voltage rise as negative and voltage drop as positive. 8. What is superposition theorem? Superposition theorem is one of those strokes of genius that takes a complex subject and simplifies it in a way that makes perfect sense. A theorem like Millman's certainly works well, but it is not quite obvious why it works so well. Superposition, on the other hand, is obvious. The strategy used in the Superposition Theorem is to eliminate all but one source of power within a network at a time, using series/parallel analysis to determine voltage drops (and/or currents) within the modified network for each power source separately. Then, once voltage drops and/or currents have been determined for each power source working separately, the values are all superimposed on top of each other (added algebraically) to find the actual voltage drops/currents with all sources active. Let's look at our example circuit again and apply Superposition Theorem to it:

Since we have two sources of power in this circuit, we will have to calculate two sets of values for voltage drops and/or currents, one for the circuit with only the 28 volt battery in effect.

96 | P a g e

9. What is Thevnins theorem? Thevenin's Theorem states that it is possible to simplify any linear circuit, no matter how complex, to an equivalent circuit with just a single voltage source and series resistance connected to a load. The qualification of linear is identical to that found in the Superposition Theorem, where all the underlying equations must be linear (no exponents or roots). If we're dealing with passive components (such as resistors, and later, inductors and capacitors), this is true. However, there are some components (especially certain gas-discharge and semiconductor components) which are nonlinear: that is, their opposition to current changes with voltage and/or current. As such, we would call circuits containing these types of components, nonlinear circuits. Thevenin's Theorem is especially useful in analyzing power systems and other circuits where one particular resistor in the circuit (called the load resistor) is subject to change, and re -calculation of the circuit is necessary with each trial value of load resistance, to determine voltage across it and current through it. Let's take another look at our example circuit:

10. Write the procedure for solving problems using Thevnins theorem. Following is the procedures for solving problems using Thevnins theorem: 1. Remove the element through which the current is required (load element). 2. Calculate the open circuit voltage Vth. 3. Calculate the open circuit resistance Rth. 4. Draw Thevnin equivalent circuit as shown in the figure. 5. Reconnect Rl back to AB. 6. Current Il can now be easily calculated.

97 | P a g e

PART-B 11. What is Nortons theorem? Norton's Theorem states that it is possible to simplify any linear circuit, no matter how complex, to an equivalent circuit with just a single current source and parallel resistance connected to a load. Just as with Thevenin's Theorem, the qualification of linear is identical to that found in the Superposition Theorem: all underlying equations must be linear (no exponents or roots). Contrasting our original example circuit against the Norton equivalent: it looks something like this:

12. Write the Thevnins and Norton equivalent. Since Thevenin's and Norton's Theorems are two equally valid methods of reducing a complex network down to something simpler to analyze, there must be some way to convert a Thevenin equivalent circuit to a Norton equivalent circuit, and vice versa (just what you were dying to know, right?). Well, the procedure is very simple. You may have noticed that the procedure for calculating Thevenin resistance is identical to the procedure for calculating Norton resistance: remove all power sources and determine resistance between the open load connection points. As such, Thevenin and Norton resistances for the same original network must be equal. Using the example circuits from the last two sections, we can see that the two resistances are indeed equal:

13. What is maximum power transfer theorem? The Maximum Power Transfer Theorem is not so much a means of analysis as it is an aid to system design. Simply stated, the maximum amount of power will be dissipated by a load resistance when
98 | P a g e

that load resistance is equal to the Thevenin/Norton resistance of the network supplying the power. If the load resistance is lower or higher than the Thevenin/Norton resistance of the source network, its dissipated power will be less than maximum. This is essentially what is aimed for in radio transmitter design, where the antenna or transmission line impedance is matched to final power amplifier impedance for maximum radio frequency power output. Impedance, the overall opposition to AC and DC current, is very similar to resistance, and must be equal between source and load for the greatest amount of power to be transferred to the load. A Load impedance that is too high will result in low power output. A load impedance that is too low will not only result in low power output, but possibly overheating of the amplifier due to the power dissipated in its internal (Thevenin or Norton) impedance. Taking our Thevenin equivalent example circuit, the Maximum Power Transfer Theorem tells us that the load resistance resulting in greatest power dissipation is equal in value to the Thevenin resistance (in this case, 0.8 ):

14. What is Millimans theorem equation? Millman's Theorem is nothing more than a long equation, applied to any circuit drawn as a set of parallel-connected branches, each branch with its own voltage source and series resistance:

15. What is Millmans theorem? In Millman's Theorem, the circuit is re-drawn as a parallel network of branches, each branch containing a resistor or series battery/resistor combination. Millman's Theorem is applicable only to those circuits which can be re-drawn accordingly. Here again is our example circuit used for the last two analysis methods:

99 | P a g e

And here is that same circuit, re-drawn for the sake of applying Millman's Theorem:

By considering the supply voltage within each branch and the resistance within each branch, Millman's Theorem will tell us the voltage across all branches. Please note that I've labeled the battery in the rightmost branch as B3 to clearly denote it as being in the third branch, even though there is no B2 in the circuit! 16. What is Star-Delta transformation? A star network of three impedances ZAN, ZBN and ZCN connected together at common node N can be transformed into a delta network of three impedances ZAB,ZBC and ZCA by the following equations: ZAB = ZAN + ZBN + (ZANZBN / ZCN) = (ZANZBN + ZBNZCN + ZCNZAN) / ZCN ZBC = ZBN + ZCN + (ZBNZCN / ZAN) = (ZANZBN + ZBNZCN + ZCNZAN) / ZAN ZCA = ZCN + ZAN + (ZCNZAN / ZBN) = (ZANZBN + ZBNZCN + ZCNZAN) / ZBN Similarly, using admittances: YAB = YANYBN / (YAN + YBN + YCN) YBC = YBNYCN / (YAN + YBN + YCN) YCA = YCNYAN / (YAN + YBN + YCN) In general terms: Zdelta = (sum of Zstar pair products) / (opposite Zstar) Ydelta = (adjacent Ystar pair product) / (sum of Ystar) 17. What is Joules law? When a current I is passed through a resistance R, the resulting power P dissipated in the resistance is equal to the square of the current I multiplied by the resistance R: P = I 2R

100 | P a g e

By substitution using Ohm's Law for the corresponding voltage drop V (= IR) across the resistance: P = V2 / R = VI = I2R 18. What is reciprocity theorem? If a voltage source E acting in one branch of a network causes a current I to flow in another branch of the network, then the same voltage source E acting in the second branch would cause an identical current I to flow in the first branch. 19. What is Network analysis? Generally speaking, network analysis is any structured technique used to mathematically analyze a circuit (a network of interconnected components). Quite often the technician or engineer will encounter circuits containing multiple sources of power or component configurations which defy simplification by series/parallel analysis techniques. In those cases, he or she will be forced to use other means. This chapter presents a few techniques useful in analyzing such complex circuits. To illustrate how even a simple circuit can defy analysis by breakdown into series and parallel portions, take start with this series-parallel circuit:

To analyze the above circuit, one would first find the equivalent of R2 and R3 in parallel, then add R1 in series to arrive at a total resistance. Then, taking the voltage of battery B 1 with that total circuit resistance, the total current could be calculated through the use of Ohm's Law (I=E/R), then that current figure used to calculate voltage drops in the circuit. 20. What is Network theorem? Anyone whose studied geometry should be familiar with the concept of a theorem: a relatively simple rule used to solve a problem, derived from a more intensive analysis using fundamental rules of mathematics. At least hypothetically, any problem in math can be solved just by using the simple rules of arithmetic (in fact, this is how modern digital computers carry out the most complex mathematical calculations: by repeating many cycles of additions and subtractions!), but human beings aren't as consistent or as fast as a digital computer. We need shortcut methods in order to avoid procedural errors. In electric network analysis, the fundamental rules are Ohm's Law and Kirchhoff's Laws. While these humble laws may be applied to analyze just about any circuit configuration (even if we have to resort to complex algebra to handle multiple unknowns), there are some shortcut methods of analysis to make the math easier for the average human.
101 | P a g e

As with any theorem of geometry or algebra, these network theorems are derived from fundamental rules. In this chapter, I'm not going to delve into the formal proofs of any of these theorems. If you doubt their validity, you can always empirically test them by setting up example circuits and calculating values using the old (simultaneous equation) methods versus the new theorems, to see if the answers coincide. PART-C 21. What is compensation theorem? if the impedance Z of a branch in a network in which a current I flows is changed by a finite amount dZ, then the change in the currents in all other branches of the network may be calculated by inserting a voltage source of -IdZ into that branch with all other voltage sources replaced by their internal impedances. 22. What is delta-star transformation? A delta network of three impedances ZAB, ZBC and ZCA can be transformed into a star network of three impedances ZAN, ZBN and ZCN connected together at common node N by the following equations: ZAN = ZCAZAB / (ZAB + ZBC + ZCA) ZBN = ZABZBC / (ZAB + ZBC + ZCA) ZCN = ZBCZCA / (ZAB + ZBC + ZCA) Similarly, using admittances: YAN = YCA + YAB + (YCAYAB / YBC) = (YABYBC + YBCYCA + YCAYAB) / YBC YBN = YAB + YBC + (YABYBC / YCA) = (YABYBC + YBCYCA + YCAYAB) / YCA YCN = YBC + YCA + (YBCYCA / YAB) = (YABYBC + YBCYCA + YCAYAB) / YAB In general terms: Zstar = (adjacent Zdelta pair product) / (sum of Zdelta) Ystar = (sum of Ydelta pair products) / (opposite Ydelta) 23. Write Thevnin model from Norton model Thevnin model from Norton model Voltage = Current / Admittance Impedance = 1 / Admittance

E=I/Y Z = Y -1

24. Write Norton model from Thevnin model. Norton model from Thevnin model Current = Voltage / Impedance Admittance = 1 / Impedance I=E/Z Y = Z -1

When performing network reduction for a Thevnin or Norton model, note that: - nodes with zero voltage difference may be short-circuited with no effect on the network current distribution,
102 | P a g e

- branches carrying zero current may be open-circuited with no effect on the network voltage distribution. 26. Derive Maximum power transfer theorem. When the impedance of a load connected to a power source is varied from open-circuit to shortcircuit, the power absorbed by the load has a maximum value at a load impedance which is dependent on the impedance of the power source. Note that power is zero for an open-circuit (zero current) and for a short-circuit (zero voltage). Voltage Source When a load resistance RT is connected to a voltage source ES with series resistance RS, maximum power transfer to the load occurs when RT is equal to RS. Under maximum power transfer conditions, the load resistance RT, load voltage VT, load current IT and load power PT are: RT = RS VT = ES / 2 IT = VT / RT = ES / 2RS PT = VT2 / RT = ES2 / 4RS 27. What is complex impedance? When a load impedance ZT (comprising variable resistance RT and variable reactance XT) is connected to an alternating voltage source ES with series impedance ZS (comprising resistance RS and reactance XS), maximum power transfer to the load occurs when ZT is equal to ZS* (the complex conjugate of ZS) such that RT and RS are equal and XT and XS are equal in magnitude but of opposite sign (one inductive and the other capacitive). When a load impedance ZT (comprising variable resistance RT and constant reactance XT) is connected to an alternating voltage source ES with series impedance ZS (comprising resistance RS and reactance XS), maximum power transfer to the load occurs when RT is equal to the magnitude of the impedance comprising ZS in series with XT: RT = |ZS + XT| = (RS2 + (XS + XT)2)* Note that if XT is zero, maximum power transfer occurs when RT is equal to the magnitude of ZS: RT = |ZS| = (RS2 + XS2)* When a load impedance ZT with variable magnitude and constant phase angle (constant power factor) is connected to an alternating voltage source ES with series impedance ZS, maximum power transfer to the load occurs when the magnitude of ZTis equal to the magnitude of ZS: (RT2 + XT2)* = |ZT| = |ZS| = (RS2 + XS2)* 28. Write Millmans Theorem (parallel generator theorem). Millman's Theorem (Parallel Generator Theorem) If any number of admittances Y1, Y2, Y3, ... meet at a common point P, and the voltages from another point N to the free ends of these admittances are E1, E2, E3, ... then the voltage between
103 | P a g e

points P and N is: VPN = (E1Y1 + E2Y2 + E3Y3 + ...) / (Y1 + Y2 + Y3 + ...) VPN = SEY / SY The short-circuit currents available between points P and N due to each of the voltages E1, E2, E3, ... acting through the respective admittances Y1, Y2, Y3, ... areE1Y1, E2Y2, E3Y3, ... so the voltage between points P and N may be expressed as: VPN = SIsc / SY 29. Find the power in the 2W resistor R6 using Thevenin's theorem.

Step One Remove R6 from the circuit

Step Two Redraw the circuit after finding the resistance, RA of the parallel branch R3||R4.

Redraw the circuit and calculate the voltage across the break AB which is the voltage across RA = VT. Remember that 20V is dropped across R1 and also across R2+RA. Since R2 = RA = 5W half of the voltage is dropped across each resistor VT = 10V

104 | P a g e

Step Three Remove V1 and replace it with a short circuit

This action also short circuits R1 out of the circuit

Step Four Determine Resistance looking into the break AB = RT

RT = (RAxR2) / (RA+R2) + R5 + R7 = 14.5W Step Five Determine the value of the current from the Thevenin equivalent circuit
105 | P a g e

I = VT/ (RT + R6) = 10/16.5 = 0.606A Power dissipated by R6 = I x I x R = 0.73 Watts 30. What are the fundamental parameters in RLC circuit? There are two fundamental parameters that describe the behavior of RLC circuits: the resonant frequency and the damping factor. In addition, other parameters derived from these first two are discussed below. Resonant frequency: The undamped resonance or natural frequency of an RLC circuit (in radians per second) is:

In the more familiar unit hertz, the natural frequency becomes

Damping factor: The damping

factor

of

series

RLC

circuit

(in

radians

per

second)

is:

For a parallel RLC circuit is

Derived Parameters Bandwidth: The RLC circuit may be used as a band pass or band-stop filter by replacing R with a receiving device with the same input resistance, and the bandwidth (in radians per second) is

alternatively, the bandwidth in hertz is

The bandwidth is a measure of the width of the frequency response at the two half-power frequencies. As a result, this measure of bandwidth is sometimes called the full-width at halfpower. Since electrical power is proportional to the square of the circuit voltage (or current), the Frequency response will drop to at the half-power frequencies. Quality or Q factor: The Quality of the series tuned circuit, or Q factor, is calculated as the ratio of the resonance frequency o to the bandwidth ss (in radians per second):

Or in hertz:
106 | P a g e

For the parallel tuned circuit:

Q is a dimensionless quantity.

107 | P a g e

UNIT-3
PART-A 1. What is self induction? The flux due to the current in the coil links that coil and if the current changes the resulting flux change induces an e.m.f. in the coil itself. The process is called self-induction. 2. Write the equation of the self inductance. If the e.m.f. induced in the coil is , when the rate of change of current in it is dI/dt, The inductance L of the coil is defined by the equation L= - /(dI/dt) 3. What is the time constant of the circuit?

The solution to the above equation can be shown to be

.......... (1) Where

Is known as the time-constant of the circuit.. 4. What is the method of calculation of self inductance? Calculation of self-inductance, L of the coil At the rising edge of the square wave

When

Substituting the above expressions into equation (1) and solving the equation,

The same expression is obtained at the falling edge of the square wave. 5. What is the RC time constant of a series RC circuit that contains a 12-megohm resistor and a 12-microfarad capacitor?

108 | P a g e

Universal Time Constant Because the impressed voltage and the values of R and C or R and L in a circuit are usually known, a universal time constant chart ,can be used to find the time constant of the circuit. Curve A is a plot of both capacitor voltages during charge and inductor current during growth. Curve B is a plot of both capacitor voltages during discharge and inductor current during decay. 6. What is quality factor? Ans. It is defined as the ratio of the circulating current to the line current or the current magnification. Qfactor = 7. What is a parallel circuit?

Ans: A parallel circuit is one in which components are connected in parallel. i.e. resistor, inductor and /Or capacitor are connected in parallel. 8. What is series circuit? A series circuit is one in which component such as resistor; capacitor and/or inductor are connected in series. 9. What are active elements? Those elements which are responsible for generation of current are known as active elements of the we electrical circuit such that voltage source, transistor, battery, source of emf. 10. What are passive elements? Those elements which are responsible for generation of current are known as active elements of the electrical circuit such that voltage source, transistor, battery, source of EmF.

PART-B 11. What is R L circuit? A resistor-inductor circuit (RL circuit), or RL filter or RL network, is one of the simplest analogue infinite impulse response electronic filters. It consists of a resistor and an inductor, either in series or in parallel, driven by a voltage source. 12. What is R C circuit? A resistorcapacitor circuit (RC circuit), or RC filter or RC network, is an electric circuit composed of resistors and capacitors driven by a voltage or current source. A first order RC circuit is composed of one resistor and one capacitor and is the simplest type of RC circuit. RC circuits can be used to filter a signal by blocking certain frequencies and passing others. The four most common RC filters are the high-pass filter, low-pass filter, band-pass filter and band-stop filter.
109 | P a g e

13. What is LC circuit? An LC circuit is a resonant circuit or tuned circuit that consists of an inductor, represented by the letter L, and a capacitor, represented by the letter C. When connected together, an electric current reaches a maximum at the circuit's resonant frequency.LC circuits are used either for generating signals at a particular frequency, or picking out a signal at a particular frequency from a more complex signal. They are key components in many applications such as oscillators, filters, tuners and frequency mixers. An LC circuit is an idealized model since it assumes there is no dissipation of energy due to resistance. For a model incorporating resistance see RLC circuit. While no practical circuit is without losses, it is instructive, nonetheless, to study this pure form to gain a good understanding. 14. What is R L C circuit? An RLC circuit (or LCR circuit) is an electrical circuitss consisting of a resistor, an inductor, and a capacitor, connected in series or in parallel. The RLC part of the name is due to those letters being the usual electrical symbols for resistance, inductance and capacitance respectively. The circuit forms a harmonic oscillator for current and will resonate in just the same way as an LC circuit will. The difference that the presence of the resistor makes is that any oscillation induced in the circuit will die away over time if it not kept going by a source. This effect of the resistor is called damping some resistance is unavoidable in real circuits, even if a resistor is not specifically included as a component. A pure LC circuit is an ideal which really only exists in theory. There are many applications for this circuit. They are used in many different types of oscillator circuit. Another important application is for tuning, such as in radio receivers or television sets, where they are used to select a narrow range of frequencies from the ambient radio waves. In this role the circuit is often referred to as a tuned circuit. An RLC circuit can be used as a band-pass filter or a band-stop filter. The tuning application, for instance, is an example of band-pass filtering. The RLC filter is described as a second-order circuit, meaning that any voltage or current in the circuit can be described by a second-order differential equation in circuit analysis. The three circuit elements can be combined in a number of different topologies All three elements in series or all three elements in parallel are the simplest in concept and the most straightforward to analyze. 15. Explain parallel Resonance? In simple reactive circuits with little or no resistance, the effects of radically altered impedance will manifest at the resonance frequency predicted by the equation given earlier. In a parallel (tank) LC circuit, this means infinite impedance at resonance. In a series LC circuit, it means zero impedance at resonance:

However, as soon as significant levels of resistance are introduced into most LC circuits, this simple calculation for resonance becomes invalid. We'll take a look at several LC circuits with added resistance, using the same values for capacitance and inductance as before: 10 F and 100 mH, respectively. According to our simple equation, the resonant frequency should be 159.155 Hz. Watch, though, where current reaches maximum or minimum in the following SPICE analyses:

110 | P a g e

PART B
16. What is series resonance? series resonance A similar effect happens in series inductive/capacitive circuits. (Figure below) When a state of resonance is reached (capacitive and inductive reactances equal), the two impedances cancel each other out and the total impedance drops to zero!

Simple series resonant circuit.

With the total series impedance equal to 0 at the resonant frequency of 159.155 Hz, the result is a short circuit across the AC power source at resonance. In the circuit drawn above, this would not be good. I'll add a small resistor (Figure below) in series along with the capacitor and the inductor to keep the maximum circuit current somewhat limited, and perform another SPICE analysis over the same range of frequencies: (Figure below)

111 | P a g e

17. What is bandwidth? Bandwidth is the difference between the upper and lower frequencies in a contiguous set of frequencies. It is typically measured in hertz, and may sometimes refer to passband bandwidth, sometimes to baseband bandwidth, depending on context. Passband bandwidth is the difference between the upper and lower cutoff frequencies of, for example, an electronic filter, a communication channel, or a signal spectrum. Bandwidth is measured in hertz. For analog signals, which can be mathematically viewed as functions of time, bandwidth, BW or f is the width, measured in hertz, of the frequency range. 18. Derive the expression for RC parallel circuit?

112 | P a g e

The procedure for deriving the equation for the discharging capacitor is similar, though you have to use i = -dq/dt because dq/dt is negative for a discharging capacitor, by definition. 19. What is power factor? The power factor of an AC electric power system is defined as the ratio of the real power flowing to the load to the apparent power in the circuit, and is a dimensionless number between 0 and 1 (frequently expressed as a percentage, e.g. 0.5 pf = 50% pf). Real power is the capacity of the circuit for performing work in a particular time. Apparent power is the product of the current and voltage of the circuit. Due to energy stored in the load and returned to the source, or due to a non-linear load that distorts the wave shape of the current drawn from the source, the apparent power will be greater than the real power. 20. A voltage of 120 V at 50 Hz is applied to a resistance, R in series with a capacitance, C. The current drawn is 2 A, and the power loss in the resistance is 100 W. Calculate the resistance. Solution V= 120 V I=2A P = 100 W f = 50 Hz R = 100/4 =25 ohms PART-C 21. Write differences between series resonance and parallel resonance?

series resonance parallel resonance The impedance of the circuit is minimum and The admittance of the circuit is minimum and equals the resistance of circuit equals the conductance of circuit. The current drawn is maximum. The current drawn is minimum. The p f is unity. The p f is unity. 1/2 Resonance frequency fr=(1/2pi)(1/LC) Resonance frequency fr=(1/2pi)(1/LC)1/2

113 | P a g e

22. Find equivalent inductance? Inductors Inductors follow the same law, in that the total inductance of non-coupled inductors in parallel is equal to the reciprocal of the sum of the reciprocals of their individual inductances:

. 23. Find equivalent capacitance? Capacitors Capacitors follow the same law using the reciprocals. The total capacitance of capacitors in parallel is equal to the sum of their individual capacitances:

. The working voltage of a parallel combination of capacitors is always limited by the smallest working voltage of an individual capacitor. 24. Calculate the amount of energy stored in capacitor. Energy storage Work must be done by an external influence to "move" charge between the conductors in a capacitor. When the external influence is removed the charge separation persists in the electric field and energy is stored to be released when the charge is allowed to return to its equilibrium position. The work done in establishing the electric field, and hence the amount of energy stored, is given by:[11]
114 | P a g e

25. Calculate RC TIME CONSTANT? The time required charging a capacitor to 63 percent (actually 63.2 percent) of full charge or to discharge it to 37 percent (actually 36.8 percent) of its initial voltage is known as the TIME CONSTANT (TC) of the circuit. The charge and discharge curves of a capacitor are shown in figure.

26. What happens to the voltage across the resistance in an LR circuit during current buildup in the circuit, and during current decay in the circuit? L/R Time Constant The L/R TIME CONSTANT is a valuable tool for use in determining the time required for current in an inductor to reach a specific value. As shown in figure 2-11, one L/R time constant is the time required for the current in an inductor to increase to 63 percent (actually 63.2 percent) of the maximum current. Each time constant is equal to the time required for the current to increase by 63.2 percent of the difference in value between the current flowing in the inductor and the maximum current. Maximum current flows in the inductor after five L/R time constants are completed. The following example should clear up any confusion about time constants. Assume
115 | P a g e

that maximum current in an LR circuit is 10 amperes. As you know, when the circuit is energized, it takes time for the current to go from zero to 10 amperes. When the first time constant is completed, the current in the circuit is equal to 63.2% of 10 amperes. Thus the amplitude of current at the end of 1 time constant is 6.32 amperes. Figure- L/R time constant.

27. What is complex impedance in RL circuit? The fundamental passive linear circuit elements are the resistor (R), capacitor (C) and inductor (L). These circuit elements can be combined to form an electrical circuit in four distinct ways: the RC circuit, the RL circuit, the LC circuit and the RLC circuit with the abbreviations indicating which components are used. These circuits exhibit important types of behavior that are fundamental to analogue electronics. In particular, they are able to act as passive filters. This article considers the RL circuit in both series and parallel as shown in the diagrams. In practice, however, capacitors (and RC circuits) are usually preferred to inductors since they can be more easily manufactured and are generally physically smaller, particularly for higher values of components. The complex impedance ZL (in ohms) of an inductor with inductance L (in henries) is

The complex frequency s is a complex number,

Where

j represents the imaginary unit: j2 = 1

is the exponential decay constant (in radians per second), and is the angular frequency (in radians per second).

116 | P a g e

28. Explain RL series circuit.

By viewing the circuit as a voltage divider, we see that the voltage across the inductor is:

and the voltage across the resistor is: . The current in the circuit is the same everywhere since the circuit is series:

. Transfer functions The transfer function for the inductor is

Similarly, the transfer function for the resistor is

29. What is RLC circuit? Find the power in parallel RLC circuit. An RLC circuit (or LCR circuit) is an electrical circuit consisting of resistor, an inductor, and a capacitor, connected in series or in parallel. The RLC part of the name is due to those letters being the usual electrical symbols for resistance, inductance and capacitance respectively. The circuit forms a harmonic oscillator for current and will resonate in just the same way as an LC circuit will. The difference that the presence of the resistor makes is that any oscillation induced in the circuit will die away over time if it not kept going by a source. This effect of the resistor is called damping. Some resistance is unavoidable in real circuits, even if a resistor is not specifically included as a component. A pure LC circuit is an ideal which really only exists in theory. When dealing with a parallel ac circuit, you will find that the concepts presented in this chapter for series ac circuits still apply. There is one major difference between a series circuit and a parallel circuit that must be considered. The difference is that current is the same in all parts of a series circuit, whereas voltage is the same across all branches of a parallel circuit. Because of this difference, the total impedance of a parallel circuit must be computed on the basis of the current in the circuit.
117 | P a g e

You should remember that in the series RLC circuit the following three formulas were used to find reactance, impedance, and power factor:

When working with a parallel circuit you must use the following formulas instead:

NOTE: If no value for E is given in a circuit, any value of E can be assumed to find the values of I L, IC, IX, IR, and IZ. The same value of voltage is then used to find impedance. 30. Write are the basic terms used in RL,RC,and RLC circuit. INDUCTANCE IN AC CIRCUITS - An inductor in an ac circuit opposes any change in current flow just as it does in a dc circuit. PHASE RELATIONSHIPS OF AN INDUCTOR - The current lags the voltage by 90 in an inductor (ELI). INDUCTIVE REACTANCE - The opposition an inductor offers to ac is called inductive reactance. It will increase if there is an increase in frequency or an increase in inductance. The symbol is X L, and the formula is XL = 2pfL. CAPACITANCE IN AC CIRCUITS - A capacitor in an ac circuit opposes any change in voltage just as it does in a dc circuit. PHASE RELATIONSHIPS OF A CAPACITOR - The current leads the voltage by 90 in a capacitor (ICE). CAPACITIVE REACTANCE - The opposition a capacitor offers to ac is called capacitive reactance. Capacitive reactance will decrease if there is an increase in frequency or an increase in capacitance TOTAL REACTANCE - The total reactance of a series ac circuit is determined by the formula X = X L XC or X = XC - X L. The total reactance in a series circuit is either capacitive or inductive depending upon the largest value of XC and XL. In a parallel circuit the reactance is determined by <figureeq2"> where IX = IC - IL or IX = IL - IC. The reactance in a parallel circuit is either capacitive or inductive depending upon the largest value of IL and IC.

118 | P a g e

PHASE ANGLE - The number of degrees that current leads or lags voltage in an ac circuit is called the phase angle. The symbol is q. OHM'S LAW FORMULAS FOR AC - The formulas derived for Ohm's law used in ac are: E = IZ and I = E/Z. TRUE POWER - The power dissipated across the resistance in an ac circuit is called true power. It is measured in watts and the formula is: True Power = (IR)2 R. REACTIVE POWER - The power returned to the source by the reactive elements of the circuit is called reactive power. It is measured in volt-amperes reactive (var). The formula is: Reactive Power = (IX)2 X. APPARENT POWER - The power that appears to the source because of circuit impedance is called apparent power. It is the combination of true power and reactive power and is measured in voltamperes (VA). The formulas are:

POWER FACTOR - The portion of the apparent power dissipated in a circuit is called the power factor of the circuit. It can be expressed as a decimal or a percentage. The formulas for power

119 | P a g e

POWER FACTOR CORRECTION - To reduce losses in a circuit the power factor should be as close to unity or 100% as possible. This is done by adding capacitive reactance to a circuit when the total reactance is inductive. If the total reactance is capacitive, inductive reactance is added in the circuit.

120 | P a g e

UNIT-4
PART-A 1. What is electrical measuring instrument? The instrument which are used to measure the various electrical quantities are called electrical measuring instruments. 2. What is unit? In order to measure magnitude of electrical quantities, a fixed quantity is taken as unit. 3. Write the classification of electrical instruments. All measuring instruments are classified into two categories. a. Absolute Instrument An instrument whose calibration can be determined by means of physical measurements on the instrument. b. Secondary instrument An instrument whose calibration is determined by comparison with an absolute instrument. 4. Write the type of secondary instruments. (i) Indicating Instruments: Instruments that indicate the instantaneous value of the Electrical quantity being measured at the time at which it is being measured. Their indications are given by points moving over calibrated chief. Example: Ammeters, Voltmeters & Watt meters (ii) Recording Instruments: Instruments which give a continuous record Of the variation of an electrical quantity over a selected period of time. The pointer in these types of instruments is an in fed pen which leaves a trace on a paper put over a moving drum. (iii) Integrating Instruments: Instruments which measure the total Quantity of Electricity delivered in a particular time. Example: Ampere-hour, Watt-hour etc 5. What is the mode of classification of electrical instruments? ANOTHER MODE OF CLASSIFICATION The Electrical instruments can also be classified as 1. Quantity being measured 2. According to the kind of current 3. According to the Accuracy limits 4. According to the Principle of operations 5. According to the Types of indication 6. According to the Application. 6. In which instrument magnetic effect is present? This effect is present in volt-meters, ammeters,
121 | P a g e

Watt-meters, power-factor meter etc. 7. In which thermal effect is present? This effect is present in volt-meters, ammeters, Watt-meters etc. 8. In which instrument chemical effect is present? This effect is present in D.C ampere hour meters (Integrating meters) 9. In which instrument chemical effect is present? This effect is present in volt-meters which can Indirectly be used as ammeters & watt-meters. 10. In which instrument Electromagnetic induction effect is present? This effect is present in volt-meters,Ammeters, watt-meters & integrating meters used in A.C only. PART-B 11. What is the working principle of electrical instruments? The working of the entire electrical instrument is based on utilizing one or more of the various effects of current or voltage. Most of the electrical quantities eg:-current, voltage and power etc. cannot be observed physically. These quantities are converted into a mechanical force and then the force is measured. This mechanical force will move the pointer over the calliberated scale, indicating the value of electrical quantities to be measured. This conversion from electrical quyantity under measurement into mechanical force is done by using the following effects: 1. Magnetic effect. 2. Thermal effect. 3. Electrostatics effect. 4. Electromagnetic induction effect. 5. Electrodynamics effect. 6. Chemical effect. 12. What is deflecting force? The deflecting force is required for moving the pointer from its zero position. The system Producing the deflecting force is called deflecting system or moving system. 13. What is controlling force? The controlling force is required in an indicating instrument to ensure the definite position of pointer corresponding to every value of the quantity being measured. This force opposes the deflecting force and is produced by this system is called controlling torque (T c) or restoring torque.

122 | P a g e

14. Compare between the spring control and gravity control. Spring control Tc Leveling of instrument is not required. Tc Leveling instrument is required. Gravity control

Practically no increase of weight of the Increase the weight of the moving system. system Scale is uniform It is costlier. Non-Uniform scale. It is cheaper.

Spring action changes with fatigue and Not affected by temperature or fatigue. temperature.Hence controlling torque also changes. This will lead to error in readings. 15. What is damping force? A damping device. The damping force ensures that the Pointer comes to rest in its final position quickly and without undue oscillation. There are three main types of damping used eddy-current damping, air-friction damping and fluid-friction damping. 16. Write the type of scale. There are basically two types of Scale linear and non-linear. A linear scale where the divisions or graduations are evenly spaced. The voltmeter shown has a range 0100 V, i.e. a full-scale deflection (f.s.d.) of 100 V. A non-linear scale the scale is cramped at the beginning and the graduations are uneven throughout the range. Theammeter shown has an f.s.d. of 10. 17. What is moving iron instrument? An attraction type of moving-iron instrument is when current flows in the solenoid, a pivoted softiron disc are attracted towards the solenoid and the movement causes a pointer to move across a scale. In the repulsion type moving-iron instrument, two pieces of iron are placed inside the solenoid, one being fixed, and the other attached to the spindle carrying the pointer. 18. What is deflection torque? Controlling Torque or Restoring torque: The deflection of the moving system is indefinite if there is no controlling torque. This torque opposes the deflecting torque and increases with the deflection of the moving System. The pointer is brought to rest when these two torques are equal. Controlling torque is obtained in indicating instrument by TWO ways 1. Spring Control. 2. Gravity control. 19. What is spring control method? A hair-spring, usually Phosphor bronze, is used to provide the controlling torque. With the deflection of the pointer, the spring is twisted in the opposite direction
123 | P a g e

The twist in the spring produces the restoration torque which is directly proportional to the angle of deflection. Since the deflection is proportional to current, the scale on the spring controlled meter is uniform or equally-spaced scale over the whole of their range. 20. What is gravity control method? Gravity control is obtained by attaching small adjustable weights to some part of the moving system such that the two exert torques in the opposite directions. In Gravity control, controlling torque is proportional to the site of the angle of deflection. The degree of control is adjusted by screwing the weight up or down the carrying system. At equilibrium point, the current is proportional to sine of the angle of deflection Gravity controlled instruments have scales cramped or crowded at their lower region. PART-C

21. What is loading effect? Some measuring instruments depend for their operation on power taken from the circuit in which measurements are being made. Depending on the loading effect of the instrument (i.e. the current taken to enable it to operate), the prevailing circuit conditions may change. The resistance of voltmeters may be calculated since each have a stated sensitivity (or figure of merit), often stated in k_ per volt off.s.d. A voltmeter should have as high a resistance as possible (_ ideally infinite). In a.c. circuits the impedance of the instrument varies with frequency and thus the loading effect of the instrument can change. 22. Write the operating principle of PMMC. Permanent Magnet Moving Coil: Principle of Working When a current carrying conductor is placed in a magnetic field, it experiences a force and tends to move in the direction as per Flemings left hand rule. If the first and the second finger and the thumb of the left hand are held so that they are at right angle to each other, then the thumb shows the direction of the force on the conductor, the first finger points towards the direction of the magnetic field and the second finger shows the direction of the current in the wire. Construction: A coil of thin wire is mounted on an aluminum frame (spindle) positioned between the poles of a U shaped permanent magnet which is made up of magnetic alloys like alnico. The coil is pivoted on the jeweled bearing and thus the coil is free to rotate. The current is fed to the coil through spiral springs which are two in numbers. The coil which carries a current, which is to be measured, moves in a strong magnetic field produced by a permanent magnet and a pointer is attached to the spindle which shows the measured value. When a current flow through the coil, it generates a magnetic field which is proportional to the current in case of an ammeter. The deflecting torque is produced by the electromagnetic action of the current in the coil and the magnetic field.
124 | P a g e

he controlling torque is provided by two phosphorous bronze flat coiled helical springs. These springs serve as a flexible connection to the coil conductors. Damping is caused by the eddy current set up in the aluminum coil which prevents the oscillation of the coil. 23. Write the application of PMMC. The PMMC has a variety of uses onboard ship. It can be used as: Ammeter: When PMMC is used as an ammeter, except for a very small current range, the moving coil is connected across a suitable low resistance shunt, so that only small part of the main current flows through the coil. The shunt consists of a number of thin plates made up of alloy metal, which is usually magnetic and has a low temperature coefficient of resistance, fixed between two massive blocks of copper. A resistor of same alloy is also placed in series with the coil to reduce errors due to temperature variation. Voltmeter: When PMMC is used as a voltmeter, the coil is connected in series with high resistance. Rest of the function is same as above. The same moving coil can be used as an ammeter or voltmeter with an interchange of above arrangement Galvanometer: Galvanometer is used to measure small value of current along with its direction and strength. It is mainly used onboard to detect and compare different circuits in a system. Ohmmeter: The ohm meter is used to measure resistance of the electric circuit by applying a voltage to a resistance with the help of battery. A galvanometer is used to determine the flow of current through the resistance. The galvanometer scale is marked in ohms and as the resistance varies, since the voltage is fixed, the current through the meter will also vary. 24. Write the advantages and disadvantages of PMMC. Advantages: The PMMC consumes less power and has great accuracy. It has uniformly divided scale and can cover arc of 270 degree. The PMMC has a high torque to weight ratio. It can be modified as ammeter or voltmeter with suitable resistance. It has efficient damping characteristics and is not affected by stray magnetic field. It produces no losses due to hysteresis. Disadvantage: The moving coil instrument can only be used on D.C supply as the reversal of current produces reversal of torque on the coil.
125 | P a g e

1)

Its very delicate and sometimes uses ac circuit with a rectifier. Its costly as compared to moving coil iron instruments. It may show error due to loss of magnetism of permanent magnet. 25. What is two watt meter method? The Two-Wattmeter Method In a three phase, wye or delta three wire systems, under balanced or unbalanced conditions, with any power factor, the two-wattmeter method is a practical and commonly used method of measuring total three phase power. A simplified circuit diagram of the two-wattmeter connections. Circuit is simplified in the sense that the ammeters, current transformers, selection switch, and polarity switch have been omitted. The polarities of the voltage and current connections to the wattmeters are significant. Note that the line, in which the current is not measured, line c, is connected to the negative voltage terminal on both wattmeters. The total power delivered to the load is given by Eq.1. P total = P 1 + P 2 26. What is the difference between magnetic circuit and Electric circuit? Magnetic circuits 1. Strictly speaking, magnetic flux does not flow. 2. In magnetic circuit, energy is required only for creating the flux intensity but not for maintaining it. 3.There is no magnetic insulator to oppose the flux 4. Flux can pass through air. 5. Residual flux persists after removal of MMF. Electric circuits 1. Electric current actually flows in electric circuit. Electric circuit requires continuous expenditure of energy when current flows. There are a number of electric insulators. Current cannot flow through air. At most an arch is struck Current is reduce to Zero after removal of source(EMF)

27. What are energy meters? Write the types of energy meter. Energy meters integrating instruments and are used to measure the quantity of electric energy supplied to a circuit in given time. They give o direct indication of power that is being consumed I a circuit at that time. Types of energy meter1. Electrolitic meter-An instrument for measuring the throwing power, electrochemical efficiency and operating current density of an electrolyte comprises a number of removable electrodes (20) arranged inside a nonconductive probe body (10) with an open end (11) and an optional solution circulation opening at its opposite end (30) with the electrodes connected via current meters (80) to a current controller (70).

126 | P a g e

2. Motor meter-An integrating meter which has a rotor, one or more stators, a retarding element which makes the speed of the rotor proportional to the quantity (such as power or current) whose integral over time is being measured, and a register which counts the total number of revolutions of the rotor. 28. What is moving iron type instruments? Measuring instruments are classified according to both the quantity measured by the instrument and the principle of operation. Three general principles of operation are available: (i) electromagnetic, which utilizes the magnetic effects of electric currents; (ii) electrostatic, which utilizes the forces between electrically-charged conductors; (iii) electro-thermic, which utilizes the heating effect. Electric measuring instruments and meters are used to indicate directly the value of current, voltage, power or energy. In this lesson, we will consider an electromechanical meter (input is as an electrical signal results mechanical force or torque as an output) that can be connected with additional suitable components in order to act as an ammeters and a voltmeter. The most common analogue instrument or meter is the permanent magnet moving coil instrument and it is used for measuring a dc current or voltage of a electric circuit. On the other hand, the indications of alternating current ammeters and voltmeters must represent the RMS values of the current, or voltage, respectively, applied to the instrument. 29. A PMMC meter with a coil resistance 100 and a full scale deflection current of 100 A is to be used in the voltmeter circuit as shown in Fig.42.6. The voltmeter ranges are to be 50. Determine the required value of resistances for each range. , 100, 150. 1. V=50 v, I=100 2. V=100 v, I=100 =0.9999 . 3. V=150 v, I=100 10010-6A, and Rm=100 , the value of series resistance (Rs1) =0.4999 . 10010-6A, and Rm=100 , the required value of series resistance (Rs2) 10010-6A, and Rm=100 , the value of series resistance (Rs3) =1.4999 .

30. What is ammeter and voltmeter sensivity? Ammeter Sensitivity: Ammeter sensitivity is determined by the amount of current required by the meter coil to Produce full-scale deflection of the pointer. The smaller the amount of current required Producing this deflection, the greater the sensitivity of the meter. A meter movement that Requires only 100 microamperes for full- scale deflection has a greater sensitivity than a Meter movement that requires 1 mA for the same deflection. Voltmeter Sensitivity: The sensitivity of a voltmeter is given in ohms per volt. It is determined by dividing the Sum of the resistance of the meter (Rm), plus the series resistance (Rs), by the full-scale Reading in volts. In equation form, sensitivity is expressed as follows: Sensivity= Rm+ Rs/E

This is the same as saying the sensitivity is equal to the reciprocal of the full-scale Deflection current. In equation form, this is expressed as follows:
127 | P a g e

Sensivity = ohm/volt=1/volt/ohm=1/ampere Therefore, the sensitivity of a 100-microampere movement is the reciprocal of 0.0001 Ampere or 10,000 ohms per volt. Sensivity = 1/ampere=10,000ohms per volt

128 | P a g e

UNIT-5 PART-A 1. Define transformer. A transformer is a static device which transfers electric Power from one circuit to another without changing its Frequency. 2. Write the classification of transformer. Classification of transformer according to input: a. Step-up: A transformer which receives the energy at a lower voltage and delivers at higher voltage is known as step-up transformer. b. Step-down: A transformer which receives the energy at a higher voltage and delivers at lower voltage is known as step-down transformer. 3. Write the classification as per service and use of transformer. Based on the type of service and use, the transformers are classified as: 1. 2. 3. 4. Distribution transformer. Power transformer. Instrument transformer for measuring purpose. Instrument transformer for protection purpose.

4. Write the constructional features of a transformer. The transformer has a simple structure, and there are no rotating parts or bearings. The chief elements of construction are: a. Magnetic circuit comprising limbs, yoke and clamping structure. b. Electric circuit comprising primary and secondary windings, formers, insulation and bracing devices. c. Terminals comprising tappings and tapping switches, terminal bushings and leads. d. Tank comprising of oil and /or cooling apparatus, conservator, breather and ancillary apparatus such as oil gauge, temperature gauge, valves etc. 5. What are the advantages of air Cooled oil-immersed Transformers? The advantages of air Cooled oil-immersed Transformers: 1. Dust does not clog the cooling ducts. 2. Winding and insulation is free from the affect of moisture. 3. The design for higher voltages is greatly improved. 6. What is ideal transformer? Ideal transformer is nothing but a hypothetical transformer. It is supposed to consist of two purely inductive coils wound on a loss-free core. 7. What are the assumptions made in ideal transformer? The assumption for an ideal transformer is:
129 | P a g e

1. 2. 3. 4.

The primary and secondary windings have no winding resistance. There is no leakage flux. There are no losses in the transformer. There is no magnetizing current.

8. What is the difference between the ideal transformer and practical transformer? IDEAL TRANSFORMER 1. It consists of purely inductive coils wound on lossless core. 2. There are no losses. 3. There is no leakage flux. 4. Efficiency is 100%. 5. %voltage regulation is 0% 6. Construction is impossible. PRACTICAL TRANSFORMER 1. Windings have small ohmic resistance. 2. Iron and copper losses are present. 3. Leakage flux is present. 4. Efficiency is above 95%. 5. Definite voltage regulation. 6. All transformers constructed are practical transformer.

9. Write the emf equation of transformer. The emf equation of transformer is e = -N d/dt Where, e is the instantaneous value of emf induced in volt. is the instantaneous value of mutual flux. N is the number of turns in the coil. 10. What are the tests on the transformer? The various methods employed for testing of Transformers are: 1. 2. 3. Direct method. Indirect method. Regenerative method.

PART-B
11. What is all day efficiency? All-day efficiency is defined as the ratio of total energy Output to that of the total energy input over a given Period of time which is generally taken to be 24 hr (1 Day). 12. What is commercial efficiency? It is defined as the ratio of total energy Output in watt to that of the total energy input in watt over a given period of time.

130 | P a g e

13. Explain the principle and operation of a single phase transformer? The basic working principle of transformer Transformer refers to the static electromagnetic setting which can transfer power from one circuit to another one. In AC circuits, AC voltage, current and waveform can be transformed with the help of Transformers. Each transformation is usually to transfer from one circuit to another one by the way of electromagnetism, but it has no direct relation with this circuit. It also can be transformed through electromagnetism (electrical manner). This electromagnetism is known as autotransformer. Transformer plays an important role in electronic equipment. AC and DC voltage in Power supply equipment are almost achieved by transformers transformation and commutation. At the same time the electrical parameters transformed by transformer are not one but a few ones. Most of the isolation, matching and impedance in the circuit carry out by transformer. Most of isolation, matching and impedance in the circuit carry out by transformer. Simple schematic diagram of the transformer is shown in (1-1). It is connected by closed-magnet (iron cores), two windings and AC power supply. The winding is called the primary winding; another winding is connected with load, and it is called secondary windings.

Figure 1-1 structural schematic diagram of transformer No-load state of Transformer: viz. the disconnecting state between he secondary winding and load (Figure 1-2). Connect the primary winding and the power supply of AC voltageU1, and then it will produce alternating current I0, this current is called no-load currents. This current set up alternating magnetic flow0 which is closed along iron core magnetic circuit. At the same time, it traverses the primary winding and secondary winding, and then produces inducting electromotive forceE2 (secondary no-load voltage). 14. What is the effect of load in a transformer? EFFECT OF A LOAD When a load device is connected across the secondary winding of a transformer, current flows through the secondary and the load. The magnetic field produced by the current in the secondary interacts with the magnetic field produced by the current in the primary. This interaction results from the mutual inductance between the primary and secondary windings. 15. Define mutual flux. The total flux in the core of the transformer is common to both the primary and secondary windings. It is also the means by which energy is transferred from the primary winding to the
131 | P a g e

secondary winding. Since this flux links both windings, it is called MUTUAL FLUX. The inductance which produces this flux is also common to both windings and is called mutual inductance. 16. What is the theoretical comparison between motional emf and transformer emf? Theoretical comparison between electromagnetic dampers based on a motional emf and transformer emf design is presented. Transformer emf devices are based on the generation of emf in a stationary circuit, in which the emf is generated by a time-varying magnetic field linking the circuit. Motional emf devices are based on the generation of emf due to a moving conductor within a stationary magnetic field. Both of these designs can be used as damping elements for applications such as semi-active and regenerative vehicle suspension systems. The findings herein are provided so as to evaluate the most efficient device for such applications. The analysis consists of comparing the damping coefficient of the electromagnetic devices for a given magnetic field and given volume of conducting material. It has been found that for a limited range of dimensions, the transformer emf devices can be more then 12 times as efficient as the motional emf devices. However, for most realistic situations, motional emf devices will have the highest efficiency. 17. Describe the power losses in a transformer. When transformers transfer power, they do so with a minimum of loss. As it was stated earlier, modern power transformer designs typically exceed 95% efficiency. It is good to know where some of this lost power goes, however, and what causes it to be lost. There is, of course, power lost due to resistance of the wire windings. Unless superconducting wires are used, there will always be power dissipated in the form of heat through the resistance of current-carrying conductors. Because transformers require such long lengths of wire, this loss can be a significant factor. Increasing the gauge of the winding wire is one way to minimize this loss, but only with substantial increases in cost, size, and weight. Resistive losses aside, the bulk of transformer power loss is due to magnetic effects in the core. Perhaps the most significant of these "core losses" is eddy-current loss, which is resistive power dissipation due to the passage of induced currents through the iron of the core. Because iron is a conductor of electricity as well as being an excellent "conductor" of magnetic flux, there will be currents induced in the iron just as there are currents induced in the secondary windings from the alternating magnetic field. These induced currents -- as described by the perpendicularity clause of Faraday's Law -- tend to circulate through the cross-section of the core perpendicularly to the primary winding turns. Their circular motion gives them their unusual name: like eddies in a stream of water that circulates rather than move in straight lines. 18. Difference between core type and shell type transformer. The basic difference between these two transformers is: 1. The core type has two limbs & shell type has three limbs. 2. Core type has longer mean length of iron core & shorter mean length of coil turn. Shell type has shorter mean length of iron core & longer mean length of coil turn. 3. In core type transformers the LV (low voltage) coil is wound next to the core & HV (high voltage) coil is wound on the LV coil after the insulation layer. In Shell type transformers the LV & HV windings are sandwiched between each other.
132 | P a g e

19. What is hysteresis loss and eddy current loss in a transformer? Each time the direction of magnetization is reversed, some useful energy is wasted in overcoming internal friction. This is known as "hysteresis loss" and it also produces heating in the core. Due to variation in magnetic flux eddy currents are induced on the surface of iron core which in turn produce heating and therefore reduce the amount of power to the secondary coil. 20. What is the efficiency of transformer? Efficiency of a device is equal to the ratio of output to input. Since, Output = Ps or Output = Vs Is and Input = Pp or Input = Vp Ip Efficiency = output / input Efficiency = Ps / Pp Efficiency = Vs Is / Vp Ip Efficiency in percent = (Vs Is / Vp Ip) x 100 In actual practice output is not equal to input therefore actual transformers are not 100% efficient. However commercial transformers have very high efficiency in the range of 95% to 99%. PART - C 21. State the principle of working of a transformer? Transformer consists of two inductive coils which are electrically separated but magnetically linked through a path of low reluctance. The two coils possess high mutual inductance. If one coil connected to a source of alternating voltage, an alternating flux is set up in the laminated core, most of which linked with the other coil in which it produces mutually induced emf. If the second coil is closed current flows in it and so electrical energy is transferred from the first coil to the second coil. 22. List the types of transformers based on construction: 1. Core type transformer 2.Shell type transformer 23. State why the transformer core is made of magnetic materials? To provide a continuous magnetic path with the minimum air gap. 24. Define transformation ratio. Transformation ratio is defined as the ratio of number of turns in the secondary
133 | P a g e

Winding to number of turns in primary winding. K= N2/N1 = E2/E1 Types : Step up transformer Step down transformer

25. Define voltage regulation of a transformer. The change is secondary terminal voltage from no load condition to full load condition is known as voltage regulation 26. State the methods of reducing leakage flux: To minimize the leakage flux, transformer core is sectionalized and by Interleaving the primary and secondary windings. 27. List the various losses in a transformer and state the condition for maximum efficiency: Core loss Copper loss Condition for maximum efficiency: Iron loss = copper loss 28. Define all-day efficiency of a transformer? All day efficiency is defined as the ratio of output in Kwh to input in Kwh calculated for 24 hour 29. State the various types of a transformers depending on its voltage transformation ratio: Step-up transformer Step- down transformer 30. Write the emf equation of a transformer and explain the terms used: E=4.44m f N (volts) 31. Why Core loss of a transformer practically constant? Since the core flux is practically constant for all loads the core loss is Practically same for all loads. 32. What is the main difference between core type and shell type Transformers? Core type: The coils wound are form wounded are cylindrical type. Wound in helical layers insulated from each other by paper, cloth, mica. Used forlow voltage transformers. Windings surround a considerable part of thecore. Shell type: coils are form wound but are multi-layer disc type, insulated from each other by paper. Transformer core consists of laminations arranged in groups which radiate out from the centre. Used for high voltage transformers. Core surrounds a considerable portion of the winding.

134 | P a g e

33. Why the rating of the transformers is in KVA? Copper loss of transformer depends on current and iron loss on voltage. Total Loss depends on volt-ampere and not on phase angle. 34. In what way the iron losses can be minimized? Laminating the core, laminations being insulated from each other by varnish 35. Derive the EMF equation of a Transformer? N1=No of turns in Primary N2=No of turns in secondary m=Max flux in the core =Bm *A Average rate of change of flux=m/1/4f=4fmwb/s Average emf/turn=4fmvolt Form factor=R.M.S value/average value=1.11 Rms value of emf/turn=1.11*4fm=4.44fmv Rms value of the induced emf in the whole of primary winding=4.44fN1m=4.44fN1BmA E2=4.44fN2m=4.44fN2BmA E1/N1=E2/N2=4.44fm

135 | P a g e

QUESTION BANK FOR


BASIC ELECTRONICS
(01BT203)

136 | P a g e

BASIC ELECTRONICS
CREDIT - 3 CODE 01BC202

UNIT I
Part A Electronics Engineering Materials and Semiconductor Devices (EEMSD): Q. 1 Write definition of valence band and conduction band. Ans . The range of energies possessed by valence electrons is called valence band. The range of energies possessed by conduction electrons is called conduction band. The free electrons that are left in the valence band are occupying the conduction band. Q. 2 What is forbidden energy gap? Ans . The energy gap between the valence band and conduction band is called forbidden energy gap. For insulators it is 6 eV, for semiconductors its value is comparatively low. Ge has energy gap 0.7 eV and silicon has 1.1 eV. For conductors it is zero. Q. 3 What is a semiconductor? Ans . The semi conductors are those substances w hose conductivity lies between insulators and conductors. With rise in temperature conductivity increases . Semi conductors have negative coefficient of resistance. Q. 4 Explain different types of semiconductors? Ans . Intrinsic semiconductors and extrinsic semiconductors. Intrinsic semi conductors are pure form of semiconductors and extrinsic semiconductors are impure which are of two types P type and N type . When tri valent impurities are added with pure substance the semiconductor so formed is called p type . When penta valent impurities are added with pure semiconductors then semiconductor so formed is called N type semiconductors. E.g.; Boron, Aluminums, Gallium, Indium ( BAGI) added for p type. Arsenic , Antimony , Phosphorus (AAP) is added for n type. Q. 5 What is doping? Ans. The process of adding impurity to pure semiconductor is known as doping .As a result of this characteristics of semiconductors changes hence conductivity increases.; Q. 6 What is Fermi level? Ans. Fermi level is defined as the maximum energy level, which is occupied electron at absolute zero temperature. In p type semiconductor the Fermi level will be above the valence band . In N type it lies below the bottom of the conduction band. Q. 7 State law of mass action. Ans. It states that in a semiconductor the product of the number of holes and the number of electrons is constant and is independent of the amount of the onor and acceptor impurity doped. np =ni2 N =free electron concentration and p = hole concentration n i= intrinsic concentration.

137 | P a g e

Q. 8 Define drift current? Ans. If a steady electric field is applied across a semi conductor , it causes the free electron to move towards the positive terminal and the holes move towards the negative terminal of the battery. This combine d effect causes a current flow in the semiconductor. The current so produced in this manner is called drift current. Drift current density due to electron is Jn = q n n E Drift current density due to holes is Jp = q p p E Q. 9 What is diffusion current? Ans. In a semiconductor it is possible to have a non uniform distribution of carriers. A concentration gradient exists if the number of both holes and electrons is greater in one region compared to the rest of the region. The holes and eletrons then tend to move from a region of higher concentration to lower concentration region. This process is known as diffusion and electric current produced due to this process is called diffusion current. Q. 10 What is P N junction diode? Ans. A pn junction is formed by joining P-type and N-type semiconductors together in very close contact. The term junction refers to the boundary interface where the two regions of the semiconductor meet. If they were constructed of two separate pieces this would introduce a grain boundary so pn junctions are created in a single crystal of semiconductor by doping, for example by ion implantation, diffusion of dopants, or by epitaxial (growing a layer of crystal doped with one type of dopant on top of a layer of crystal doped with another type of dopant). Q. 11 What is depletion region? Ans. In semiconductor physics the depletion region, also called depletion layer, depletion zone, junction region or the space charge region, is an insulating region within a conductive, doped semiconductor material where the mobile charge carriers have diffused away, or have been forced away by an electric field. The only elements left in the depletion region are ionized donor or acceptor impurities. The 'depletion region' is so named because it is formed from a conducting region by removal of all free charge carriers, leaving none to carry a current. Understanding the depletion region is key to explaining modern semiconductor electronics diodes bipolar junction transistors, field-effect transistors, and variable capacitance diodes all rely on depletion region phenomena. Q. 12 Define barrier potential? Ans. It is electric potential that is established across the junction , during the initial diffusion of charge carriers at the junction ,which restricts further movement of charge Carriers across the unction. Q. 13 Explain the term knee voltage and break down voltage. Ans. The forward voltage at which the current through the PN junction increasing rapidly. It is called cut in voltage or threshold voltage. Q. 14 Write down and explain junction diode equation Ans. The equation which explain the forward and reverse characteristics of semiconductor diode / equation I = I0(ev -1) where I0 = reverse saturation current, = 1 for Ge diode and 2 for silicon diode V = external voltage, VT= volt equivalent of temperature = T/11,600; Q. 15 Define PIV? Ans. As a general term applied to semiconductor diodes, peak reverse voltage or peak inverse voltage is the maximum voltage that a diode can withstand in the reverse direction without
138 | P a g e

breaking down or avalanching If this voltage is exceeded the diode may be destroyed. Diodes must have a peak inverse voltage rating that is higher than the maximum voltage that will be applied to them in a given application. Q. 16 Write applications of the diode. Ans. Diode converts AC into DC, Used as charger of mobile, Q. 17 Differentiate Drift and diffusion current. Ans:
Drift Current Developed due to potential gradient Phenomena found both in semiconductor and metals Jn= qn n E Jp= qp p E Diffusion Current Developed due to concentration gradient Phenomena found semiconductor Jn= qDn dn/dx Jn= qDpdp/dx

Q. 18 What is avalanche breakdown in P N Junction diode? Ans . A process that occurs in a diode when high voltage causes free electrons to travel at high speeds, colliding with other electrons and knocking them out of their orbits. The result is a rapidly increasing amount of free electrons. Q. 19 Define the term diffusion capacitance and storage capacitance? Ans. This effect is found when the diode is in the forward bias and it is defined as the rate of change of injected charge with voltage and is given by CD = T I/

Q. 20 List the application of P N junction diode? Ans . (i)Used as rectifier in DC power supplies (ii) Used as signal diodes in communication circuits. I (iii)Used in clipper and clamper circuit

PART B
Q.1 What do you mean by doping and doping level? Ans. Doping is the process of mixing or adding materials to the intrinsic semiconductor. Doping level refers to the quantity of impurity that is being(to be)added to the pure semiconductor material. Q.2 State the properties of semiconductor. Ans. Semiconductor behaves like insulator at absolute zero temperature. There is no fear of hazard due to shock, since working voltage is less that humane body can bear. There is no heating and minimum loss.

139 | P a g e

Q.3 Define drift and diffusion current and derive equation for it. Ans. The drift velocity is the average velocity that a particle, such as an electron, attains due to an electric field. Because current is proportional to drift velocity, which is, in turn, proportional to the magnitude of an external electric field, Ohm's law can be explained in terms of drift velocity. Drift velocity is expressed in the following equations: , where Jdrift is the current density, is charge density in units C/m3, and vavg is the average velocity of the carriers (drift velocity);, where is the electron mobility in (m/s)/(V/m) and E is the electric field in V/m. To find an equation for drift velocity, one can begin with the definition of current: Vd = I/nqA A is the cross sectional area x is a small length along the wire q is the charge of the charge carriers Now, normally particles move randomly, but under the influence of an electric field in the wire, the charge carriers gain an average velocity in a specific direction. This is what's called drift velocity, vd. And since x = vd t, we can plug it into the above equation. Putting that back into the original equation and re-arranging to solve for the drift velocity: Q.4 Explain the operation of a P N junction diode in forward and reverse condition and characteristics? Ans. The pn junction possesses some interesting properties which have useful applications in modern electronics. A p-doped semiconductor is relatively conductive. The same is true of an ndoped semiconductor, but the junction between them can become depleted of charge carriers, and hence nonconductive, depending on the relative voltages of the two semiconductor regions. By manipulating this non-conductive layer, pn junctions are commonly used as diodes: circuit elements that allow a flow of electricity in one direction but not in the other (opposite) direction. This property is explained in terms of forward bias and reverse bias, where the term bias refers to an application of electric voltage to the pn junction. Q.5 Explain the dynamic resistance of a diode using necessary diagrams and derive its relation. Ans. Dynamic resistance of the diode is the resistance for an AC source depending of the DC polarization of the diode.The DC polarization determines the dynamic resistance, a high diode DC current is a minimum dynamic resistance because the variation in the curve of diode is more high then less resistance. If the AC source is approximated 5mVpp the diode presents a dynamic resistance of Rd=25.9mV/I where 25.9mV is a constant and Id y the DC diode current. The AC source should be small to avoid great changes in Id because: Id(Total)=Id(AC)+Id(DC) [superposition principle. Q.6 Explain intrinsic and extrinsic semiconductor in details. Ans. Intrinsic semiconductor is the pure form of semiconductor. Silicon and Germanium are pure form of semiconductor. Extrinsic semiconductors are impure form of semiconductors. This type of semiconductors are formed due to mixing of materials from 3rd and 5th group of periodic table. Boron, Aluminum, gallium, Indium and Thallium are mixed from 3rd group and hence doped material so formed is called p type extrinsic semiconductor. Arsenic, Antimony and Phosphorus etc are doped from 5th group and material so formed called N type extrinsic semiconductor. Q.7 Write short notes on diffusion capacitance and derive the relation? Ans. Diffusion capacitance is the capacitance due to transport of charge carriers between two terminals of a device, for example, the diffusion of carriers from anode to cathode in forward bias mode of a diode or from emitter to base (forward-biased junction in active region) for a transistor. In a semiconductor device with a current flowing through it (for example, an ongoing transport of charge by diffusion) at a particular moment there is necessarily some charge in the process of transit through the device. If the applied voltage changes to a different value and the current changes to a different value, a different amount of charge will be in transit in the new
140 | P a g e

circumstances. The change in the amount of transiting charge divided by the change in the voltage causing it is the diffusion capacitance. The adjective "diffusion" is used because the original use of this term was for junction diodes, where the charge transport was via the diffusion mechanism. See Ficks law. To implement this notion quantitatively, at a particular moment in time let the voltage across the device be V. Now assume that the voltage changes with time slowly enough that at each moment the current is the same as the DC current that would flow at that voltage, say I = I(V) (the quasistatic approximation). Suppose further that the time to cross the device is the forward transit time F. In this case the amount of charge in transit through the device at this particular moment, denoted Q, is given by Q = I(V)F. Q.8 Explain P N junction switching times. Ans. For a frequency synthesizer, the switching time or more colloquially the switching speed is the amount of time from when the command for the next frequency is requested until the time that the synthesizer's output becomes usable and meets the specified requirements. Such requirements will vary depending on the design of the synthesizer. In the 1970s switching speeds ranged from 1 milli-sec to 10 micro-seconds. Q.9 Explain the effect of temperature on P N junction. Ans. Current Io varies with temperature T that it is 8 % per degree C for Silicon and 11% per degree C for Germanium . Reverse saturation current approx. doubles for every 10o rise in temperature . Q.10 Underline the application of zener diode. Ans. Diode (some people incorrectly call them rectifiers) is a semiconductor device that allows a current to flow in one direction. A Zener diode allows a reverse to current to flow at a defined voltage. A common application for them is as a voltage regulator. Named for C M Zener, US Physicist.A zener diode is a diode and like all diodes it will conduct in both directions. If a reversed voltage is applied it will breakdown and conduct current. Most diodes when they breakdown the reversed voltage cannot be predictable. However a zener diode when they do breakdown in the reverse voltage mode that voltage can be made as predictable and remain +/- % of the breakdown voltage. Therefore this steady voltage can be used as a regulator for instance or a definite voltage drop if need be. Applications of Zener Diodes are as follows: a. Voltage Regulators(in shunt mode), b. Surge Suppressors .i.e. for device protection. Q.11 Draw the V- I characteristics of PN Junction diode. Ans. I V

Q.12 Explain half wave rectifier. Ans. Half-wave rectification In half wave rectification, either the positive or negative half of the AC wave is passed, while the other half is blocked. Because only one half of the input waveform reaches the output, it is very inefficient if used for power transfer. Half-wave rectification can be achieved with a single diode in
141 | P a g e

a one-phase supply, or with three diodes in a three-phase supply.The output DC voltage of a half wave rectifier can be calculated with the following two ideal equations.

Q.13 Explain full wave rectifier. Ans. full-wave rectifier converts the whole of the input waveform to one of constant polarity (positive or negative) at its output. Full-wave rectification converts both polarities of the input waveform to DC (direct current), and is more efficient. However, in a circuit with a non-center tapped transformer, four diodes are required instead of the one needed for half-wave rectification. (See semiconductors, diode). Four diodes arranged this way are called a diode bridge or bridge rectifier:Graetz bridge rectifier: a full-wave rectifier using 4 diodes.For single-phase AC, if the transformer is center-tapped, then two diodes back-to-back (i.e. anodes-to-anode or cathode-tocathode) can form a full-wave rectifier. Twice as many windings are required on the transformer secondary to obtain the same output voltage compared to the bridge rectifier above. Full-wave rectifier using a transformer and 2 diodes. Full-wave rectifier, with vacuum tube having two anodes. A very common vacuum tube rectifier configuration contained one cathode and twin anodes inside a single envelope; in this way, the two diodes required only one vacuum tube. The 5U4 and 5Y3 were popular examples of this configuration.A three-phase bridge rectifier3-phase AC input, half & full wave rectified DC output waveforms For three-phase AC, six diodes are used. Typically there are three pairs of diodes, each pair, though, is not the same kind of double diode that would be used for a full wave single-phase rectifier. Instead the pairs are in series (anode to cathode). Typically, commercially available double diodes have four terminals so the user can configure them as singlephase split supply use, for half a bridge, or for three-phase use.Disassembled automobile alternator, showing the six diodes that comprise a full-wave three-phase bridge rectifier.Most devices that generate alternating current (such devices are called alternators) generate three-phase AC. For example, an automobile alternator has six diodes inside it to function as a full-wave rectifier for battery charging applications.The average and root-mean-square output voltages of an ideal single phase full wave rectifier can be calculated asWhere:Vdc,Vav - the average or DC output voltage, Vp - the peak value of half wave, Vrms - the root-mean-square value of output voltage. = ~ 3.14159 Q.14 Explain the working of diode in reverse bias. Ans. Reverse biased usually refers to how a diode is used in a circuit. If a diode is reverse biased, the voltage at the cathode is higher than that at the anode. Therefore, no current will flow until the diode breaks down. Connecting the P-type region to the negative terminal of the battery and the Ntype region to the positive terminal, corresponds to reverse bias. Because the p-type material is now connected to the negative terminal of the power supply, the 'holes' in the P-type material are pulled away from the junction, causing the width of the depletion zone to increase. Similarly, because the N-type region is connected to the positive terminal, the electrons will also be pulled away from the junction. Therefore the depletion region widens, and does so increasingly with increasing reverse-bias voltage. This increases the voltage barrier causing a high resistance to the flow of charge carriers thus allowing minimal electric current to cross the p n junction. The increase in resistance of the p-n junction results in the junction to behave as an insulator. This is important for radiation detection because if current was able to flow, the charged particles would just dissipate into the material. The reverse bias ensures that charged particles are able to make it to the detector system.The strength of the depletion zone electric field increases as the reverse-bias voltage increases. Once the electric field intensity increases beyond a critical level, the p n junction depletion zone breaks-down and current begins to flow, usually by either the Zener or avalanche breakdown processes. Both of these breakdown processes are non-destructive and are reversible, so long as the amount of current flowing does not reach levels that cause the semiconductor
142 | P a g e

material to overheat and cause thermal damage. This effect is used to one's advantage in zener diode regulator circuits. Zener diodes have a certain - low - breakdown voltage.

Q.15 Define static resistance and dynamic resistance. Ans. Static resistance is defined as ratio of V and I . At any point on the V-I characteristics of diode the resistance is equal to the reciprocal of the slope of a line joining the operating to the origin. It varies widely with V and I. Dynamic resistance or increamental resistance r is defined as reciprocal of slope of V I Characteristics dv/dI . It depends on operating voltage.Also it is = V T/I Q.16 What is forward and reverse recovery time? Ans. The most important difference between p-n and Schottky diode is reverse recovery time, when the diode switches from non-conducting to conducting state and vice versa. Where in a p-n diode the reverse recovery time can be in the order of hundreds of nanoseconds and less than 100 ns for fast diodes, Schottky diodes do not have a recovery time, as there is nothing to recover from. The switching time is ~100 ps for the small signal diodes, and up to tens of nanoseconds for special high-capacity power diodes. With p-n junction switching, there is also a reverse recovery current, which in high-power semiconductors brings increased EMI noise. With Schottky diodes switching essentially instantly with only slight capacitive loading, this is much less of a concern.It is often said that the Schottky diode is a "majority carrier" semiconductor device. This means that if the semiconductor body is doped n-type, only the n-type carriers (mobile electrons) play a significant role in normal operation of the device. The majority carriers are quickly injected into the conduction band of the metal contact on the other side of the diode to become free moving electrons. Therefore no slow, random recombination of n- and p- type carriers is involved, so that this diode can cease conduction faster than an ordinary p-n rectifier diode. This property in turn allows a smaller device area, which also makes for a faster transition. This is another reason why Schottky diodes are useful in switch-mode power converters; the high speed of the diode means that the circuit can operate at frequencies in the range 200 kHz to 2 MHz, allowing the use of small inductors and capacitors with greater efficiency than would be possible with other diode types. Small-area Schottky diodes are the heart of RF detectors and mixers, which often operate up to 50 GHz. Q.17 Establish the relation between diode capacitance and reverse bias voltage. Ans. When an external reverse voltage reverses the junction, if far from junction theminority carrriers are equal to their thermal equilibrium values pno and npo are not at same levels. As the minority cariers approach the junction they are rapidly swept across ,and trhe density of minority carriers diminishes to zero at this junction. CD = I/VT Q.18 Define and explain PIV. Ans. As a general term applied to semiconductor diodes, peak reverse voltage or peak inverse voltage is the maximum voltage that a diode can withstand in the reverse direction without breaking down or avalanching If this voltage is exceeded the diode may be destroyed. Diodes must have a peak inverse voltage rating that is higher than the maximum voltage that will be applied to them in a given application. Q.19 Explain the working of zener diode.
143 | P a g e

Ans. A diodes 'IV characteristic' can be approximated by four regions of operation.At very large reverse bias , beyond the peak inverse voltage or PIV, a process called reverse breakdown occurs which causes a large increase in current (i.e. a large number of electrons and holes are created at, and move away from the pn junction) that usually damages the device permanently. The avalanche diode is deliberately designed for use in the avalanche region. In the zener diode, the concept of PIV is not applicable. A zener diode contains a heavily doped p-n junction allowing electrons to tunnel from the valence band of the p-type material to the conduction band of the n-type material, such that the reverse voltage is clamped to a known value (called the zener voltage), and avalanche does not occur. Both devices, however, do have a limit to the maximum current and power in the clamped reverse voltage region. Also, following the end of forward conduction in any diode, there is reverse current for a short time. The device does not attain its full blocking capability until the reverse current ceases. The second region, at reverse biases more positive than the PIV, has only a very small reverse saturation current. In the reverse bias region for a normal P-N rectifier diode, the current through the device is very low (in the A range). However, this is temperature dependent, and at sufficiently high temperatures, a substantial amount of reverse current can be observed (mA or more). The third region is forward but small bias, where only a small forward current is conducted. As the potential difference is increased above an arbitrarily defined cut-in voltage or on-voltage or diode forward voltage drop (Vd), the diode current becomes appreciable (the level of current considered appreciable and the value of cut-in voltage depends on the application), and the diode presents a very low resistance. The currentvoltage curve is exponential. In a normal silicon diode at rated currents, the arbitrary cut-in voltage is defined as 0.6 to 0.7 volts. The value is different for other diode types Schottky diodes can be rated as low as 0.2 V, Germanium diodes 0.25-0.3 V, and red or blue light-emitting diodes (LEDs) can have values of 1.4 V and 4.0 V respectively.At higher currents the forward voltage drop of the diode increases. A drop of 1 V to 1.5 V is typical at full rated current for power diodes.The thermal voltage VT is approximately 25.85 mV at 300 K, a temperature close to room temperature commonly used in device simulation software. At any temperature it is a known constant defined by:where k is the Boltzmann constant, T is the absolute temperature of the p-n junction, and q is the magnitude of charge on an electron (the elementary charge).The Shockley ideal diode equation or the diode law is derived with the assumption that the only processes giving rise to the current in the diode are drift (due to electrical field), diffusion, and thermal recombination-generation. It also assumes that the recombinationgeneration (R-G) current in the depletion region is insignificant. This means that the Shockley equation doesnt account for the processes involved in reverse breakdown and photon -assisted RG. Additionally, it doesnt describe the leveling off of the IV curve at high forward bias due to internal resistance.Under reverse bias voltages (see Figure 5) the exponential in the diode equation is negligible, and the current is a constant (negative) reverse curre nt value of IS. The reverse breakdown region is not modeled by the Shockley diode equation.For even rather small forward bias voltages (see Figure 5) the exponential is very large because the thermal voltage is very small, so the subtracted 1 in the diode equation is negligible and the forward diode current is often approximated asThe use of the diode equation in circuit problems is illustrated in the article on diode modeling.For circuit design, a small-signal model of the diode behavior often proves useful. A specific example of diode modeling is discussed in the article on small-signal circuits.Following the end of forward conduction in a PN type diode, a reverse current flows for a short time. The device does not attain its full blocking capability until the reverse current ceases.The effect can be significant when switching large currents very quickly (di/dt on the order of 100 A/s or more). A certain amount of "reverse recovery time" (tr) (on the order of tens of nanoseconds) may be required to remove the "reverse recovery charge" Qr (on the order of tens of nanoCoulombs) from the diode. During this recovery time, the diode can actually conduct in the reverse direction! That is to say, current will effectively flow from the cathode to the anode! In certain real-world cases it can be important to consider the losses incurred by this non-ideal diode effect.However, when the slew
144 | P a g e

rate of the current is not so severe (di/dt on the order of 10 A/s or less), the effect can be safely ignored. For most applications, the effect is also negligible for Schottky diodes. Q.20 Write the difference between Avalanche and zener breakdown. Ans. Zener diode' and 'avalanche diode' are terms often used interchangeably, with the former much more common. Both refer to breakdown of a diode under reverse bias. Specifically, when a diode is reverse biased, very little current flows, and the diode is to a first order approximation an open circuit. As the reverse voltage is increased, though, a point is reached where there is a sudden increase in current. Equivalently, there is a dramatic reduction in the dynamic resistance (slope of the V-I curve) that can be as low as 1- 2 W in this region. This voltage is called the reverse breakdown voltage and it is fairly independent of the reverse current flowing."Avalanche breakdown is caused by impact ionization of electron-hole pairs.While very little current flows under reverse bias conditions, some current does flow.The electric field in the depletion region of a diode can be very high. Electron/holes that enter the depletion region undergo a tremendous acceleration. As these accelerated carriers collide with the atoms they can knock electrons from their bonds, creating additional electron/hole pairs and thus additional current. As these secondary carriers are swept into the depletion region, they too are accelerated and the process repeats itself.The efficiency of the avalanche effect is characterized by a so-called multiplication factor M that depends on the reverse voltage. Avalanche breakdown occurs in lightly-doped pn-junctions where the depletion region is comparatively long.The doping density controls the breakdown voltage.The temperature coefficient of the avalanche mechanism is positive.That is, as the temperature increases, so does the reverse breakdown voltage.The magnitude of the temperature coefficient also increases with increasing breakdown voltage. For example, the temperature coefficient of a 8.2 V diode is in the range 3 - 6 mV/K while the temperature coefficient of an 18 V diode is in the range of 12 - 18 mV/K."Zener breakdown occurs in heavily doped pn-junctions.The heavy doping makes the depletion layer extremely thin. So thin, in fact carriers cant accelerate enough to cause impact ionization. With the depletion layer so thin, however, quantum mechanical tunneling through the layer occurs causing current to flow.The temperature coefficient of the Zener mechanism is negative the breakdown voltage for a particular diode decreases with increasing temperature. However, the temperature coefficient is essentially independent of the rated breakdown voltage, and on the order of-3 mV/K. "In a 'Zener' diode either or both breakdown mechanisms may be present. At low doping levels and higher voltages the avalanche mechanism dominates while at heavy doping levels and lower voltages the Zener mechanism dominates.At a certain doping level and around 6 V for Si, both mechanism are present with temperature coefficients that just cancel. It is possible to make Zener diodes with quite small temperature coefficients."Neither Zener nor avalanche breakdown are inherently destructive in that the crystal lattice is damaged.However, the heat generated by the large current flowing can cause damage, so either the current must be limited and/or adequate heat sinking must be supplied."

145 | P a g e

UNIT II Transistor Amplifiers Power Electronic Circuits (TAPEC): Part A Q. 1 why base is made thin BJT? Ans. Transistor consists of three portions namely emitter, base and collector. Among them base forms the middle part. It is very thin lightly doped because it allows most of the emitter current carriers towards the collector. Since base is acting as an interface it does not need more area. Q. 2 What do you mean by bias a transistor? Ans. Transistor biasing is the process of maintaining proper flow of zero signal collector current. And collector emitter voltage during the passage of signal. Biasing keeps emitter base junction forward bias junction and collector- base junction reversed biased during the passage of signal. Q. 3 Define different operating regions of a transistor? Ans. Active region: When emitter base junction is forward biased and collector base junction is reverse biased is known as active region. Cut-off region: When both Jc, and JE are reverse biased . Saturation region: The region in which both Jc and JE are forward biased. Q. 4 Define base width modulation (Early effect). Ans. In a common base configuration an increase in collector voltage increases the width of the depletion region. At the output junction diode. The will decrease the effective width of the base. This is known as the early effect. Due to this effect recombination rate reduces at the base region and charge gradient is increased within the base. Q. 5 Explain the significance of the base width modulation(Early Effect)? Ans. i. It reduces the charge recombination of electrons with holes In the base region hence the current gain increases with the increase in collector base voltage . ii. The charge gradient is increased within base, hence current due to minority carriers injected across emitter junction increases. Q. 6 What are the three types of configurations? 1. Common base configuration 2. Common emitter configurations 3. Common collector configurations Q. 7 Which one is most important among above configurations? Ans. The CE configuration is important due to following reasons (i.) High current gain (ii.) Output to input impedance ratio is moderate therefore easy coupling is possible between various transistor stages (iii.) It finds excellent uses in audio frequency applications hence receivers and transmitters. Q. 8 Give the advantages of CE configuration (I.)High output impedance (ii) High current gain (Iii.)High power gain. Q. 9 What is thermal run away?
146 | P a g e

Ans . The reverse saturation current in a semiconductor doubles for every 10 0 rise in temperature. As temperature increases the leakage current increases and the collector current also increases. The increase in collector current produces an increase I power dissipation at the collector base junction . This is in turn further increases the temperature of the collector base junction causing the collector current to further increase. This may become cumulative and it is possible for the transistor to burn out. This is known as thermal run away. Q. 10 How thermal run away can be avoided? Ans. Thermal run away can be avoided using a stabilisaqtion or heat sink with the transistor. Q. 11 What is the use of heat sink? Power transistors are temperature dependent devices. As they handle large currents they can be heated during operation which may lead to self destruction. To avoid this transistor fixed on matel sheet preferabaly aluminum to dissipate heat from the power transistor. This production method is known as heat sink protection. Q. 12 How transited is used as switch? Ans. Transistor should be operated in saturation region and cut off regions to use it as switch, while operating ion saturation region transistor carry heavy current hence considered as ON state in cut off it does not carry current and it is equivalent to open switch. Q. 13 Define and 0f a transistor? Ans. Alpha is the current gain of a common base transistor and is given by = Ice /IE at constant VCB is the current gain of common emitter transistor = IC/IB at constant VCE Q. 14 Which configuration is known as emitter follower and why it is named so? Ans. CC configuration is called emitter follower, whatever may be the signal applied at the input may produce same signal at the output. In other words, gain of the circuit is unity. So that the common collector circuit is named emitter follower. Q. 15 Why do the output characteristics of CB transistor have a slight upward slope? Ans. The emitter and the collector are forward biased under the saturation region. Hence a small change in collector voltage causes a significant change in collector current. Therefore a slight upward slope is found in the output characteristics. Q. 16 Compare the performance of CB CE AND CC transistor. Ans.
Parameters Current gain Ai Voltage gain VI Input resistance Rib Output resistance Rib CB Low HIGH Low HIGH CE HIGH HIGH medium medium CC HIGH Low HIGH Low

Q. 17 What do you understand by h parameter? Ans. Any liner circuit can be analyzed by four parameters of mixed dimensions (hi measured in ohms, while ho is measured in mho, parameters like her and dimensionless). Since the dimension of the parameters has mixed units they are referred as h-parameters. The h- parameters are determined by both open circuit and short circuit terminations.

147 | P a g e

Q. 18 Define h- parameters of a transistor. Ans. The equations are. Vic = hoe I + hark Vice I = I+h V ce huff oe ice The h- parameters of CE transistor are: (A) Input impedance: it is defined as the ratio between input voltage Vibe and input cutter IB, when output voltage vie zero. its unit is ohms. Hoe = vibe/I at vs. e = 0 (b) Reverse voltage gain: it is define as the ratio between input voltages Vibe and output voltage vice, when input current I is zero. Hoe= vibe/ vice at Ib = 0 Q. 19 what is the significance of h parameter? Ans . h parameter can be determined from transistor at audio range of frequenciesas parameters are real numbers at audio range of frequency can be employed easily for circuit analysis and design. Q. 20 define turn on time. Ans . The turn on time is the sum of delay time and rise time

PART B Q. 1 Define turn off time Ans .The turn off time is the sum of storage time and fall time . Q.2 Explain the construction and operations of BJT in unbiased condition and biased condition.? Ans. Bipolar Transistor Basics Bipolar Junction Transistor, or BJT for short. This type of transistor is generally known as a Bipolar Transistor, because its basic construction consists of two PN-junctions with each terminal or connection being given a name to identify it and these are known as the Emitter, Base and Collector respectively. The word Transistor is an acronym, and is a combination of the words Transfer varistor used to describe their mode of operation way back in their early days of development. There are two basic types of bipolar transistor construction, NPN and PNP, which basically describes the physical arrangement of the P-type and N-type semiconductor materials from which they are made. Bipolar Transistors are "CURRENT" Amplifying or current regulating devices that control the amount of current flowing through them in proportion to the amount of biasing current applied to their base terminal. The principle of operation of the two transistor types NPN and PNP, is exactly the same the only difference being in the biasing (base current) and the polarity of the power supply for each type. The construction and circuit symbols for both the NPN and PNP bipolar transistor are shown above with the arrow in the circuit symbol always showing the direction of conventional current flow between the base terminal and its emitter terminal, with the direction of the arrow pointing from the positive P-type region to the negative N-type region, exactly the same as for the standard diode symbol. There are basically three possible ways to connect a Bipolar Transistor within an electronic circuit with each method of connection responding differently to its input signal as the static characteristics of the transistor vary with each circuit arrangement.
148 | P a g e

Common Base Configuration - has Voltage Gain but no Current Gain. Common Emitter Configuration - has both Current and Voltage Gain. Common Collector Configuration - has Current Gain but no Voltage Gain. Q.3 Explain the basic techniques for transistor construction? Ans. Basic techniques for transistor construction are: Grown type: Alloy type Diffusion type Epitaxial type Q.4 Explain the characteristic of a transistor in common base configuration. Also explain the current relation in common base transistor configuration. Ans. The Common Base Configuration. As its name suggests, in the Common Base or Grounded Base configuration, the BASE connection is common to both the input signal AND the output signal with the input signal being applied between the base and the emitter terminals. The corresponding output signal is taken from between the base and the collector terminals as shown with the base terminal grounded or connected to a fixed reference voltage point. The input current flowing into the emitter is quite large as its the sum of both the base current and collector current respectively therefore, the collector current output is less than the emitter current input resulting in a Current Gain for this type of circuit of less than "1", or in other words it "Attenuates" the signal. The Common Base Amplifier Circuit

This type of amplifier configuration is a non-inverting voltage amplifier circuit, in that the signal voltages Vin and Vout are In-Phase. This type of arrangement is not very common due to its unusually high voltage gain characteristics. Its Output characteristics represent that of a forward biased diode while the Input characteristics represent that of an illuminated photo-diode. Also this type of configuration has a high ratio of Output to Input resistance or more importantly "Load" resistance (RL) to "Input" resistance (Rin) giving it a value of "Resistance Gain". Q.5 Explain the characteristics of a transistor in common emitter configuration. Also explain current relation in common emitter configuration.

Ans. It is called the common-emitter configuration because (ignoring the power supply battery) both the signal source and the load share the emitter lead as a common connection point. This is not the only way in which a transistor may be used as an amplifier, as we will see in later sections of this chapter.

149 | P a g e

Common-emitter amplifier: The input and output signals both share a connection to the emitter. Before, a small solar cell current saturated a transistor, illuminating a lamp. Knowing now that transistors are able to throttle their collector currents according to the amount of base current supplied by an input signal source, we should see that the brightness of the lamp in this circuit is controllable by the solar cell's light exposure. When there is just a little light shone on the solar cell, the lamp will glow dimly. The lamp's brightness will steadily increase as more light falls on the solar cell. Suppose that we were interested in using the solar cell as a light intensity instrument. We want to measure the intensity of incident light with the solar cell by using its output current to drive a meter movement. It is possible to directly connect a meter movement to a solar cell for this purpose. In fact, the simplest light-exposure meters for photography work are designed like this.

High intensity light directly drives light meter. Although this approach might work for moderate light intensity measurements, it would not work as well for low light intensity measurements. Because the solar cell has to supply the meter movement's power needs, the system is necessarily limited in its sensitivity. Supposing that our need here is to measure very low-level light intensities, we are pressed to find another solution. Perhaps the most direct solution to this measurement problem is to use a transistor to amplify the solar cell's current so that more meter deflection may be obtained for less incident light.

150 | P a g e

Q.6 Explain the characteristics of a transistor in common collector configuration. Also explain current relation in common collector configuration Ans.

Emitter Follower The common collector junction transistor amplifier is commonly called an emitter follower. The voltage gain of an emitter follower is just a little less than one since the emitter voltage is constrained at the diode drop of about 0.6 volts below the base . Its function is not voltage gain but current or power gain and impedance matching. It's input impedance is much higher than its output impedance so that a signal source does not have to work so hard. This can be seen from the fact that the base current in on the order of 100 times less that the emitter current. The low output impedance of the emitter follower matches a low impedance load and buffers the signal source from that low impedance.

NPN Common Collector Amplifier

151 | P a g e

NPN Common Collector Design

Q.7 Explain detailed study of currents in a transistor and derive ebers moll model? Ans. EbersMoll model EbersMoll Model for an NPN transistor.IB, IC, IE: base, collector and emitter currents ICD, IED: collector and emitter diode currents F, R: forward and reverse common-base current gains EbersMoll Model for a PNP transistor. The DC emitter and collector currents in active mode are well modeled by an approximation to the EbersMoll model: The base internal current is mainly by diffusion (see Fick's law) and where VT is the thermal voltage kT / q (approximately 26 mV at 300 K room temperature). IE is the emitter current IC is the collector current T is the common base forward short circuit current gain (0.98 to 0.998) 15 IES is the reverse saturation current of the baseemitter diode (on the order of 10 to 1012 amperes) VBE is the baseemitter voltage Dn is the diffusion constant for electrons in the p-type base W is the base width The and forward parameters are as described previously. A reverse is sometimes included in the model.The unapproximated EbersMoll equations used to describe the three currents in any operating region are given below. These equations are based on the transport model for a bipolar junction transistor whereiC is the collector current iB is the base current iE is the emitter current F is the forward common emitter current gain (20 to 500) R is the reverse common emitter current gain (0 to 20) IS is the reverse saturation current (on the order of 10 15 to 1012 amperes) VT is the
152 | P a g e

thermal voltage (approximately 26 mV at 300 K room temperature). VBE is the baseemitter voltage VBC is the basecollector voltage Q.8 Explain transistor switching time? Ans. If apulse is apllied to an inverter, the transistor acts as a swich, since it operates from cut off to saturation and then tretuns to cut off.When transistor makes transition between voltage levels at a voltage level it remains in cut off and at otherb voltage level it is at saturation . Time taken to make this transistion is called transistor switching time. Q.9 Explain power MOSFET construction characteristics and operations? Ans. A Power MOSFET is a specific type of metal oxide semiconductor field-effect transistor (MOSFET) designed to handle significant power levels. Compared to the other power semiconductor devices (IGBT, Thyristor..), its main advantages are high commutation speed and good efficiency at low voltages. It shares with the IGBT an isolated gate that makes it easy to drive. It was made possible by the evolution of CMOS technology, developed for manufacturing Integrated circuits in the late 1970s. The power MOSFET shares its operating principle with its lowpower counterpart, the lateral MOSFET. The power MOSFET is the most widely used low-voltage (i.e. less than 200 V) switch. It can be found in most power supplies, DC to DC converters, and low voltage motor controllers. Power MOSFETs have a different structure than the lateral MOSFET: as with all power devices, their structure is vertical and not planar. In a planar structure, the current and breakdown voltage ratings are both functions of the channel dimensions (respectively width and length of the channel), resulting in inefficient use of the "silicon estate". With a vertical structure, the voltage rating of the transistor is a function of the doping and thickness of the N epitaxial layer (see cross section), while the current rating is a function of the channel width. This makes possible for the transistor to sustain both high blocking voltage and high current within a compact piece of silicon.It is worth noting that power MOSFETs with lateral structure exist. They are mainly used in high-end audio amplifiers. Their advantage is a better behaviour in the saturated region (corresponding to the linear region of a bipolar transistor) than the vertical MOSFETs. Vertical MOSFETs are designed for switching applications, so they are only used in On or Off states. the power MOSFET is in the onstate (see MOSFET for a discussion on operation modes), it exhibits a resistive behaviour between the drain and source terminals. It can be seen in figure 2 that this resistance (called R DSon for "drain to source resistance in on-state") is the sum of many elementary contributions: RS is the source resistance. It represents all resistances between the source terminal of the package to the channel of the MOSFET: resistance of the wire bonds, of the source metallisation, and of the N+ wells; Rch. This is the channel resistance. It is inversely proportional to the channel width, and for a given die size, to the channel density. The channel resistance is one of the main contributors to the RDSon of low-voltage MOSFETs, and intensive work has been carried out to reduce their cell size in order to increase the channel density; Ra is the access resistance. It represents the resistance of the epitaxial zone directly under the gate electrode, where the direction of the current changes from horizontal (in the channel) to vertical (to the drain contact); RJFET is the detrimental effect of the cell size reduction mentioned above: the P implantations (see figure 1) form the gates of a parasitic JFET transistor that tend to reduce the width of the current flow; Rn is the resistance of the epitaxial layer. As the role of this layer is to sustain the blocking voltage, Rn is directly related to the voltage rating of the device. A high voltage MOSFET requires a thick, low-doped layer (i.e. highly resistive), whereas a low-voltage transistor only requires a thin layer with a higher doping level (i.e. less resistive). As a result, Rn is the main factor responsible for the resistance of high-voltage MOSFETs;
153 | P a g e

When in the OFF-state, the power MOSFET is equivalent to a PIN diode (constituted by the P + diffusion, the N- epitaxial layer and the N+ substrate). When this highly non-symmetrical structure is reverse-biased, the space-charge region extends principally on the light-doped side, i.e over the Nlayer. This means that this layer has to withstand most of the MOSFET's OFF-state drain-to-source voltage.However, when the MOSFET is in the ON-state, this N- layer has no function. Furthermore, as it is a lightly-doped region, its intrinsic resistivity is non-negligible and adds to the MOSFET's ONstate Drain-to-Source Resistance .Two main parameters govern both the breakdown voltage and the RDSon of the transistor: the doping level and the thickness of the N- epitaxial layer. The thicker the layer and the lower its doping level, the higher the breakdown voltage. On the contrary, the thinner the layer and the higher the doping level, the lower the RDSon (and therefore the lower the conduction losses of the MOSFET). Therefore, it can be seen that there is a trade-off in the design of a MOSFET, between its voltage rating and its ON-state resistance. It can be seen in figure 1 that the source metallization connects both the N+ and P implantations, although the operating principle of the MOSFET only requires the source to be connected to the N + zone. However, if it were, this would result in a floating P zone between the N-doped source and drain, which is equivalent to a NPN transistor with a non-connected base. Under certain conditions (under high drain current, when the on-state drain to source voltage is in the order of some volts), this parasitic NPN transistor would be triggered, making the MOSFET uncontrollable. The connection of the P implantation to the source metallization shorts the base of the parasitic transistor to its emitter (the source of the MOSFET) and thus prevents spurious latching. Explain transistor hybrid model for common emitter configuration? Ans. Small-signal models This two-port network is particularly suited to BJTs as it lends itself easily to the analysis of circuit behaviour, and may be used to develop further accurate models. As shown, the term "x" in the model represents a different BJT lead depending on the topology used. For common-emitter mode the various symbols take on the specific values as: x = 'e' because it is a common-emitter topology Terminal 1 = Base Terminal 2 = Collector Terminal 3 = Emitter ii = Base current (ib) io = Collector current (ic) Vin = Base-to-emitter voltage (VBE) Vo = Collector-to-emitter voltage (VCE) and the h-parameters are given by: hix = hie The input impedance of the transistor (corresponding to the emitter resistance re). hrx = hre Represents the dependence of the transistor's IBVBE curve on the value of VCE. It is usually very small and is often neglected (assumed to be zero). hfx = hfe The current-gain of the transistor. This parameter is often specified as hFE or the DC current-gain (DC) in datasheets. hox = hoe The output impedance of transistor. This term is usually specified as an admittance and has to be inverted to convert it to an impedance. As shown, the h-parameters have lower-case subscripts and hence signify AC conditions or analyses. For DC conditions they are specified in upper-case. For the CE topology, an approximate h-parameter model is commonly used which further simplifies the circuit analysis. For this the hoe and hre parameters are neglected (that is, they are set to infinity and zero, respectively). It should also be noted that the h-parameter model as shown is suited to low-frequency, small-signal analysis. For high-frequency analyses the inter-electrode capacitances that are important at high frequencies must be added. 10. What is safe operating area of a BJT?
154 | P a g e

For power semiconductor devices (such as BJT, MOSFET, thyristor or IGBT), the safe operating area (SOA) is defined as the voltage and current conditions over which the device can be expected to operate without self-damage. SOA is usually presented in transistor datasheets as a graph with VCE (collector-emitter voltage) on the abscissa and ICE (collector-emitter current) on the ordinate; the safe 'area' referring to the area under the curve. The SOA specification combines the various limitations of the device maximum voltage, current, power, junction temperature, second breakdown into one curve, allowing simplified design of protection circuitry.SOA specifications are useful to the design engineer working on power circuits such as amplifiers and power supplies as they allow quick assessment of the limits of device performance, the design of appropriate protection circuitry, or selection of a more capable device. Q.10. What are the advantages of BJT over MOSFET? Ans. Advantages of BJT over MOSFETBJTs have some advantages over MOSFETs for at least two digital applications. Firstly, in high speed switching, they do not have the "larger" capacitance from the gate, which when multiplied by the resistance of the channel gives the intrinsic time constant of the process. The intrinsic time constant places a limit on the speed a MOSFET can operate at because higher frequency signals are filtered out. Widening the channel reduces the resistance of the channel, but increases the capacitance by exactly the sa me amount. Reducing the width of the channel increases the resistance, but reduces the capacitance by the same amount. R*C=Tc1, 0.5R*2C=Tc1, 2R*0.5C=Tc1. There is no way to minimize the intrinsic time constant for a certain process. Different processes using different channel lengths, channel heights, gate thicknesses and materials will have different intrinsic time constants. This problem is mostly avoided with a BJT because it does not have a gate.The second application where BJTs have an advantage over MOSFETs stems from the first. When driving many other gates, called fanout, the resistance of the MOSFET is in series with the gate capacitances of the other FETs, creating a secondary time constant. Delay circuits use this fact to create a fixed signal delay by using a small CMOS device to send a signal to many other, many times larger CMOS devices. The secondary time constant can be minimized by increasing the driving FET's channel width to decrease its resistance and decreasing the channel widths of the FETs being driven, decreasing their capacitance. The drawback is that it increases the capacitance of the driving FET and increases the resistance of the FETs being driven, but usually these drawbacks are a minimal problem when compared to the timing proble m. BJTs are better able to drive the other gates because they can output more current than MOSFETs, allowing for the FETs being driven to charge faster. Many chips use MOSFET inputs and BiCMOS. Q 11. What are merits of hybrid model? Ans. The hybrid model is a popular circuit model used for analyzing the small signal behavior of bipolar junction and field effect transistors. The model can be quite accurate for low-frequency circuits and can easily be adapted for higher frequency circuits with the addition of appropriate inter-electrode capacitances and other parasitic elements. The hybrid model is a linearized two-port network approximation to the BJT using the small-signal base-emitter voltage vbe and collector-emitter voltage vce as independent variables, and the smallsignal base current ib and collector current ic as dependent variables.A basic, low-frequency hybrid model for the bipolar transistor is the transconductance in siemens, evaluated in a simple mode where: is the quiescent collector current (also called the collector bias or DC collector current) is the thermal voltage, calculated from Boltzmann's constant k, the charge of an electron q, and the transistor temperature in kelvins, T. At 295 K (approximately room temperature) VT is about 25 mV (Google calculator). in ohms where: is the current gain at low frequencies (commonly called h FE). Here IB is the Q-point base current. This is a parameter specific to each transistor, and can be found on a datasheet; is a function of the choice of collector current.
155 | P a g e

Q 12. Differentiate small signal model with large signal model? Ans. small signal model large signal model to approximate the behavior to describe nonlinear devices in terms of the of nonlinear devices with underlying nonlinear equations. In circuits linear equations This containing nonlinear elements such as linearization is formed about transistors, diodes, and vacuum tubes, under the DC bias point of the device "large signal conditions", AC signals have high (that is, the voltage/current enough magnitude that nonlinear effects must levels present when no signal be considered."Large signal" is the opposite of is applied), and can be "small signal", which means that the circuit can accurate for small excursions be reduced to a linearized equivalent circuit about this point. around its operating point with sufficient accuracy Q 13. Mention the importance of power transistor? Ans. The transistor is the key active component in practically all modern electronics, and is considered by many to be one of the greatest inventions of the twentieth century.ts importance in today's society rests on its ability to be mass produced using a highly automated process (semiconductor device fabrication) that achieves astonishingly low per-transistor costs.Although several companies each produce over a billion individually packaged (known as discrete) transistors every year, the vast majority of transistors now produced are in integrated circuits (often shortened to IC, microchips or simply chips), along with diodes, resistor, capacitors and other electronic components, to produce complete electronic circuits. Q 14. Derive the relation among alpha. Beta and Gama? Ans. We know that = Ic/Ie = Ic/Ib and = Ie/Ib Also Ie = Ib + Ic Putting this values in above we will get = / . Q 15. Explain early effect? Ans. The Early effect is the variation in the width of the base in a bipolar junction transistor (BJT) due to a variation in the applied base-to-collector voltage. greater reverse bias across the collector base junction, for example, increases the collectorbase depletion width, decreasing the width of the charge neutral portion of the base.In Figure 1 the neutral (i.e. active) base is green, and the depleted base regions are hashed light green. The neutral emitter and collector regions are dark blue and the depleted regions hashed light blue. Under increased collectorbase reverse bias, the lower panel of Figure 1 shows a widening of the depletion region in the base and the associated narrowing of the neutral base region.The collector depletion region also increases under reverse bias, more than does that of the base, because the collector is less heavily doped. The principle governing these two widths is charge neutrality. The narrowing of the collector does not have a significant effect as the collector is much longer than the base. The emitterbase junction is unchanged because the emitterbase voltage is the same.Base-narrowing has two consequences that affect the current:There is a lesser chance for recombination within the "smaller" base region.
156 | P a g e

The charge gradient is increased across the base, and consequently, the current of minority carriers injected across the emitter junction increases. Both these factors increase the collector or "output" current of the transistor with an increase in the collector voltage. Q 16. What do you mean by biasing of a transistor? Ans. Bipolar transistor amplifiers must be properly biased to operate correctly. In circuits made with individual devices (discrete circuits), biasing networks consisting of resistor are commonly employed. Much more elaborate biasing arrangements are used in integrated circuits, for example, bandgap voltage references and current mirrors.The operating point of a device, also known as bias point, quiescent point, or Q-point, is the point on the output characteristics that shows the DC collectoremitter voltage (Vce) and the collector current (Ic) with no input signal applied. The term is normally used in connection with devices such as transistors. Q 17 .What is thermal run away? Ans. Maximum average power which is a transistor can dissipate depends upon the transistor construction and may lie in the range from a few milli watts to 200 watts. This maximum power is limited by the temperature that collector to base junction cam with stand. For silicon transistor this temperature is in the range 150 to 2250 C, and for Ge it is between 60 and 1000C. This temperature may rise either because the ambient temperature rises or because of self heating. The maximum power dissipation is usually specified for the transistor enclosure(case) or ambient temperature of 250 C. Q 18. What are the advantages of BJT? Ans. Smaller size (despite continuing miniaturization of vacuum tubes)Highly automated manufacture Lower cost (in volume production) Lower possible operating voltages Operation without a warm-up period (most vacuum tubes need 10 to 60 seconds to "warm up")Lower power dissipation (no heater power, very low saturation voltage) Q19. Underline the Importance of h parameter. Ans. (i)H parameter depends on temperature. (ii) more semsitive to collector. (iii) Input impedence of an emitter follower is very high. Q20. Write uses of hall effect? Ans. (i) Determination of semiconductor type (ii) Determination of carrier concentration (iii) Determination of mobility (iv) Measurement of power in EM wave. (v) Measurement of magnetic flux density

157 | P a g e

Unit III Power Electronic Circuits: Part A Q. 1 What UPS stands for? Ans. UPS stands for Un intrruptible Power Supply. Q. 2 Explain the working principal ofThyristor. Ans. A thyristor is a solid-state semiconductor device with four layers of alternating N and P-type material. They act as bistable switches, conducting when their gate receives a current pulse, and continue to conduct for as long as they are forward biased (that is, as long as the voltage across the device has not reversed).Some sources define silicon controlled rectifiers and thyristors as synonymous.Other sources define thrusters as a larger set of devices with at least four layers of alternating N and P-type material, including Q. 3 Explain the Principles of Operation of thyristor. Ans. The thyristor has three p-n junctions (serially named J1, J2, J3 from the anode).When the anode is at a positive potential VAK with respect to the cathode with no voltage applied at the gate, junctions J1 and J3 are forward biased, while junction J2 is reverse biased. As J2 is reverse biased, no conduction takes place (Off state). Now if VAK is increased beyond the breakdown voltage VBO of the thyristor, avalanche breakdown of J2 takes place and the thyristor starts conducting (On state).If a positive potential VG is applied at the gate terminal with respect to the cathode, the breakdown of the junction J2 occurs at a lower value of VAK. By selecting an appropriate value of VG, the thyristor can be switched into the on state suddenly.Once avalanche breakdown has occurred, the thyristor continues to conduct, irrespective of the gate voltage, until both: (a) the potential VG is removed and (b) the current through the device (anodecathode) is less than the holding current specified by the manufacturer. Hence VG can be a voltage pulse, such as the voltage output from a UJT relaxation oscillator.These gate pulses are characterized in terms of gate trigger voltage (VGT) and gate trigger current (IGT). Gate trigger current varies inversely with gate pulse width in such a way that it is evident that there is a minimum gate charge required to trigger Q. 4 Explain the characteristics of SCR. Ans. Switching characteristic : in a conventional thyristor, once it has been switched on by the gate terminal, the device remains latched in the on-state (i.e. does not need a continuous supply of gate current to conduct), providing the anode current has exceeded the latching current ( IL). As long as the anode remains positively biased, it cannot be switched off until the anode current falls below the holding current (IH).

V - I characteristics. A thyristor can be switched off if the external circuit causes the anode to become negatively biased. In some applications this is done by switching a second thyristor to discharge a capacitor into the cathode of the first thyristor. This method is called forced commutation.
158 | P a g e

After a thyristor has been switched off by forced commutation, a finite time delay must have elapsed before the anode can again be positively biased and retain the thyristor in the off-state. This minimum delay is called the circuit commutated turn off time (tQ). Attempting to positively bias the anode within this time causes the thyristor to be self-triggered by the remaining charge carriers (holes and electrons) that have not yet recombined. For applications with frequencies higher than the domestic AC mains supply (e.g. 50 Hz or 60 Hz), thyristors with lower values of tQ are required. Such fast thyristors are made by diffusing into the silicon heavy metals ions such as gold or platinum which act as charge combination centres. Q. 5 What is Snubber circuit? Ans. Thyristors can be triggered by a high rate of rise of off-state voltage. This is prevented by connecting a resistor-capacitor (RC) snubber circuit between the anode and cathode terminals in order to limit the dV/dt (i.e., rate of change of voltage versus time). Q. 6 Explain the working principle of DIAC? Ans. DIAC or 'diode for alternating current, is a trigger diode that conducts current only after its breakdown voltage has been reached momentarily. When this occurs, diode enters the region of negative dynamic resistance, leading to a decrease in the voltage drop across the diode and, usually, a sharp increase in current through the diode. The diode remains "in conduction" until the current through it drops below a value characteristic for the device, called the holding current. Below this value, the diode switches back to its high-resistance (non-conducting) state. This behavior is bidirectional, meaning typically the same for both directions of current. Q. 7 Explain the working principle of TRIAC? Ans. TRIAC, from Triode for Alternating Current, is a genericized tradename for an electronic component which can conduct current in either direction when it is triggered (turned on), and is formally called a bidirectional triode thyristor or bilateral triode thyristor. Q. 8 What SCR stands for? Ans. Silicon controlled rectifiers Q. 9 What does UJT stand for ? Ans. Uni junction transistor . it is a three terminal . It consists of three terminals but has only one junction since its structure resembles transistor hence the name . Uni junction transistor . Q. 10 What is intrinsic stand off ratio? Ans. Intrinsic stand off ratio = RB1 / RB1+RB2 11. How does UJT differ from FET? Ans.
FET Gate is much larger Gate junction reverse biased Amplifying device Used in amplifiers UJT Emitter junction area is smaller Emitter junction is forward biased Switching device Used in relaxationoscilator ccc .

Q. 11 What is an inverter? Ans. An inverter is an electrical device that converts direct current (DC) to alternating current (AC); the converted AC can be at any required voltage and frequency with the use of appropriate
159 | P a g e

transformers, switching, and control circuits.Static inverters have no moving parts and are used in a wide range of applications, from small switching power supplies in computers, to large electric utility high-voltage direct current applications that transport bulk power. Inverters are commonly used to supply AC power from DC sources such as solar panels or batteries.The electrical inverter is a high-power electronic oscillator. It is so named because early mechanical AC to DC converters were made to work in reverse, and thus were "inverted", to convert DC to AC. Q. 12 Write short notes on power supply. Ans. power supply is a device that supplies electrical energy to one or more electric loads. The term is most commonly applied to devices that convert one form of electrical energy to another, though it may also refer to devices that convert another form of energy (e.g., mechanical, chemical, solar) to electrical energy. A regulated power supply is one that controls the output voltage or current to a specific value; the controlled value is held nearly constant despite variations in either load current or the voltage supplied by the power supply's energy source.Every power supply must obtain the energy it supplies to its load, as well as any energy it consumes while performing that task, from an energy source. Depending on its design, a power supply may obtain energy from:Electrical energy transmission systems. Common examples of this include power supplies that convert AC line voltage to DC voltage. Energy storage devices such as batteries and fuel cells electromechanical systems such as generators and alternators. Solar power. A power supply may be implemented as a discrete, stand-alone device or as an integral device that is hardwired to its load. In the latter case, for example, low voltage DC power supplies are commonly integrated with their loads in devices such as computers and household electronics. Constraints that commonly affect power supplies include:The amount of voltage and current they can supply. How long they can supply energy without needing some kind of refueling or recharging (applies to power supplies that employ portable energy sources). How stable their output voltage or current is under varying load conditions. Whether they provide continuous or pulsed energy. Q. 13 What is chopper circuit? Ans. chopper circuit is used to refer to numerous types of electronic switching a chopper is an electronic switch that is used to interrupt one signal under the control of another. Most modern uses also use alternative nomenclature which helps to clarify which particular type of circuit is being discussed. These include: switched mode power supplies, including DC to DC converters. Speed controllers for DC motors Class D Electronic amplifiers, Switched capacitor filters, Variable Frequency Drive, Chopper amplifiers, One classic use for a chopper circuit and where the term is still in use is in chopper amplifiers. These are DC amplifiers. Some types of signal that need amplifying can be so small that an incredibly high gain is required, but very high gain DC amplifiers are much harder to build offset and 1/ f noise, and reasonable stability and bandwidth. It's much easier to build an AC amplifier instead. A chopper circuit is used to break up the input signal so that it can be processed as if it were an AC signal, then integrated back to a DC signal at the output. In this way, extremely small DC signals can be amplified. This approach is often used in electronic instrumentation where stability and accuracy are essential; for example, it is possible using these techniques to construct pico-voltmeters and Hall sensors The input offset voltage of amplifiers becomes important when trying to amplify small signals with very high gain. Because this technique creates a very low input offset voltage amplifier, and because this input offset voltage does not change much with time and temperature, these techniques are also called "Zero-Drift" amplifiers (because there is no drift in input offset voltage with time and temperature). Related techniques that also give these Zero-drift advantages are Auto-zero and Chopper-stabilized Amplifiers. Autozero amplifiers use a secondary auxiliary amplifier to correct the input offset voltage of a main amplifier. Chopper-stabilized amplifiers use a combination of auto-zero and chopper techniques to give some excellent DC precision specifications.
160 | P a g e

Q. 14 What SMPS stands for? Ans. SMPS stands for Switched Mode Power Supply. switched-mode power supply (switching-mode power supply, SMPS, or simply switcher). Q. 15 Explain the importance of SMPS. Ans. Highly efficient in the conversion of electrical power. Like other types of power supplies, an SMPS transfers power from a source like the electrical power grid to a load (e.g., a personal computer) while converting voltage and current characteristics. An SMPS is usually employed to efficiently provide a regulated output voltage, typically at a level different from the input voltage. Unlike a linear power supply, the pass transistor of a switching mode supply switches very quickly (typically between 50 kHz and 1 MHz) between full-on and full-off states, which minimizes wasted energy. Voltage regulation is provided by varying the ratio of on to off time. In contrast, a linear power supply must dissipate the excess voltage to regulate the output. Q. 16 What are the advantages of using SMPS? Ans. This higher efficiency is the chief advantage of a switch-mode power supply. Switching regulators are used as replacements for the linear regulators when higher efficiency, smaller size or lighter weight are required. They are, however, more complicated, their switching currents can cause electrical noise problems if not carefully suppressed, and simple designs may have a poor power factor. Q. 17 Three-phase half-wave rectifier? Ans. half wave rectifier uses three, common-cathode thyristor arrangement., thepower supply, and the transformer are assumed ideal. The thyristor will conduct (ON state)when the anode-tocathode voltage vAK is positive, and a firing current pulse iG is applied to thegate terminal. Delaying from the crossing point between the phase supply voltages, as shownin At that point, the anode-tocathode thyristor voltage vAK begins to be positive. the possible range for gating delay is between is limited to around160. when the load is resistive, the current id has the same waveform of the loadvoltage. As the load becomes more and more inductive, the current flattens and finally becomesconstant. The thyristor goes to the non-conducting condition (OFF state) when the followingthyristor is switched ON, or the current, tries to reach a negative value.With the help of figure, the load average voltage. Q. 18 What MOSFET stands for? Ans. Metal Oxide Semiconductor Field Effect Transistor.
Q. 19 What is power MOSFET? Ans. Power MOSFET is a specific type of metal oxide semiconductor field-effect transistor (MOSFET) designed to handle significant power levels. Compared to the other power semiconductor devices (IGBT, Thyristor), its main advantages are high commutation speed and good efficiency at low voltages. It shares with the IGBT an isolated gate that makes it easy to drive. It was made possible by the evolution of CMOS technology. The power MOSFET shares its operating principle with its low-power counterpart, the lateral MOSFET.

Q.20 Write the application of power MOSFET. Ans. The power MOSFET is the most widely used low-voltage (i.e. less than 200 V) switch. It can be found in most power supplies, DC to DC converters, and low voltage motor controllers.

161 | P a g e

PART - B
Q 1. What SCR stands for? Ans. It stands for Silicon Controlled Rectifier. Q 2. What does UJT stand for? Ans. It stands for Uni junction Transistor. Q 3. Explain the principle of working of Thyristor. Ans. The current introduced into oneor both gate terminals may be used to control the anode to cathode break over voltage.if current through one or both outer junctions is increased as a result of current is introduced in the gate terminal, alpha increase and break over voltage increase. At higher gate current break over may occur at lower voltage. Q 4. What are different types of thyristor? Ans. Gate turn on thyristor, SCS,DIAC,TRIAC etc Q 5. Explain the difference between UJT and FET? Ans. UJT FET It consists of only one junction. It consists of two PN junctions. Single type of current carrier is responsible for Both majority as well as minority type of the conduction of current. current carrier is responsible for the conduction of current. Q 6. Explain the working principle of DIAC? Ans. A DIAC operated with a DC voltage across it behaves exactly the same as a Shockley diode. With AC, however, the behavior is different from what one might expect. Because alternating current repeatedly reverses direction, DIACs will not stay latched longer than one-half cycle. If a DIAC becomes latched, it will continue to conduct current only as long as voltage is available to push enough current in that direction. When the AC polarity reverses, as it must twice per cycle, the DIAC will drop out due to insufficient current, necessitating another breakover before it conducts again. Q 7. Explain the working principle of TRIAC? Ans. The triac is similar in operation to two thyristors connected in reverse parallel but using a common gate connection. This gives the triac the ability to be triggered into conduction while having a voltage of either polarity across it. In fact it acts rather like a "full wave" thyristor. Either positive or negative gate pulses may be used. Triacs are mainly used in power control to give full wave control. This enables the voltage to be controlled between zero and full power. With simple "half wave" thyristor circuits the controlled voltage may only be varied between zero and half power as the thyristor only conducts during one half cycle. The triac provides a wider range of control in AC circuits without the need for additional components, e.g. bridge rectifiers or a second thyristor, needed to achieve full wave control with thyristors. The triggering of the triac is also simpler than that required by thyristors in AC circuits, and can normally be achieved using a simple DIAC circuit. A simplified triac control circuit is shown in Figure 2. The operation will be explained after introducing the Diac.

162 | P a g e

Q 8. Explain the advantages and disadvantages of power devices? Ans. The thyristor appeared in 1957. Thyristors are able to withstand very high reverse breakdown voltage and are also capable of carrying high current. One disadvantage of the thyristor for switching circuits is that once it is 'latched-on' in the conducting state it cannot be turned off by external control. The thyristor turn-off is passive, i.e., the power must be disconnected from the device. The first bipolar transistors devices with substantial power handling capabilities were introduced in the 1960s. These components overcame some limitations of the thyristors because they can be turned on or off with an applied signal. Q 9. Why power devices are used? Ans. Provide a way to remove the heat generated by the device; protect the die from the external environment (moisture, dust); Many of the reliability issues of power device are either related to excessive temperature of fatigue due to thermal cycling. Research is currently carried out on the following topics: improve the cooling performance. improve the resistance to thermal cycling by closely matching the Coefficient of thermal expansion of the packaging to that of the silicon. increase the maximum operating temperature of the packaging material The schematic symbol is Figu

Q 10. Explain the principle of working of UJT? Ans. Unijunction transistor (abbreviated as UJT), also called the double-base diode is a 2-layer, 3terminal solid-state (silicon) switching device. The device has-a unique characteristic that when it is triggered, its emitter current increases re generatively (due to negative resistance characteristic) until it is restricted by emitter power supply. The low cost per unit, combined with its unique characteristic, have warranted its use in a wide variety of applications. A few include oscillators, pulse generators, saw-tooth generators, triggering circuits, phase control, timing circuits, and voltage-or current-regulated supplies. The device is in general, a low-power-absorbing device under normal operating conditions and provides tremendous aid in the continual effort to design relatively efficient systems!

163 | P a g e

Construction of a UJT The basic structure of a unijunction transistor is shown in figure. It essentially consists of a lightlydoped N-type silicon bar with a small piece of heavily doped P-type material alloyed to its one side to produce single P-N junction. The single P-N junction accounts for the terminology unijunction. The silicon bar, at its ends, has two ohmic contacts designated as base-1 (B1) and base-2 (B2), as shown and the P-type region is termed the emitter (E). The emitter junction is usually located closer to base-2 (B2) than base-1 (B1) so that the device is not symmetrical, because symmetrical unit does not provide optimum electrical characteristics for most of the applications.

UJT Symbol and Construction The symbol for unijunction transistor is shown in figure. The emitter leg is drawn at an angle to the vertical line representing the N-type material slab and the arrowhead points in the direction of conventional current when the device is forward-biased, active or in the conducting state. The basic arrangement for the UJT is shown in figure. A complementary UJT is formed by diffusing an N-type emitter terminal on a P-type base. Except for the polarities of voltage and current, the characteristics of a complementary UJT are exactly the same as those of a conventional UJT. The worth noting points about UJT are given below:

The device has only one junction, so it is called the unijunction device. The device, because of one P-N junction, is quite similar to a diode but it differs from an ordinary diode as it has three terminals. The structure of a UJT is quite similar to that of an N-channel JFET. The main difference is that Ptype (gate) material surrounds the N-type (channel) material in case of JFET and the gate surface of the JFET is much larger than emitter junction of UJT. In a unijunction transistor the emitter is heavily doped while the N-region is lightly doped, so the resistance between the base terminals is relatively high, typically 4 to 10 kilo Ohm when the emitter is open. The N-type silicon bar has a high resistance and the resistance between emitter and base-1 is larger than that between emitter and base-2. It is because emitter is closer to base-2 than base-1. UJT is operated with emitter junction forward- biased while the JFET is normally operated with the gate junction reverse-biased.

UJT does not have ability to amplify but it has the ability to control a large ac power with a small signal. It exhibits a negative resistance characteristic and so it can be employed as an oscillator.

164 | P a g e

Operation of a UJT Imagine that the emitter supply voltage is turned down to zero. Then the intrinsic stand-off voltage reverse-biases the emitter diode, as mentioned above. If VB is the barrier voltage of the emitter diode, then the total reverse bias voltage is VA + VB = VBB + VB. For silicon VB = 0.7 V.Now let the emitter supply voltage VE be slowly increased. When VE becomes equal to VBB, IEo will be reduced to zero. With equal voltage levels on each side of the diode, neither reverse nor forward current will flow. When emitter supply voltage is further increased, the diode becomes forward-biased as soon as it exceeds the total reverse bias voltage ( VBB + VB). This value of emitter voltage VE is called the peak-point voltage and is denoted by VP. When VE = VP, emitter current IE starts to flow through RB1 to ground, that is B1. This is the minimum current that is required to trigger the UJT. This is called the peak-point emitter current and denoted by IP. Ip is inversely proportional to the interbase voltage, VBB. Now when the emitter diode starts conducting, charge carriers are injected into the RB region of the bar. Since the resistance of a semiconductor material depends upon doping, the resistance of region RB decreases rapidly due to additional charge carriers (holes). With this decrease in resistance, the voltage drop across RB also decrease, cause the emitter diode to be more heavily forward biased. This, in turn, results in larger forward current, and consequently more charge carriers are injected causing still further reduction in the resistance of the RB region. Thus the emitter current goes on increasing until it is limited by the emitter power supply. Since VA decreases with the increase in emitter current, the UJT is said to have negative resistance characteristic. It is seen that the base-2 (B2) is used only for applying external voltage VBB across it. Terminals E and B1 are the active terminals. UJT is usually triggered into conduction by applying a suitable positive pulse to the emitter. It can be turned off by applying a negative trigger pulse. Q 11. Explain the principle of working of mosfet? Ans. MOSFET Mosfet Metal (or poly-silicon doped heavily to act like a metal) Oxide (SiO2, Acts as an insulator.) . Semiconductor (One can selectively change the carrier type to n-type or p-type.) Field Effect (Device is controlled by an electric field as opposed to current.) Transistor (Three terminal device)

165 | P a g e

(a) When VGS (Gate-source voltage) is not supplied (b) When VGS (Gate-source voltage) is supplied

Figure 3: The Structure of an Enhancement Type MOSFET and its Operation (a) When VGS (Gate-source voltage) is not supplied (b) When VGS (Gate-source voltage) is supplied Q 12. Enlist advantages of the lateral MOSFET ? The advantages of the lateral MOSFET are: 1. Low gate signal power requirement. No gate current can flow into the gate after the small gate oxide capacitance has been charged. 2. Fast switching speeds because electrons can start to flow from drain to source as soon as the channel opens. The channel depth is proportional to the gate volage and pinches closed as soon as the gate voltage is removed, so there is no storage time effect as occurs in bipolar transistors. Q 13. Enlist advantages of the lateral MOSFET ? Ans. The major disadvantages are 1. High resistance channels. In normal operation, the source is electrically connected to the substrate. With no gate bias, the depletion region extends out from the Nadrain in a pseudo166 | P a g e

hemispherical shape. The channel length L cannot be made shorter than the minimum depletion width required to support the rated voltage of the device. 2. Channel resistance may be decreased by creating wider channels but this is costly since it uses up valuable silicon real estate. It also slows down the switching speed of the device by increasing its gate capacitance.

Q 14. Explain the principle of working of rectifier? Ans. When four diodes are connected as shown in figure 4-8, the circuit is called a BRIDGE RECTIFIER. The input to the circuit is applied to the diagonally opposite corners of the network, and the output is taken from the remaining two corners.

One complete cycle of operation will be discussed to help you understand how this circuit works. We have discussed transformers in previous modules in the NEETS series and will not go into their characteristics at this time. Let us assume the transformer is working properly and there is a positive potential at point A and a negative potential at point B. The positive potential at point A will forward bias D3 and reverse bias D4. The negative potential at point B will forward bias D1 and reverse bias D2. At this time D3 and D1 are forward biased and will allow current flow to pass through them; D4 and D2 are reverse biased and will block current flow. The path for current flow is from point B through D1, up through RL, through D3, through the secondary of the transformer back to point B. This path is indicated by the solid arrows. Waveforms (1) and (2) can be observed across D1 and D3. One-half cycle later the polarity across the secondary of the transformer reverses, forward biasing D2 and D4 and reverse biasing D1 and D3. Current flow will now be from point A through D4, up through RL, through D2, through the secondary of T1, and back to point A. This path is indicated by the broken arrows. Waveforms (3) and (4) can be observed across D2 and D4. You should have noted that the current flow through RL is always in the same direction. In flowing through RL this current develops a voltage corresponding to that shown in waveform (5). Since current flows through the load (RL) during both half cycles of the applied voltage, this bridge rectifier is a full-wave rectifier. One advantage of a bridge rectifier over a conventional full-wave rectifier is that with a given transformer the bridge rectifier produces a voltage output that is nearly twice that of the conventional full-wave circuit. This may be shown by assigning values to some of
167 | P a g e

the components shown in views A and B of figure 4-9. Assume that the same transformer is used in both circuits. The peak voltage developed between points X and Y is 1000 volts in both circuits. In the conventional full-wave circuit shown in view A, the peak voltage from the center tap to either X or Y is 500 volts. Since only one diode can conduct at any instant, the maximum voltage that can be rectified at any instant is 500 volts. Therefore, the maximum voltage that appears across the load resistor is nearly - but never exceeds - 500 volts, as a result of the small voltage drop across the diode. In the bridge rectifier shown in view B, the maximum voltage that can be rectified is the full secondary voltage, which is 1000 volts. Therefore, the peak output voltage across the load resistor is nearly 1000 volts. With both circuits using the same transformer, the bridge rectifier circuit produces a higher output voltage than the conventional full-wave rectifier circuit.

Q 15. What do you understand by phase controlled rectification? Ans. Shockley diodes are curious devices, but rather limited in application. Their usefulness may be expanded, however, by equipping them with another means of latching. In doing so, each becomes true amplifying devices (if only in an on/off mode), and we refer to these as silicon-controlled rectifiers, or SCRs. The progression from Shockley diode to SCR is achieved with one small addition, actually nothing more than a third wire connection to the existing PNPN structure: (Figure below)

Q 16. What do you understand by half wave phase controlled rectification? Ans. SCR control of AC power Being a unidirectional (one-way) device, at most we can only deliver half-wave power to the load, in the half-cycle of AC where the supply voltage polarity is positive on the top and negative on the bottom. However, for demonstrating the basic concept of time-proportional control, this simple circuit is better than one controlling full-wave power (which would require two SCRs). With no triggering to the gate, and the AC source voltage well below the SCR's breakover voltage rating, the SCR will never turn on. Connecting the SCR gate to the anode through a standard rectifying diode (to prevent reverse current through the gate in the event of the SCR containing a built-in gate-cathode resistor), will allow the SCR to be triggered almost immediately at the beginning of every positive half-cycle: (Figure below)
168 | P a g e

Gate connected directly to anode through a diode; nearly complete half-wave current through load. We can delay the triggering of the SCR, however, by inserting some resistance into the gate circuit, thus increasing the amount of voltage drop required before enough gate current triggers the SCR. In other words, if we make it harder for electrons to flow through the gate by adding a resistance, the AC voltage will have to reach a higher point in its cycle before there will be enough gate current to turn the SCR on. The result is in Figure below.

Resistance inserted in gate circuit; less than half-wave current through load. With the half-sine wave chopped up to a greater degree by delayed triggering of the SCR, the load receives less average power (power is delivered for less time throughout a cycle). By making the series gate resistor variable, we can make adjustments to the time-proportioned power: (Figure below)

169 | P a g e

Increasing the resistance raises the threshold level, causing less power to be delivered to the load. Decreasing the resistance lowers the threshold level, causing more power to be delivered to the load. Unfortunately, this control scheme has a significant limitation. In using the AC source waveform for our SCR triggering signal, we limit control to the first half of the waveform's half-cycle. In other words, it is not possible for us to wait until after the wave's peak to trigger the SCR. This means we can turn down the power only to the point where the SCR turns on at the very peak of the wave: (Figure below)

Circuit at minimum power setting Raising the trigger threshold any more will cause the circuit to not trigger at all, since not even the peak of the AC power voltage will be enough to trigger the SCR. The result will be no power to the load.

170 | P a g e

An ingenious solution to this control dilemma is found in the addition of a phase-shifting capacit

Addition of a phase-shifting capacitor to the circuit

Q 17. What do you understand by full wave phase controlled rectification? Ans. Controlled bridge rectifier In any bridge rectifier circuit, the rectifying diodes (in this example, the rectifying SCRs) must conduct in opposite pairs. SCR1 and SCR3 must be fired simultaneously, and SCR2 and SCR4 must be fired together as a pair. As you will notice, though, these pairs of SCRs do not share the same cathode connections, meaning that it would not work to simply parallel their respective gate connections and connect a single voltage source to trigger both: (Figure below)

This strategy will not work for triggering SCR2 and SCR4 as a pair. Although the triggering voltage source shown will trigger SCR4, it will not trigger SCR2 properly because the two thyristors do not share a common cathode connection to reference that triggering voltage. ss

171 | P a g e

Transformer coupling of the gates allows triggering of SCR2 and SCR4 . Bear in mind that this circuit only shows the gate connections for two out of the four SCRs. Pulse transformers and triggering sources for SCR1 and SCR3, as well as the details of the pulse sources themselves, have been omitted for the sake of simplicity. Controlled bridge rectifiers are not limited to single-phase designs. In most industrial control systems, AC power is available in three-phase form for maximum efficiency, and solid-state control circuits are built to take advantage of that. A three-phase controlled rectifier circuit built with SCRs, without pulse transformers or triggering circuitry shown, would look like Figure below.

Three-phase bridge SCR control of load Q 18. Explain the working of SMPS? Ans. Theory of operation If the SMPS has an AC input, then the first stage is to convert the input to DC. This is called rectification. The rectifier circuit can be configured as a voltage doubler by the addition of a switch operated either manually or automatically. This is a feature of larger supplies to permit operation from nominally 120 V or 240 V supplies. The rectifier produces an unregulated DC voltage which is then sent to a large filter capacitor. The current drawn from the mains supply by this rectifier circuit occurs in short pulses around the AC voltage peaks. These pulses have significant high frequency
172 | P a g e

energy which reduces the power factor. Special control techniques can be employed by the following SMPS to force the average input current to follow the sinusoidal shape of the AC input voltage thus the designer should try correcting the power factor. An SMPS with a DC input does not require this stage. An SMPS designed for AC input can often be run from a DC supply (for 230 V AC this would be 330 V DC), as the DC passes through the rectifier stage unchanged. It's however advisable to consult the manual before trying this, though most supplies are quite capable of such operation even though nothing is mentioned in the documentation. However, this type of use may be harmful to the rectifier stage as it will only use half of diodes in the rectifier for the full load. This may result in overheating of these components, and cause them to fail prematurely. If an input range switch is used, the rectifier stage is usually configured to operate as a voltage doubler when operating on the low voltage (~120 V AC) range and as a straight rectifier when operating on the high voltage (~240 V AC) range. If an input range switch is not used, then a full-wave rectifier is usually used and the downstream inverter stage is simply designed to be flexible enough to accept the wide range of DC voltages that will be produced by the rectifier stage. In higher-power SMPSs, some form of automatic range switching may be used.The inverter stage converts DC, whether directly from the input or from the rectifier stage described above, to AC by running it through a power oscillator, whose output transformer is very small with few windings at a frequency of tens or hundreds of kilohertz (kHz). The frequency is usually chosen to be above 20 kHz, to make it inaudible to humans. The output voltage is optically coupled to the input and thus very tightly controlled. The switching is implemented as a multistage (to achieve high gain) MOSFET amplifier. MOSFETs are a type of transistor with a low on-resistance and a high current-handling capacity.If the output is required to be isolated from the input, as is usually the case in mains power supplies, the inverted AC is used to drive the primary winding of a high-frequency transformer This converts the voltage up or down to the required output level on its secondary winding. The output transformer in the block diagram serves this purpose.If a DC output is required, the AC output from the transformer is rectified. For output voltages above ten volts or so, ordinary silicon diodes are commonly used. For lower voltages, Schottky diodes are commonly used as the rectifier elements; they have the advantages of faster recovery times than silicon diodes (allowing low-loss operation at higher frequencies) and a lower voltage Q 19. Explain the working of UPS? Ans. UPS, or an uninterruptible power source, is an electronic device able to maintain an alternative continuous electric power supply to electronic equipment connected to it when the primary power sources become unavailable. Unlike auxiliary power, a UPS provides instant protection of the powered equipment but can output energy only for small amounts of time, usually up to several tens of minutes, allowing the electronic device connected to it to shutdown properly, which could otherwise result in extensive physical damage. Although its use can extend to a virtually unlimited list of applications, in the past years the UPS became even more popular as a means of protecting computers and telecommunication equipment, thus preventing serious hardware damage and data loss.A typical UPS for computers has four basic protection roles, being able to cope with power surges, voltage shortage, complete power failure and wide variations in the electric current frequency. There are three types of UPS systems, depending on how the electric power is being stored and relayed to the electronic device connected to them: the off-line UPS, also known as standby UPS, the line-interactive or continuous UPS and the on-line uninterruptible power supply, often called double conversion supply. The first two types are currently amongst the most common devices used to safeguard computers from unusual electric currents. Typical off-line UPS systems redirect the whole electric current received from the outlet to the computer and only switch to providing power from the battery when a problem is detected in the

173 | P a g e

utility power. Performing this action usually takes several milliseconds, during which time the power inverter starts supplying electric current from the battery to the computer. Line-interactive uninterruptible power supplies on the other hand, always relay electric energy through the battery to the computer, meaning that the battery of the UPS is continuously charging and the lag related to coupling the inverter is basically close to zero. Due to these characteristics,continu us UPS systems tend to be somehow more expensive than standby supplies. The last type, and the most expensive of the three UPS designs, the online supply, combines the two basic technologies used in the fabrication of the previously described UPS models, having rectifiers and inverter systems working all the time. As is the case with line-interactive UPS devices, the power transfer is made instantly as an outage occurs, the rectifier simply being turned off while the inverter draws power from the battery. Q 20. Explain the methods of triggering of SCR? Ans. The methods of triggering of SCR are; Base bias method di/dt method

174 | P a g e

UNIT - IV Digital Electronics: Digital Circuits systems and Devices (DCSD): PART A Q 1 What is the difference between analog an digital systems? Ans. Analog means continuous Digital means discrete (step by step). Q 2 What are the advantages of a digital system? Ans. Easier to design. Exact values of voltage or current are not important, only the range (HIGH or LOW) in which they fall. Information storage is easy. Accuracy and precision are greater. Q 3 What are the LIMITATIONS of a digital system? Ans. Operations can be programmed. Analog systems can also be programmed, but the available operations variety and complexity is severely limited. Digital circuits are less affected by noise, as long as the noise is not large enough to prevent us from distinguishing HIGH from LOW (we discuss this in detail in an advanced digital tutorial section). More digital circuitry can be fabricated on IC chips Most physical quantities in real world are analog in nature, and these quantities are often the inputs and outputs that are being monitored, operated on, and controlled by a system. Thus conversion to digital format and re-conversion to analog format is needed. Q 4 Draw the truth table for OR gate? Ans.
Input 1 0 0 1 Input 2 0 1 0 OUTPUT 0 1 1

Q 5 Draw the truth table for AND gate? ANS .


Input 1 0 0 1 1 Input 2 0 1 0 1 OUTPUT 0 0 0 1

175 | P a g e

Q 6 Draw the truth table for NOT gate. Ans. Input output 0 1 1 0

Q 7 Draw the truth table for NOR gate. Ans. Input 1 Input 2 OUTPUT 0 0 1 1 0 1 0 1 1 0 0 0

Q 8 Drawn the truth table for NAND gate Input 1 0 0 1 1 Input 2 0 1 0 1 OUTPUT 1 1 1 0

Q 9 What is register ? Ans . computer architecture, a processor register (or general purpose register) is a small amount of storage available on the CPU whose contents can be accessed more quickly than storage available elsewhere. Typically, this specialized storage is not considered part of the normal memory range for the machine. Most, but not all, modern computers adopt the so-called load-store architecture. Under this paradigm, data is loaded from some larger memory be it cache or RAM into registers, manipulated or tested in some way (using machine instructions for arithmetic/logic/comparison) and then stored back into memory, possibly at some different location. A common property of computer programs is locality of reference: the same values are often accessed repeatedly; and holding these frequently used values in registers improves program execution performance. Q 10 What is a multiplexer? Ans . In electronics, a multiplexer or mux (occasionally the terms muldex or muldem are also foun for a combination multiplexer-demultiplexer) is a device that performs multiplexing; it selects one of many analog or digital input signals and forwards the selected input into a single line. A
176 | P a g e

multiplexer of 2n inputs has n select lines, which are used to select which input line to send to the output. An electronic multiplexer makes it possible for several signals to share one device or resource, for example one A/D converter or one communication line, instead of having one device per input signal.On the other end, a demultiplexer (or demux) is a device taking a single input signal and selecting one of many data-output-lines, which is connected to the single input. A multiplexer is often used with a complementary demultiplexer on the receiving end. Q 11 What is an encoder? Ans. encoder is a device, circuit, transducer, software program, algorithm or person that converts information from one format or code to another, for the purposes of standardization, speed, secrecy, security, or saving space by shrinking size. Q 12 What a D- A converter does? Ans. Electronics digital-to-analog converter (DAC or D-to-A) is a device that converts a digital (usually binary) code to an analog signal (current voltage or electric charge). An analog-to-digital converter (ADC) performs the reverse operation.

Q 13 What is half adder? Ans. In electronics, an adder or summer is a digital circuit that performs addition of numbers. In modern computers adders reside in the arithmetic logic unit (ALU) where other operations are performed. Although adders can be constructed for many numerical representations, such as Binary-coded deciml or excess-3, the most common adders operate on binary numbers. In cases where two's complement or one's complement is being used to represent negative numbers, it is trivial to modify an adder into an adder-subtractor. Other signed number representations require a more complex adder. Q 14 How full adder works? Ans. A full adder adds binary numbers and accounts for values carried in as well as out. A one-bit full adder adds three one-bit numbers, often written as A, B, and Cin; A and B are the operands, and Cin is a bit carried in (in theory from a past addition). Q15 What is counter? Ans. Digital logic and computing, a counter is a device which stores (and sometimes displays) the number of times a particular event or process has occurred, often in relationship to a clock signal. In practice, there are two types of counters: Up counters, which increase (increment) in value Down counters, which decrease (decrement) in value Q 16 Give the truth table for JK flip flop? Ans.
Q 0 0 0 0 1 1 1 1 J 0 0 1 1 0 0 1 1 K 0 1 0 1 0 1 0 1 Q(t+1) 0 0 1 1 1 0 1 0

177 | P a g e

Q 17 WRT to JK FF , what is racing? Ans. Because of the feedback connection in the JK FF, When both J and K are equal to 1 at the time, the output will be implemented while activating the clock pulse. The output is complemented again and again if the pulse duration of the clock signal is greater than the signal propagation delay of the JK FF for this particularinput combination (j=k=1) There is a race between 0 and 1 within a single clock pulse . This conditon of the JK FF is called race around condition. Q 18 Define combinational circuits? Ans . A combinational circuit consists of logic gates whose outputs at anytime are Determined from the present combination of inputs. A combinational circuit performs an operation that can be specified logically by a set of Boolean functions. It consists of input variables, gates and output variables. Q19 What is Microprocessors(8085)? Ans. microprocessor incorporates most or all of the functions of a computer's central processing unit (CPU) on a single integrated circuit (IC, or microchip).The first microprocessors emerged in the early 1970s and were used for electronic calculators using binary-coded decimal(BCD) arithmetic on 4-bit words. Other embedded uses of 4-bit and 8-bit microprocessors, such as terminals, printers, various kinds of automation etc., followed soon after. Affordable 8-bit microprocessors with 16-bit addressing also led to the first general-purpose microcomputers from the often constructed out of small and medium-scale ICs containing from tens to a few hundred transistors. The integration of a whole CPU onto a single chip greatly reduced the cost of processing power. The Intel 8085 is an 8bit microprocessor introduced by Intel in 1977. It was binary-compatible with the more-famous Intel 8080 but required less supporting hardware, thus allowing simpler and less expensive microcomputer systems to be built.The "5" in the model number came from the fact that the 8085 requires only a +5-volt (V) power supply rather than the +5V, -5V and +12V supplies the 8080 needed. Both processors were The 8085 had a long life as a controller. Q 20 Define Microcontroller? Ans. Microcontroller (sometimes abbreviated C, uC or MCU) is a small computer on a single integrated circuit containing a processor core, memory, and programmable input/output peripherals. Program memory in the form of NOR flash or OTP ROM is also often included on chip, as well as a typically small amount of RAM. Microcontrollers are designed for embedded applications, in contrast to the microprocessors used in personal computers or other general purpose applications. Microcontrollers are used in automatically controlled products and devices, such as automobile engine control systems, implantable medical devices, remote controls, office machines, appliances, power tools, and toys. By reducing the size and cost compared to a design that uses a separate microprocessor, memory, and input/output devices, microcontrollers make it economical to digitally control even more devices and processes. Mixed signal microcontrollers are common, integrating analog components needed to control non-digital electronic systems.

PART B
1. Write full form of EBCDIC, BCD, and ASCII? Ans. EBCDIC: Extended Binary Coded Decimal Interchange Code BCD : Binary Coded Decimal ASCII : American Standard Code for Information Interchange
178 | P a g e

2. Convert the following numbers into binary: 345,344. Ans. (101011001)2, (101011000)2, 3. Convert the following numbers into decimal : (1100101)2 ,(0011101010)2. Ans. 101, 234. 4. Convert the following numbers into octal: (1100101)2 , (011101010)2. Ans. (145)8 , (352)8. 5. Convert the following hexadecimal numbers into binary: (2D4B)16 , (9BC)16. Ans. (0010110101001011)2, (100110111100)2. 6. State and prove de- Morgans Theorem. Ans. DeMorgan's Theorems: a. (A + B) = A* B b. A*B = A + B Note: * = AND operation Proof of DeMorgan's Theorem (b): For any theorem X=Y, if we can show that X Y = 0, and that X + Y = 1, then by the complement postulates, A A = 0 and A + A = 1, X = Y. By the uniqueness of the complement, X = Y. Thus the proof consists of showing that (A*B)*( A + B) = 0; and also that (A*B) + ( A + B) = 1.

Prove: (A*B)*( A + B) = 0 (A*B)*( A + B) = (A*B)*A + (A*B)*B) by distributive postulate = (A*A)*B + A*(B*B) by associativity postulate = 0*B + A*0 by complement postulate =0+0 by nullity theorem =0 by identity theorem (A*B)*( A + B) = 0 Q.E.D. Prove: (A*B) + ( A + B) =1 (A*B) + ( A + B) =(A + A + B))*(B + A + B) by distributivity B*C + A = (B + A)*(C + A) (A*B) + ( A + B) =(A + A + B))*(B + B + A) by associativity postulate =(1 + B)*(1 + A) by complement postulate =1*1 by nullity theorem =1 by identity theorem (A*B) + ( A + B) =1 Q.E.D. Since (A*B)*( A + B) = 0, and (A*B) + ( A + B) =1, A*B is the complement of A + B, meaning that A*B=(A + B)'; (note that ' = complement or NOT - double bars don't show in HTML) Thus A*B= (A + B)''. The involution theorem states that A'' = A. Thus by the involution theorem, (A + B)'' = A + B. Q 7. What are the differences between sequential circuit and combinational circuit?
179 | P a g e

Ans. Sequential circuits are the circuits which depends upon previous output to give you the current output..in short we can say that sequential circuit must have a memory element and a combinational circuit..while combinational circuit is just a simple circuit which we get by combining different gates..so difference comes only with a memory element( like flip flop). Q 8. Write the name of different types of basic logic gates? Ans. AND,OR, NOT, NAND, XOR, XNOR etc. Q 9. Write truth table for Basic Logic Gates e.g.; AND, OR , NOT, NAND, XOR, XNOR. Ans. Q 10.Explain the following terms counter, multiplexer and encoders. Ans. Counter: counter is a device which stores (and sometimes displays) the number of times a particular event or process has occurred, in relationship to a clock signal. Multiplexer: It is a device that performs multiplexing; it selects one of many analog or digital input signals and forwards the selected input into a single line. A multiplexer of 2 n inputs has n select lines, which are used to select which input line to send to the output. Encoder: Encoder is a device, circuit, transducer, software program, algorithm or person that converts information from one format or code to another, for the purposes of standardization, speed, secrecy, security, or saving space by shrinking size.

Q 11. Explain the following terms flip flop and Registers. Ans. flip flop : flip-flop is a circuit that has two stable states and can be used to store state information.A flip-flop is usually controlled by control signals that can include a clock signal. The outputs usually include the complement as well as the normal output.Today the word latch is mainly used for simple transparent storage elements, while slightly more advanced nontransparent (or clocked) devices are described as flip-flops. Informally, as this distinction is quite new, the two words are sometimes used interchangeably.

Registers: is a small amount of storage available on the CPU whose contents can be accessed more quickly than storage available elsewhere. Typically, this specialized storage is not considered part of the normal memory range for the machine. Most, but not all, modern computers adopt the socalled load-store architecture. Under this paradigm, data is loaded from some larger memory be it cache or RAM into registers, manipulated or tested in some way (using machine instructions for arithmetic/logic/comparison) and then stored back into memory, possibly at some different location. Q 12. Explain the following terms half adder and full adder. Ans . Half adder adds two one-bit binary numbers A and B. It has two outputs, S and C (the value theoretically carried on to the next addition); the final sum is 2 C + S. The simplest half-adder design, pictured on the right, incorporates an XOR gate for S and an AND gate for C. Half adders cannot be used compositely, given their incapacity for a carry-in bit. Full adder adds binary numbers and accounts for values carried in as well as out. A one-bit full adder adds three one-bit numbers, often written as A, B, and Cin; A and B are the operands, and Cin is a bit carried in (in theory from a past addition). The circuit produces a two-bit output sum
180 | P a g e

typically represented by the signals Cout and S, where . The one-bit full adder's truth table is: Q13. What do you understand by Minimization Techniques. Ans. Minimization Technique refers to simplification of complex boolean expressions into simpler ones.Various methods used for this are Karnaugh map, Quine Mac Clusky Techniques etc Q 14. Describe the importance of dont care conditions? Ans. Karnaugh maps also allow easy minimizations of functions whose truth tables include "don't care" conditions (that is, sets of inputs for which the designer doesn't care what the output is) because "don't care" conditions can be included in a ring to make it larger. They are usually indicated on the map with a dash or X.The example to the right is the same above example but with minterm 15 dropped and replaced as a don't care. This allows the red term to expand all the way down and, thus, removes the green term completely.This yields the new minimum equation:te that the first term is just A not . In this case, the don't care has dropped a term (the green); simplified another (the red); and removed the race hazard (the yellow as shown in a following section).Also, since the inverse case no longer has to cover minterm 15, minterm 7 can be covered with rather than with similar gains. Q 15. What are the differences between SOP and POS. Ans. SOP: Sum of Products POS: Product of Sum Q 16. What is Microprocessors? Ans. Microprocessor incorporates most or all of the functions of a computer's central processing unit (CPU) on a single integrated circuit (IC, or microchip). The first microprocessors emerged in the early 1970s and were used for electronic calculators, using binary-coded decimal(BCD) arithmetic on 4-bit words. Other embedded uses of 4-bit and 8-bit microprocessors, such as terminals, printers, various kinds of automation etc., followed soon after. Affordable 8-bit microprocessors with 16-bit addressing also led to the first general-purpose microcomputers., computer processors were often constructed out of small and medium-scale ICs containing from tens to a few hundred transistors. The integration of a whole CPU onto a single chip greatly reduced the cost of processing power. From these humble beginnings, continued increases in microprocessor capacity have rendered other forms of computers almost completely obsolete (see history of computing hardware), with one or more microprocessors used in everything from the smallest embedded systems and handheld devices to the largest mainframes and supercomputers. The increase in capacity of microprocessors has been a consequence of Moore's Law, which suggests that the number of transistors that can be fitted onto a chip doubles every two years. Although originally calculated as a doubling every year, Moore later refined the period to two years. It is often incorrectly quoted as a doubling of trans

181 | P a g e

Q 17. What is the difference between 8080 and 8085? Ans. 8080 8080has 40-pin DIP packaging permitted it to provide a 16-bit address bus and an 8-bit data bus, allowing easy access to 64 kilobytes of memory. The processor had seven 8-bit registers, (A, B, C, D, E, H, and L) where A was the 8-bit accumulator and the other six could be used as either byte-registers or as three 16-bit register pairs (BC, DE, HL) depending on the particular instruction. Some instructions also enabled HL to be used as (a limited) 16-bit accumulator, and a pseudo register, M, could be used almost anywhere that any other register could be used and referred to the memory address pointed to by HL. It also had a 16-bit stack pointer to memory (replacing the 8008's internal stack), and a 16-bit program counter. 8085 Intel 8085 is an 8-bit microprocessor introduced by Intel in 1977The processor has seven 8-bit registers named A, B, C, D, E, H, and L, where A is the 8-bit accumulator and the other six can be used as independent byte-registers or as three 16-bit register pairs, BC, DE, and HL, depending on the particular instruction. Some instructions use HL as a (limited) 16-bit accumulator. As in the 8080, the contents of the memory address pointed to by HL could be accessed as pseudoregister M. It also has a 16-bit stack pointer to memory (replacing the 8008's internal stack), and a 16-bit program counter. HL pair is called the primary data pointers.

Q18. Underline the Characteristics of flags used in microprocessor 8085? Ans. The flags register in the 8085 contains various statuses...
CARRY - Carry or borrow resulted from addition or subtration SIGN - Result is negative ZERO - Result is zero PARITY - Result has an even number of one's AUXCARRY - Result has a carry out of bit 3 - Use to support BCD arithmetic Each flag is set or reset, for certain instructions only, based on the result of that instruction

Q19. Expand the following terms related with computer and its memory BIOS,FF, CPU,ALU, RAM,SRAM,SRAMROM,PROM and EEPROM. Ans. BIOS: Basic Input Output System FF: Flip flop, CPU:Central Processing Unit , ALU Arithmatic logic Unit, RAM: Random Acess Memory, SRAM: Static Random Acess Memory ROM: Read Only Memory, PROM: Programmable Read only memory EEPROM: Electrically Erasable Programmable Read only memory, Q20.Write full names of LSB, MSB , BPP? Ans. LSB: Least Significant Bit , MSB: Most Significant Bit , BPP: Bits Per Pixel

182 | P a g e

UNIT V Telecommunication Systems and Modern Techniques (TSMT): PART A Q1.What do you mean by radio communication? Ans. The process sending audio signal from a source to distant destination using carrier wave is called radio communication. Q2. Explain communication system with the help of block diagram? Ans. Source
Information

destination
transmitter Tx channel receiver
Information

Q3.Wite name of sub units of radio communication system? i. Transmitters ii. Transmission Channel iii. Receiver Q 4 What do you understand by modulation? Ans. The process of varying one of the characteristics of a high frequency wave ai accordance with the instantaneous value of a low frequency signal is called modulation. Q5 What is modulating wave? Ans. Low frequency signal is called modulating wave. Q 6 What is carrier wave? Ans. High frequency wave is called carrier wave. Q7 What are different types of modulation? Ans. i. Amplitude modulation (AM) ii. Frequency Modulation (FM) iii. Phase modulation (PM) Q8 Give the definition of Amplitude modulation (AM)? Ans . Amplitude modulation (AM) is a technique used in electronic communication, most commonly for transmitting information via a radio carrier wave. AM works by varying the strength of the transmitted signal in relation to the information being sent. Q9 Give the definition of Frequency Modulation (FM). Ans. Frequency modulation (FM) conveys information over a carrier wave by varying its instantaneous frequency in accordance with the frequency of the carrier wave. Q10 What is modulation index? Ans. Modulation index of amplitude modulation is defined as the ratio of the amplitude of the modulating wave to the amplitude of the carrier wave ma =
Percentage ma = *100 % 11.

183 | P a g e

Q11 what are the disadvantages of AM? i. Smaller operating range ii. Poor audio Quality iii. Low Efficiency iv. Reception noisy Q 12 What are the advantages of FM? Ans. i. Noiseless Reception ii. Better Quality iii. High operating range iv. Efficiency is very high Q 13. What are the disadvantages of FM? Ans. i. Costly equipments ii. Smaller area of reception iii.Much wider channel is required Q 14. What is demodulation? Ans. The process of recovering the audio signal from modulated wave is known as demodulation. Or detection. Thus demodulation is reverse process of modulation . Q 15 Give the definition of phase modulation (PM)? Ans. In this he phase of the carrier wave is varied in accordance with the instantaneous value of modulating signal. Q 16 What GSM and CDMA stands for? Ans. GSM stands for Global system for mobile communication. CDMA stands for Code division multiple access. Q 17 What do you understand by the term television. Ans. Television (TV) is the most widely used telecommunication medium for transmitting and receiving moving images that are either monochromatic ("black and white") or color, usually accompanied by sound. "Television" may also refer specifically to a television set, television programming or television transmission. Q 18 Determine the size of antenna working at frequency of 20 kHz and wave length of wave 1.5*106 m given that velocity of light to be3*108 m/s . Ans. = c/f = 3*108 m/s/20*103 = 15*103 m Size of antenna = /4 = 3750 m. ans. Q 19 What is super heterodyne radio receiver ? Ans. It is an improved radio receiver which employs the principle of frequency translation).

heterodyning (means

184 | P a g e

Q 20. What is the basic principle of communication satellites? Ans. communication satellites has uplink and down link frequency and used for various research works and weather forecasting.

PART B
1. What do you understand by the term communication systems? Ans. Communications system is a collection of individual communications networks, transmission systems, relay stations, tributary stations, and data terminal equipment (DTE) usually capable of interconnection and interoperation to form an integrated whole. The components of a communications system serve a common purpose, are technically compatible, use common procedures, respond to controls, and operate in unison. Telecommunications is a method of communication (e.g., for sports broadcasting, mass media, journalism, etc.).A communications subsystem is a functional unit or operational assembly that is smaller than the larger assembly under consideration 2. What is Modulation and why it is needed? Ans. The process of varying one of the characteristics of a high frequency wave insss accordance with the instantaneous value of a low frequency signal is called modulation. The size of antenna that is required for transmission can be revealed from the fact that EMW with higher frequency requires its transmission according to the formula and answer of question no. 18 (section A above) Which is practically not possible , we can not transmit directly with the help of antenna and hence Modulation is required. 3. What are the types of modulation? Ans. Following are the types of modulation: AM, FM, PM etc. 4. Deduce the expression for modulation index for AM. Ans. Modulation index of amplitude modulation is defined as the ratio of the amplitude of the modulating wave to the amplitude of the carrier wave ma = Percentage ma =
*100 % .

5. Explain the advantages of FM over AM. Ans. Less noissy Reception Better Quality High operating range Efficiency is higher than AM. 6. Derive the expression for modulation index or index of modulation for FM? Ans. Modulation index or index of modulation for FM is given by Mf = Change in frequency over original frequency of the modulating wave = vf / fm.

185 | P a g e

Q7. Determine the size of antenna working at frequency of 20 kHz and wave length of wave 2.5*106 m given that velocity of light to be 3*108 m/s .Comment on the result obtained. Ans. . = c/f = 3*108 m/s/20*103 =15000m Size of antenna= 15000/4 = 3750 m This size of antenna is practically not possible , we can not transmit directly with the elp of antenna and hence Modulation is required. Q 8. Explain the principle of mobile communication . Ans. Mobile Communications Principles Each mobile uses a separate, temporary radio channel to talk to the cell site. The cell site talks to many mobiles at once, using one channel per mobile. Channels use a pair of frequencies for communication. One frequency (the forward link) for transmitting from the cell site and one frequency (the reverse link) for the cell site to receive calls from the users. Radio energy dissipates over distance, so mobiles must stay near the base station to maintain communications. The basic structure of mobile networks includes telephone systems and radio services. Where mobile radio service operates in a closed network and has no access to the telephone system, mobile telephone service allows interconnection to the telephone network. Q 9. Find the wave length of electromagnetic wave at 30 KHz frequency given that velocity of light is 3*108 m/s . Ans. 10000 meters. Q 10. Explain multiple accessing techniques TDMA, FDMA. Ans. FDMA: Frequency Division Multiple Access (FDMA) is the most common analog system. It is a technique whereby spectrum is divided up into frequencies and then assigned to users. With FDMA, only one subscriber at any given time is assigned to a channel. The channel therefore is closed to other conversations until the initial call is finished, or until it is handed-off to a different channel. A full-duplex FDMA transmission requires two channels, one for transmitting and the other for receiving. FDMA has been used for first generation analog systems. TDMA: Time Division Multiple Access (TDMA) improves spectrum capacity by splitting each frequency into time slots. TDMA allows each user to access the entire radio frequency channel for the short period of a call. Other users share this same frequency channel at different time slots. The base station continually switches from user to user on the channel. TDMA is the dominant technology for the second generation mobile cellular networks. Q11. Explain the difference between GSM and CDMA? Ans. cellular service there are two main competing network technologies: Global System for Mobile Communications (GSM) and Code Division Multiple Access (CDMA). Cellular carriers including Sprint PCS, Cingular Wireless, Verizon and T-Mobile use one or the other. Understanding the difference between GSM and CDMA will allow you to choose a carrier that uses the preferable network technology for your needs.

186 | P a g e

The GSM Association is an international organization founded in 1987, dedicated to providing, developing, and overseeing the worldwide wireless standard of GSM. CDMA, a proprietary standard designed by Qualcomm in the United States, has been the dominant network standard for North America and parts of Asia. However, GSM networks continue to make inroads in the United States, as CDMA networks make progress in other parts of the world. There are camps on both sides that firmly believe either GSM or CDMA architecture is superior to the other. That said, to the noninvested consumer who simply wants bottom line information to make a choice, the following considerations may be helpful. Coverage: The most important factor is getting service in the areas you will be using your phone. Upon viewing competitors' coverage maps you may discover that only GSM or CDMA carriers offer cellular service in your area. If so, there is no decision to be made, but most people will find that they do have a choice. Data Transfer Speed: With the advent of cellular phones doing double and triple duty as streaming video devices, podcast receivers and email devices, speed is important to those who use the phone for more than making calls. CDMA has been traditionally faster than GSM, though both technologies continue to rapidly leapfrog along this path. Both boast "3G" standards, or 3rd generation technologies.

Q12. Calculate modulation index when amplitude of the signal is equal to the carrier amplitude. Ans. Zero. Q13 .An AM modulated signal has maximum voltage level of 6 milli volts and minimum voltage level of 2 milli volts. Find the modulation index. Ans. modulation index=50%. Q14. Find modulation factor, when signal amplitude is 1.5 times the carrier amplitude. Ans. modulation factor= 0.2. Q15.What are the advantages and disadvantages of FM? Ans. Advantages : Noiseless Reception Better Quality High operating range Efficiency is very high Disadvantages: Costly equipments Smaller area of reception Much wider channel is required Q16. Define uplink and downlink frequencies? Ans. Satellite telecommunication, a downlink is the link from a satellite down to one or more ground stations or receivers, and an uplink is the link from a ground station up to a satellite. Some companies sell uplink and downlink services to television stations, corporations, and to other telecommunication carriers. A company can specialize in providing uplinks, downlinks, or both.
187 | P a g e

The following table shows the main frequency bands used for satellite links.

Frequency Band C Ku Ka

Downlink 3,700-4,200 MHz 11.7-12.2 GHz 17.7-21.2 GHz

Uplink 5,925-6,425 MHz 14.0-14.5 GHz 27.5-31.0 GHz

The C band is the most frequently used. The Ka and Ku bands are reserved exclusively for satellite communication but are subject to rain attenuation. Some satellites carry transponders for both C and Ku bands.

Q17. How communication satellites work? Ans. Communications satellite is a radio relay station in orbit above the earth that receives, amplifies, and redirects analog and digital signals carried on a specific radio frequency. In addition to communications satellites, there are other types of satellites:

Weather satellites: These satellites provide meteorologists with scientific data to predict weather conditions and are equipped with advanced instruments Earth observation satellites: These satellites allow scientists to gather valuable data about the earth's ecosystem Navigation satellites: Using GPS technology these satellites are able to provide a person's exact location on Earth to within a few meters An orbit is the path that a satellite follows as it revolves around Earth. In terms of commercial satellites, there are three main categories of orbits: Geosynchronous Orbit (GEO): 35,786 km above the earth

Orbiting at the height of 22,282 miles above the equator (35,786 km), the satellite travels in the same direction and at the same speed as the Earth's rotation on its axis, taking 24 hours to complete a full trip around the globe. Thus, as long as a satellite is positioned over the equator in an assigned orbital location, it will appear to be "stationary" with respect to a specific location on the Earth. A single geostationary satellite can view approximately one third of the Earth's surface. If three satellites are placed at the proper longitude, the height of this orbit allows almost all of the Earth's surface to be covered by the satellites.
188 | P a g e

Medium Earth Orbit (MEO): 8,000-20,000 km above the earth

These orbits are primarily reserved for communications satellites that cover the North and South Pole Unlike the circular orbit of the geostationary satellites, MEO's are placed in an elliptical (ovalshaped) orbit Low Earth Orbit (LEO): 500-2,000 km above the earth

These orbits are much closer to the Earth, requiring satellites to travel at a very high speed in order to avoid being pulled out of orbit by Earth's gravity At LEO, a satellite can circle the Earth in approximately one and a half hours GEO vs. MEO vs. LEO Most communications satellites in use today for commercial purposes are placed in the geostationary orbit, because of the following advantages:

One satellite can cover almost 1/3 of Earth's surface, offering a reach far more extensive than what any terrestrial network can achieve Communications require the use of fixed antennas. Since geosynchronous satellites remain stationary over the same orbital location, users can point their satellite dishes in the right direction, without costly tracking activities, making communications reliable and secure GEO satellites are proven, reliable and secure - with a lifespan of 10-15 years For a more comprehensive understanding of satellite advantages, see benefits of satellite. Satellite Architecture Communications data passes through a satellite using a signal path known as a transponder. Typically satellites have between 24 and 72 transponders. A single transponder is capable of handling up to 155 million bits of information per second. With this immense capacity, today's communication satellites are an ideal medium for transmitting and receiving almost any kind of
189 | P a g e

content - from simple voice or data to the most complex and bandwidth-intensive video, audio and Internet content. Diagrammatic Representation of a Satellite

Orbital Location and Footprint The location of a geostationary satellite is referred to as its orbital location. International satellites are normally measured in terms of longitudinal degrees East ( E) from the Prime Meridian of 0 (for example, Intelsat's IS-805 satellite is currently located at 304.5 E). The geographic area of the Earth's surface over which a satellite can transmit to, or receive from, is called the satellite's "footprint." The footprint can be tailored to include beams with different frequencies and power levels. Frequency Bands and Beams Satellites transmit information within radio frequency bands. The frequency bands most used by satellite communications companies are called C-band and the higher Ku-band. Over the next several years, the use of a higher frequency band known as Ka-band is expected to increase. Modern satellites are designed to focus on different ranges of frequency bands and different power levels at particular geographic areas. These focus areas are called beams. Intelsat offers four beam types:

Global: covering almost 1/3 of Earth's surface Hemi: covering almost 1/6 of Earth's surface Zone: covering a large landmass area Spot: covering a specific geographic area

Q18. Briefly explain the working of super heterodyne radio receiver with the help of block diagram. Ans. The diagram below shows the basic elements of a single-conversion superheterodyne receiver. The essential elements of a local oscillator and a mixer followed by a fixed-tuned filter and IF amplifier are common to all superhet circuits. Cost-optimized designs may use one active device for both local oscillator and mixerthis is sometimes called a "converter" stage. One such example is the pentagrid converter.The advantage to this method is that most of the radio's signal path has to be sensitive to only a narrow range of frequencies. Only the front end (the part before the frequency converter stage) needs to be sensitive to a wide frequency range. For example, the front end might need to be sensitive to 130 MHz, while the rest of the radio might need to be sensitive only to 455 kHz, a typical IF. Only one or two tuned stages need to be adjusted to track over the tuning range of the receiver; all the intermediate-frequency stages operate at a fixed frequency
190 | P a g e

which need not be adjusted. To overcome obstacles such as image response, multiple IF stages are used, and in some cases multiple stages with two IFs of different values are used. For example, the front end might be sensitive to 130 MHz, the first half of the radio to 5 MHz, and the last half to 50 kHz. Two frequency converters would be used, and the radio would be a "Double Conversion Super Heterodyne"&mdash;a common example is a television receiver where the audio information is obtained from a second stage of intermediate-frequency conversion. Receivers which are tunable over a wide bandwidth (e.g. scanners) may use an intermediate frequency higher than the signal, in order to improve image rejection.Superheterodyne receivers have superior characteristics to simpler receiver types in frequency stability and selectivity. They offer better stability than Tuned radio frequency receivers (TRF) because a tuneable oscillator is more easily stabilized than a tuneable amplifier, especially with modern frequency synthesizer technology. IF filters can give narrower passbands at the same factor than an equivalent RF filter. A fixed IF also allows the use of a crystal filter when exceptionally high selectivity is necessary. Regenerative and super-regenerative receivers offer better sensitivity than a TRF receiver, but suffer from stability and selectivity problems.] these have been replaced with precision electromechanical surface acoustic wave (SAW) filters. Fabricated by precision laser milling techniques, SAW filters are cheaper to produce, can be made to extremely close tolerances, and are stable in operation. To avoid tooling costs associated with these components most manufacturers then tended to design their receivers around the fixed range of frequencies offered which resulted in de-facto standardization of intermediate frequencies.Microprocessor technology allows replacing the superheterodyne receiver design by a software defined radio architecture, where the IF processing after the initial IF filter is implemented in software. This technique is already in use in certain designs, such as very low-cost FM radios incorporated into mobile phones, since the system already has the necessary microprocessor.Radio transmitters may also use a mixer stage to produce an output frequency, working more or less as the reverse of a superheterodyne receiver. Q19. Briefly discuss the principle of mobile communication? Ans. Each mobile uses a separate, temporary radio channel to talk to the cell site. The cell site talks to many mobiles at once, using one channel per mobile. Channels use a pair of frequencies for communication. One frequency (the forward link) for transmitting from the cell site and one frequency (the reverse link) for the cell site to receive calls from the users. Radio energy dissipates over distance, so mobiles must stay near the base station to maintain communications. The basic structure of mobile networks includes telephone systems and radio services. Where mobile radio service operates in a closed network and has no access to the telephone system, mobile telephone service allows interconnection to the telephone network. The cellular concept structured the mobile telephone network in a different way. Q20. What do you understand by Multiple-Access Techniques? Ans. Multiple-access techniques: Cellular systems divide a geographic region into cells where a mobile unit in each cell communicates with a base station. The goal in the design of cellular systems is to be able to handle as many calls as possible (this is called capacity in cellular terminology) in a given bandwidth with some reliability. There are several different ways to allow access to the channel. These include the following.
o

frequency division multiple-access (FDMA) time division multiple-access (TDMA) time/frequency multiple-access random access code division multiple-access (CDMA) frequency-hop CDMA

191 | P a g e

o o

direct-sequence CDMA multi-carrier CDMA (FH or DS)

As mentioned earlier, FDMA was the initial multiple-access technique for cellular systems. In this technique a user is assigned a pair of frequencies when placing or receiving a call. One frequency is used for downlink (base station to mobile) and one pair for uplink (mobile to base). This is called frequency division duplexing. That frequency pair is not used in the same cell or adjacent cells during the call. Even though the user may not be talking, the spectrum cannot be reassigned as long as a call is in place. Two second generation cellular systems (IS-54, GSM) use time/frequency multiple-access whereby the available spectrum is divided into frequency slots (e.g., 30 kHz bands) but then each frequency slot is divided into time slots. Each user is then given a pair of frequencies (uplink and downlink) and a time slot during a frame. Different users can use the same frequency in the same cell except that they must transmit at different times. This technique is also being used in third generation wireless systems (e.g., EDGE). Code division multiple-access techniques allow many users to simultaneously access a given frequency allocation. User separation at the receiver is possible because each user spreads the modulated waveform over a wide bandwidth using unique spreading codes. There are two basic types of CDMA. Direct-sequence CDMA (DS-CDMA) spreads the signal directly by multiplying the data waveform with a user-unique high bandwidth pseudo-noise binary sequence. The resulting signal is then mixed up to a carrier frequency and transmitted. The receiver mixes down to baseband and then re-multiplies with the binary { 1} pseudo-noise sequence. This effectively (assuming perfect synchronization) removes the pseudo-noise signal and what remains (of the desired signal) is just the transmitted data waveform. After removing the pseudo-noise signal, a filter with bandwidth proportional to the data rate is applied to the signal. Because other users do not use completely orthogonal spreading codes, there is residual multiple-access interference present at the filter output. This multiple-access interference can present a significant problem if the power level of the desired signal is significantly lower (due to distance) than the power level of the interfering user. This is called the near-far problem. Over the last 15 years there has been considerable theoretical research on solutions to the near-far problem beginning with the derivation of the optimal multiuser receiver and now with many companies (e.g., Fujitsu, NTT DoCoMo, NEC) building suboptimal reduced complexity multiuser receivers. The approach being considered by companies is either successive interference cancellation or parallel interference cancellation. One advantage of these techniques is that they generally do not require spreading codes with period equal to the bit duration. Another advantage is that they do not require significant complexity (compared to a minimum mean square error-MMSE-detector or a decorrelating detector). These interference cancellation detectors can also easily be improved by cascading several stages together.

192 | P a g e

You might also like